You are on page 1of 131

ÜNİVERSİTELER İÇİN

TEMEL FİZİK

Sinan KAYAYURT
İbrahim ÇİÇEK
ÜNİVERSİTELER İÇİN

TEMEL FİZİK
MEKANİK SERİSİ

Sinan KAYAYURT
İbrahim ÇİÇEK

İstanbul Teknik Üniversitesi


Uçak ve Uzay Bilimleri Fakültesi
2016
İÇİNDEKİLER

1. Birimler, Fisiksel Büyüklükler ve Vektörler

2. Doğrusal Yolda Hareket

3. İki ve Üç Boyutta Hareket

4. Newton’ un Hareket Yasası

5. Newton Yasalarının Uygulanması

6. İş-Güç-Enerji

7. Enerjinin Korunumu

8. İtme-Momentum-Çarpışmalar

9. Katı Cisimlerin Dönme Hareketi

10. Döngüsel Hareketlerin Dinamiği

11. Esnekik ve Denge Şartları

12. Akışkan Mekaniği

13. Kütlesel Çekim Kanunları

14. Harmonik Hareketler


ÖNSÖZ

Öncelikle eğitim hayatımızda ve bu değerli ürünü hazırlama sürecin-


de bizlere desteklerini eksik etmeyen Prf.Dr. Hilmi ÜNLÜ Hocamıza ve
aile fertlerimize sonsuz teşekkürlemizi sunarız.

Bu kitap üniversitelerdeki öğrenci arkadaşlarımıza ve fizik öğrenmek


isteyen diğer arkadaşlara yardımcı olmasını dileyerek yazdığımız bir
üründür. Bizim gibi üniversitelerde okuyan arkadaşları düşünerek sı-
navlara çalışırken konuları daha iyi anlamaları için kitabı sade ve anla-
şılır kılmaya çalıştık. Uzun bir çalışma süreci sonucunda yazdığımız bu
kitap mekanik bölümü olarak fiziğin ilk konularının ele alınmasını uy-
gun gördük. Üniversitelerde özellikle mühendislik ve diğer sayısal bö-
lümlerin temel dersi olan mekaniğin bilindiği kadar zor olmadığını siz-
ler bu kitap ile hazırlanınca bizlere hak vereceksiniz. Fiziksel olayların
iyi incelenip anlaşılması için görsel efektler günlük hayatta karşılaşılan
genel problemler kullandık. Her bölümün sonunda konunun pekişme-
si için hazırladığımız sorularla daha verimli çalışmanızı ümit ediyoruz.
Hayat boyu öğrenmeye olan inancımızla bu kitabın sizlere faydalı ola-
cağını umuyor ve derslerinizde başarılar diliyoruz.
1. BÖLÜM

Birimler, Fisiksel Büyüklükler ve Vektörler

1.1 Fiziğin Doğası


1.2 Fiziksel Problemler ve Çözüm Teknikleri
1.3 Birimler ve Sabitler
1.4 Birimlerin Geçerliliği ve Dönüşümleri
1.5 Vektörler ve Bileşenleri
1.6 Vektörlerle İşlemler
Fiziğin Doğası

Evrende gerçekleşen olayları, olayların so-


nuçlarını ve bu olaylar arkasındaki sebeple-
ri inceleyen bilim dalına FİZİK denir. Fizik
bilimi ile ilgilenen insanlara da FİZİKÇİ
denir. Fizik madde ve enerji arasındaki et-
kileşimi inceler. Doğada gerçekleşen olayla-
ra mantıksal açıklamalar getirmenin yanın-
da yöntemlerle olayların tekrarlanıp çözüm
yollarının iyileştirilmesine olanak sağlayan
uygulamalı (deneysel) bir bilim dalıdır.
Bilim kümülatif geliştiği için fizik de sürekli birikim ve deneyerle
gelişen bir bilim dalı olmuştur. Fiziğin uğraş alanının genişilği evrenin
ölçüsü kadardır. Ama insanoğlu imkanların elverdiği sınırlar çerçeve-
sinde fiziği kullanır ve olayları inceler. Gönümüz teknolojisi ve bilgi
birikimleri belki de fiziğin ulaşabileceği uğraş alanının yüzde onuna
ulaşabilmiştir. Fiziğin uğraş alanı oldukça geniş olduğundan incelene-
cek konular sınıflandırılmış ve fiziğin alt dalları oluşmuştur. İşte gönü-
müzde fiziğin alt dalları:

Bu kitapta ele alınan fiziğin alt dalı mekanik olacaktır. Mekanik,


kuvvet ve hareket arasındaki ilişkiyi inceler. Hareketli olan her sistem
mekaniksel olarak incelenir. Yağmur damlasının düşüşünden bilgisa-
yarı soğutan fanlara, duvara yapışan örümcekten uzaya fırlatılan rokete
hatta Güneş sistemindeki gezegenlere kadar tüm hareketli sistemler
mekaniğin uğraş alanına girer.
BİRİMLER
Bir cismin fiziksel büyüklüğünü veya bir olayın gelişimini ölçmek için
fizikte kullanılan birimler mevcuttur. Bu birimler aşağıda ezberlenmesi
kolay olsun diye başharfleri birleştirilerek bir tamlama elde edilmiştir.
Fizikte büyüklükleri ölçerken bu birimleri kullanırız. Fiziksel büyüklük-
ler temel büyüklükler ve türetilmiş büyüklükler olarak ikiye ayrılır.

Temel büyüklükler: Başka birimlere gerek duyulmadan tek başına


ifade edilince ölçülebilen büyüklüktür. Fizikte 7 tane Temel büyüklük
vardır bunlar ; kütle, uzunluk, zaman, akım şiddeti ve sıcaklık,ışık şid-
deti ve madde miktarıdır.
Temel büyüklük Birimi Gösterimi

Kütle kilogram (kg) m
Işık Şiddeti candela (cd) I
Sıcaklık Kelvin,Celcius (K,C) T
Akım Şiddeti Amper (A) i
Madde miktarı Mol (mol) N
Uzunluk metre (m) x
Zaman saniye (s) t
*Akılda tutmak için baş harflerini birleştirdiğimizde ‘KISA MUZ’ diye
oluştuğuna dikkat edelim.
Türetilmis büyüklükler: Temel büyüklüklerinle oluşturulmuş ve temel
büyüklükler yardımıyla ifade edilen büyüklüklere türetilmiş büyüklük-
ler denir.
Türetilmiş büyüklükler; kuvvet, ivme, hız, direnç, enerji vb.

Kuvvet: Bir cismin ivmelenmesini sağlayan etkidir. F=m.a birimi


N=kg.m.s-2

İvme : Hızın zamanla değişimini sağlayan etkidir. a= DV/Dt birimi


a=m.s-2
Enerji: İş yapabilme yeteneğidir. E=F.x birimi Joule=N.m=kg.m2.s-2
Örnek Türetilmiş Büyüklük: Yoğunluk

“Ne kadar şeyde ne kadar şey var?” sorusuna verilen yanıttır. Örnek
olarak kütle yoğunluğu (halk arasında özkütle olarak da bilinir), ne
kadar hacimde(şeyde) ne kadar kütle(şey) var? Sorusunun cevabı bize
cevabı verecektir. Özkütle ρ ile gösterilir. ρ= m/v olarak ifade edilir. M
burda kütle olup, V hacimdir. Hacim üç tane uzunluğun yani üç bo-
yutun ürünü olduğundan m3 birimlendirilir. Yoğunluğun sorusu “ne
kadar hacimde ne kadar kütle var”a örnek cevap olarak 20 m3 hacimde
40 g kütle var olsun. 20 m3 ‘te 40 g, 40g/(20m3 ) olarak ifade edileceğin-
den bizim yoğunluğumuz (2g)/(1m3 )(Bu ifade 1 m3’te 2g var demektir)
olur ve 2g/m3 olarak gösteririz. Değeri bir olan ifadeleri birim olarak
adlandırabiliriz. Bu durumda yukarıdaki ifadeye birim hacimde 2 g var
deriz. Burdan da şu sonuca varırız. Özkütle birim hacimdeki madde
miktarının kütlesidir. Burda dikkat edilmesi gereken nokta yoğunluğun
daha genel bir ifade olduğudur. Sadece kütle yoğunluğu özkütledir.
Onun dışında her türlü şeyin yoğunluğu olabilir. Mesela 100 m^2 ‘lik
bir salonda 50 kişi olsun bu salonun kişi yoğunluğu nedir desek; yoğun-
luk sorumuzu soralım “ne kadar şey de ne kadar şey var?”: 100m^2 ‘de
50 kişi var.
(50 kişi)/(100m^2 )=(1 kişi)/(2m^2 )(2m^2 ‘de 1 kişi)= (0,5 kişi)/
(1m^2 )=0,5 kişi/m^2
Payda 1 olduğu için birim kelimesini m^2 (2 boyut) olduğu için de alan
ifadesini kullanırsak birim alana 0,5 kişi diye okuruz. Bu da salonun
yoğunluğu olur. Başka bir örnek olarak yük yoğunluğundan bahsedebi-
liriz. Mesela 5C/m^3 yük yoğunluğuna sahip 5 m^3 kürenin net yükü
nedir dersek.
5C/m^3 >> 1m^3’ te 5C yük varsa
5m^3’ te X yük vardır>> X=25Cou-
lomb

Birimlere dikkat edersek onlar çoğu zaman bize olayı anlatır.


Mesela 70km/sa hızla giden bir araç derken 70km/1saat ifadesi , 1 saatte
70 km olarak okunur. Bu da demektir ki araç saatte 70 km yol alıyor.
Birimlerden yola çıkarak formüllerin doğruluğunu kontrol edebilir ya
da formüllere ulaşabiliriz.
ÖRNEK 1.1

Hareketli bir cismin zamanla aldığı yolu bulmak için kullandığımız


X = V.t denkleminin birim analizinin doğruluğunu kontrol ediniz.

ÇÖZÜM :
X’ in birimi (uzunluk):L,Lenght
t’ nin birimi (zaman): T, Time
V’ nin birimi metre/saniye = L/T

x=v.t L=(L/T) *T , L’leri ve T’leri sadeleştirirsek 1=1 çıkar,


bu durumda verilen formül boyutsal olarak doğrudur.
ÖRNEK 1.2

Şekildeki yatay düzlemde r uzunluğundaki iple


kurulmuş düzenekte V hızıyla döndürülen m
kütleli cismin ivmesini bulunuz.

ÇÖZÜM :

İlk adımımız bu sistemde ivmeyi etkileyen şeyler neler olabilir, neyler


ivmenin formülünde yer alabilir bunu düşünmek.
1-m kütleli cisimden bahsediyoruz, m kenarda dursun
2-r uzunluktaki ipten bahsediyoruz,r de kenarda dursun
3-v hızıyla döndürülen bir sistemden bahsediyoruz, v de kenarda
4-bahsetmesek de yerçekimi her zaman olabilir
Bu durumda a=mx.ry.vz.gt diyelim. Ancak birim analizine geçmeden
önce a ve g nin aynı şey olduğundan g yi atalım.
>> a= mx.ry.vz
a>metre/sn2>>L/T2
m>kg>>M
r>metre>>L
v>m/sn>>L/T
L.T-2=Mx.Ly.(L/T)z= Mx.Ly+z.T-z
Eşitliğin sağlanması için x=0, x+z=1, z=2 >> y=-1 buluruz
Burdan da a= m0.r-1.v2>> a=v^2⁄r olarak ivmenin formülü bulunur.
Yalnız bu örnek için olmasa da başında katsayı olabilir, bu yöntemle
katsayıyı bulamayız.
Vektörler

Skaler ve Vektörel Nicelikler


Skaler nicelikler sayı ile belirtilen yönü olmayan büyüklüklerdir. Mesela
2m çubuk dediğimizde bu çubuğun yönünü ifade etmez. Veya bu 2m
orjinden sağa mı sola mı olduğunu bilemeyiz. Dolayısıyla yönü yoktur.
Skaler bir niceliktir.
Vektörel nicelikler büyüklüğünün yanında yönü de belli olan nicelikler-
dir. Mesela Kuzey’e doğru 10m/s hızla giden bir araç derken hız niceli-
ğinin hem yönü hem büyüklüğü belirtilmiştir. Demekki hız vektörel bir
niceliktir.
Birim Vektörler
Birim vektörler büyüklüğü 1 olan vektörlerdir. Yönleri belirle-
mek için kolaylıklık sağlarlar. Uzayda x,y,z eksenlerini ifade etmek için
x ekseni doğrultusunda i birim vektörü, y ekseni doğrultusunda j birim
vektörü ve z ekseni doğrultusunda k birim vektörü kullanılır.
ŞEKİL SF 11.

ÖRNEK 1.3
Kartezyen koordinat düzleminde konumu yarıçap vektörü ile gösteri-
len bir cismin ri= 2i+3j ilk konumundan rs=4i+8j+3k son konumuna
taşınırsa cismin yerdeğiştirmesinin büyüklüğü ne olur?
ÇÖZÜM :

Son konumdan ilk konumu çıkartırsak ne kadar yer değiştirdiğini bul-


muş oluruz. Vektörlerde çıkarma yapılırken aynı doğrultudaki vektörler
birbirinden çıkarılabilir.
rs-ri=4i+8j+3k-(2i+3j)=i(4-2)+j(8-3)+k(3-0)=2i+5j+3k olarak bulunur.
∆r=2i+5j+3k bizim yerdeğişim vektörümüzdür ancak büyüklüğünü
bulmak için Pisagor yapmamız gerekmektedir.

∆r=√(22+52+32 )=4√3
Vektörleri Bileşenlerine Ayırma

R=Rxi + Ryj
R=√(Rx2+Ry2 )
R.cosa=RX bileşenini verir. Bir vektörün hangi bileşenini bulmak is-
tiyorsak onla arasında kalan açının cosinüs değeri ile çarparız. Diğer
bileşeni bulmak için istersek onla arasında kalan açının cosinüsüne ba-
karız istersek de birinci bileşen için kullandığımız açının sinüsünü alırız.
Mesela R yi cosb ile çarparsak R nin b açısına komşu olan bileşenini
bulmuş oluruz. Ya da cosa ile bir bileşeni bulduktan sonra diğer bileşen
için sina ile çarpılarak da bulunabilir. Rcosa=Rx ise Ry=Rsina
|R|=√(Rx2+Ry2 )=√((Rcosα )2+(Rsinα )2 )=√(R2 [(cosα )2+(sinα )2])
(cosα )2+(sinα )2 = 1 trigonometrik dönüşümünden
=|R| olduğunu kanıtlanır.
Bunu 3 boyuta taşırsak

ŞEKİL SF 11(3 BOYUT)


R=Rxi + Ryj + Rzk
Rcosa = Rx (a, R-Rx arasındaki açıdır)
Rcosb = Ry (b, R-Ry arasındaki açıdır)
Rcosq = Rz (q, R-Rz arasındaki açıdır)
R=√(Rx +Ry +Rz )
2 2 2

Skaler Çarpım

A ve B vektörlerinin skaler çarpımı <A.B>=|A|.|B|.cosq şeklinde olup


buradaki q açısı A ve B vektörleri arasındaki açıdır.
ŞEKİL SF 12
ÖRNEK 1.3
A vektörü, A=2 i +3 j +4 k ve B vektörü, B=5 i +6 j +7 k olsun. Bu
durumda A ve B vektörlerinin skaler çarpımı <A.B> sonucu kaçtır?

ÇÖZÜM :
<A.B>=|A|.|B|.cosq formülünü kullanabilmemiz için q gerekli. q’ yı
belirlemek her zaman kolay olmayabilir. O yüzden A’yı B’yi bütün ola-
rak ele almadan teker teker çarpalım.
(2 i + 3 j + 4 k ).(5 i + 6 j +7 k)=
(*)10.i.i=> i.i=|i|.|i|.cos0=1, 10.i.i=10
(**)+12.i.j=>i.j=|i|.|j|.cos90=0, 12.i.j=0
+14.i.k=>i.j=|i|.|k|.cos90=0, 14.i.j=0
+15.j.i+18.j.j+21.j.k+20.k.i+24.k.j+28.k.k=10+18+28=56 bulunur.

*: i vektörü birim vektör olduğundan ve kendisi ile arasındaki


açının da 0 olacağından dolayı i.i skaler çarpımı 1 sonucunu verir.
**: i ve j vektörleri birim vektör olduğundan boyları 1’dir.Ancak bir-
nirlerine dik oldukları için cos90=0 olur. Birbirine dik vektörler skaler
olarak çarpıldığında 0 sonucunu verir. Bu durumda i.i, j.j, k.k çarpımla-
rı hariç 0 olacaktır.
Skaler çarpımın aynı yöndeki vektörleri çarpması bize şöyle bir imkan
sağlamaktadır. Eğer bir vektörün bir doğrultudaki bileşenini veyahut da
iz düşümünü bulmak istersek o doğrultudaki vektörle skaler çarparsak
sadece o yöndeki bileşeni çarpılır. Ancak çarptığımız vektör o doğrultu-
daki birin vektör olmalıdır. Çünkü sadece yön verip büyüklüğü değiştir-
memeli. Mesela A=2 i +3 j vektörünün x yönündeki bileşeni nedir diye
sorulsa. X yönündeki bir vektörü alırız.
A=2i + 3j
X=2i olsun
<A.X>=2.2+3.0=4 olur. Ancak açıkça görülmektedir ki A’nın x bile-
şeni 2’dir. Hatanın çıkma sebebi x yönünde birim vektör almadığımız
içindir. X=i olarak alırsak; <A.X>=2.1+3.0=2 olur. Bu örnek için skaler
çarpımdan gitmeye gerek yok ama daha karmaşık sorukar için güzel bir
bilgidir.
ÖRNEK 1.4

A=5i + 10j vektörünün B=3i+4j vektörü üzerindeki izdüşümü olan


vektörün büyüklüğü nekadardır? Diğer bir ifadeyle A’nın B doğrultu-
sundaki bileşeninin büyüklüğü nedir?
ÇÖZÜM :
Öncelikle B yönündeki birim vektör gerekli. B yönündeki birim vektör
u olsun.
u=B/|B| yaparak yani B vektörünü kendi boyuna bölerek B nin birim
vektörünü elde ederiz. Elde ettiğimiz bu birim vektörü A vektörü ile
skaler (iç çarpım) olarak çarparsak A’ nın B yönündeki bileşenin büyük-
lüğünü buluruz
u=(3i+4j)/5=> <A.u>=5.3/5+10.4/5=11 bulunur.

Vektörel Çarpım

A vektörel çarpım B,
AxB= |A|.|B|.sina.ê
şeklinde olup, ifadedeki a vektörler arasındaki açıdır. ê vektörü ise sağ
el kuralı ile bulunan birim vektördür. Şöyleki 4 parmak birbirine ve A
vektörüne paralel şekilde, baş parmak ise onlara dik bir şekilde tutula-
rak elimiz A vektöründen B vektörüne bir yay çizmek suretiyle(elimizin
iç yönünde) döndürüldüğünde baş parmak bize ê birim vektörünün
yönünü gösterir. ê vektörü A ve B vektörlerine diktir. Ayrıca skaler çar-
pımları da 0 dır. <A.ê>=<B.ê>=0
ÖRNEK 1.5

A=2i + 3j , B=4i+5j ise A ve B vektörlerinin vektörel çarpımını diğer bir


ifadeyle A ve
B ye dik doğrultudaki vektörü bulunuz.

ÇÖZÜM :
Soruda hemen hatırlamamız gereken
bir bilgi var ki o da iki vektörün vektörel
çarpılması ile elde edilen vektör ikisine
de dik yeni bir vektör olacaktır ama boyu
1 olmayabilir. Boyu 1 değilse ki genelde
değildir kendi boyuna böerek boyunuı 1
yapabiliriz.
AxB=(2i + 3j)x(4i + 5j)=8.i x i+10.i x
j+12.j x i + 15.j x j

Elimizi i vektörünün üstüne koyup j’ye


doğru içten çevirdiğimizde başparmak k’ yı
göstermektedir. Yani i x j=k
4 parmağımızı j ye paralel koyup, j’den i’ye
elimizin içi yönünde çevirirsek başparmak
–k’yı göstermektedir.
AxB=10k – 12k=-2k bulunur. Ancak bulduğumuz vektör birim vektör
değildir. Bu yüzden boyuna böleriz. (-2k)/2=-k olarak sonuç çıkar. So-
nuç –k çıksa da + k da aynı doğrultudadır.
Not: Vektörel çarpımlarda şekli düşünmeden yapmak istersek
i j k i j k şeklinde yazarız.

i x j diyorsa yazdığımız yerde i ve j nin yanyana olduğu duruma bakarız.


Bu çarpım sağa doğru oluyorsa sonuç pozitiftir deriz ve sağa doğru sıra-
daki vektör cevap olur.
Bu çarpım sola doğru oluyorsa sonuç negatiftir deriz ve sola doğru sıra-
daki vektör cevap olur.
Örnek olarak k x j vektörünü bulmak
isteyelim.
i j k i j k şeklinde yazıyoruz. k ve j
nin yanyana olduğu yere baktığımızda k x
j sola doğru gidiyor. Bu durumda sonuç
negatif olacak. Ayrıca sıradaki vektör de i
olduğundan sonucumuz –i olacak.
k x j= -i
Bunu başka bir pratik yolla daha yapabiliriz. Bir çemberin üstünde
herhangi üç noktaya i, j ve k vektörlerini koyarız. Yalnız dikkat etmemiz
gereken şey bir yönü pozitif seçip i,j,k sırasıyla o yönde yazılmalı. Bunu
yaptıktan sonra çarpım bulunurken istenilen vektörler seçtiğimiz pozi-
tif yönde mi yoksa tersine mi çarpılıyor ona bakarak sonucun pozitif mi
negatif mi olduğunu söyleriz ve sıradaki vektörü cevap olarak yazarız.
Şeklinde saat yönünü pozitif seçerek yerleştirelim. Şimdi bu yoldan k x
j yi bulmaya çalışalım. k x j görüldüğü üzere k dan j ye gidiş seçtiğimiz
yönün tersinedir. Bu durumda sonuç negatif ve bu yönde sıradaki vek-
tör i olduğundan sonuç olarak k x j=-i buluruz.

Matrisi yöntemi ile vektörel çarpım

A ve B vektörleri aşağıdaki gibi olsun


A=2i + 3j + 4k , B=5i + 6j + 7k bunların vektörel çarpımlarını (AxB)
matris yöntemi ile bulalım.
Gerçek şu ki bunları tek tek çarpıp ne çıkacağına bakmaya üşenebiliriz.
Bu yüzden matris yöntemi daha cazip olabilir.
Şöyle ki

=-3i+6j-3k şeklinde matris yöntemi ile bulunabilir. Bu yöntemi iki bo-


yutta da uygulayabiliriz.
Bölüm Sonu Soruları
1- SI birim sisteminde tork un birimi nedir?
2- Kütlenin birimi: kg, uzunluk birimi: m, zamanın birimi: s, alarak
Göç=Enerji/zaman olacak şekilde göcün birimini türetiniz.

3- Kuvvet ile zamanın çarpımı cismin momentunu değiştiren itme


niceliğini verdiğine göre ( I = F.t) bu niceliğin birmini bulunuz ve bu
nicelik vektörel mi skaler mi belirtiniz.
4- Uzayda üç ayrı vektörün toplamı sıfır ise bu vektörle aynı düzlemde
olmak zorunda mıdır? Cevabınıza uygun bir açıklama yapınız.
5- Alüminyum yoğunluğu: ρAl=2,7 g/cm3
Kurşunun yoğunluğu : ρPb=11,3 g/cm3 olduğuna göre 10 cm3 Al ve
30 cm3 Pb den oluşan alaşımın yoğunluğu kaç g/ cm3 tür?

6- Uzayda A= 3i + 2j + k , B= i + 4j -3k vektörleri verildiğine göre bu


vektörlerin skaler çarpımı (iç çarpım), A.B , ve vektörel çarpımı, A x B ,
bulunuz.
7- Üç vektör: A = i +j +k, B= xi -2j +2k, C= -i +2j + 2k olarak verilsin.
Eğer A ile B arasındaki açının kosinüsü 1/3 e eşit ise (arccosq=1/3);

a- x’ i bulunuz.
b- |A|’ yı ( A vektörünün normu) bulunuz.
c- A x C’ yi ( A vektörel çarpım C ) bulunuz.

8- A= (0, 2, 1 ) vektörünün B= (Ö2, 1, 1) vektörü doğrultusundaki


izdüşüm vektörünü bulunuz.

9- U = (a, 2) ve V = (2, a) vektörleri arasındaki dar açı 600 ise a=?

10- A=(1, 2, 0) ve B= (-1, 3, 2) vektörlerinin vektörel çarpımı C


wvektörüne eşittir (A x B = C). C vektörünün uzunluğunu(normu-
nu),|C|, bulunuz.
2. BÖLÜM

Doğrusal Yolda Hareket

2.1 Yerdeğiştirme, Zaman ve Oratalama Hız


2.2 Anlık Hız
2.3 Fizik için Matematik: Diferansiyel,Türev
2.4 İvme
2.5 Serbest Düşme Hareketi
2.6 Konumun-Hız-İvme Bağıntıları
Bölüm Sonu Soruları
Yerdeğiştirme, Zaman ve Oratalama Hız
Bulunduğumuz noktayı herhangi bir referans sistemine göre tarif
edersek başka bir noktaya göre uzaklığı dile getiririz. Konum, an-
lık bulunan noktanın referans noktasına göre durumudur. Hareketli
cisimlerin belirli bir süre boyunca konumu değişir. Son konum ve ilk
konum arasındaki en kısa mesafe farkına (kestirme yol da denilebilir)
yerdegistirme denir. (r2 - r1 = Dx )
Hareket genel anlamda konumdaki değişimdir. Konumumuzda olan bu değişim belli bir süre içinde gerçek-
leşmektedir. Hareketimizin ne kadar sürede gerçekleştiğini bulursak hızımızı da hesaplayabiliriz. Hız belli
bir zamanda yapılan konumdaki değişimdir. Mesela 10 sn’de 20 m yer değiştirirsek hızımızı 20m/10s=2m/s
olur. Yani saniyede 2 m yer değiştiriyoruz demektir. Yukarda anlattığımız bilgiler ışığında hızın formülünü
v=Dx/Dt olarak bulabiliriz. Burada ∆x yerdeğiştirme olup ∆t ise hareketin ne kadar sürede gerçekleştiğidir.
v= Dx/Dt
ÖRNEK 2.1
20m/s hızla giden bir A aracı kendinden 50 m ileride olan B aracını 10 saniyede yakalıyor. Buna göre B aracı-
nın hızı nedir? (Araçlar aynı yönlü, sabit hızlı ve noktasal düşünülecektir.)
ÇÖZÜM
A aracı 20m/s ile B yi yakalamak isterken B aracı v hızı ile A dan uzaklaşıyor. Bu durumda aradaki fark (20-v)
m/s hızı ile kapanıyor.
50m’lik mesafede 20-v hızıyla kapanmış
v= Dx/Dt = 50m/10s=(20-v)m/s=> v=15m/s
ÖRNEK 2.2
Saniyede 20m yer değiştiren bir A aracı KLM yolunu 10s’de alıyor. Başka bir araç ise LM yolunu 10s’de alıyor.
|KL|=|LM| ise diğer aracın hızı nedir?

ÇÖZÜM
1 saniyede 20m ise
10 saniyede x=? x=200m=|KLM|ise |LM| yolu 100m olur.
ŞEKİL SF 4
|LM|=100m=v.10s=> v=10m/s

Anlık Hız
Şimdiye kadar bahsettiğimiz kısımda cisimler sabit hızlı hareket ediyordu. Sabit hızlı hareketle cisim hiç hız
değiştirmediği için herhangi bir andaki hızı zaten bellidir. Peki ya cisim zaman geçtikçe hız değiştiriyorsa?
Yaşadığımız çevrede bazen trafik bazen yol bozukluğu bazen de başka bir sebeple de olsa araçlar hız değiş-
tirir. Biz bu değişen hızların herhangi bir andaki değerini nasıl bulabiliriz? Daha önceden anlatıldığı üzere
hız v idi. Yani belli bir süre içinde alınan yol anlamındaydı. Bizim şu an istediğimiz ise belli bir süre değil de
o andaki zamanla birim yerdeğiştirmedir. Aslında bunu söyleyerek kendimize ipucu vermiş oluyoruz. Şöyle
ki biz belli bir süre değil o anı istiyoruz. Yani geçen sürenin 0,0000…01 gibi bir şey olmasını. ∆t’miz 1,2,3 ya
da daha büyük bir değer değil sonsuz küçüklükte 0’a yakın bir değer almasını istiyoruz. Aslına bakarsanız bu
isteğimiz bizi türev-diferansiyel bilgisine yönlendirmektedir.(Türev ve diferansiyel ikisi tamamen aynı şeyler
olmasa da pratikte aynıymış gibi davranacağız. Diferansiyel kendi içindeki değişim türev ise kendi içindeki
değişimin başka bir şeydeki değişime oranı diyebiliriz)
Fizik için Matematik: Diferansiyel,Türev

Diferansiyel kelimesi İngilizcedeki differential kelimesinden o da difference kökünden türemiştir. Difference


kelimesinin Türkçe karşılığı ise fark(değişim)’dir. Bunu biraz daha somut örneklerle gösterecek olursak;
Yuvarlak bir çubuk etrafına sarılmış bant düşünelim. Çubuk etrafına sarılan bu bantın üstten görünümü
aşağıdaki şekildeki gibi olacaktır. Ve bu şeklin yarıçapı r olsun. Bu durumda üstten gördüğüm şekil bir daire-
dir ve daire ifade etmek için formülünü kullanırız.

Şimdi üstten görünümü daire olan bantın


son kabuğunu çıkaralım. İlk durumdaki
bantla son durumdaki bant arasında bir sa-
rımlık ,kabuk şeklinde bant vardır. Ve de bu
kabuk şeklindeki bantın üstten görünümü
bir çemberdir
Yukarıda yaptığımız tanıma göre bu çok küçük farkın ya da iki durum arasındaki değişimin bulunması
türevle sağlanmalı. Dairenin değişimi çember olmalı. Yani daire denkleminin türevi bize çember denklemini
vermeli. Bakalım verecek mi;
NOT(Olayın matematiksel kısmı): Eğer gerçek hayattaki gibi
( pr2)’=2pr => verdi.
düşünürsek. Çıkarılan kabuğun bir kalınlığı vardır. Bu kalınlık
(rson-rilk)’tir.
Biz buna kısaca ∆r deriz. ∆r, 0’a yaklaşırsa dr ile ifade etmek
daha doğru olur. Bu şeklin açık halini düşünürsek;

Bizim değişimimiz aslında 2prdr ’ dir. Ama türev yarıçapın değişimine göre dairenin
değişimi ne olur sorusuna verilen cevaptır. Yani dr ’de 2prdr ’ dir.
Şimdi bunu fiziğe taşırsak bizim istediğimiz şey zamandaki çok ufak bir değişimdi. ∆t
0’a yakın olduğunda ∆x/∆t’nin sonucunu istiyoruz. Bu çok küçük değişim bizi türeve
yönlendiriyor.

ÖRNEK 2.3
x(t)=(t2 – 2t) m fonksiyonuna göre hareket eden bir parçacığın
a)2-3 saniye aralığındaki ortalama hızını bulunuz
b)2. ve 3. Saniyelerdeki hızını bulunuz.

ÇÖZÜM
a) 2-3 saniye aralığındaki ortlama hızı için V=∆x/∆t ’ yi kullanabiliriz.
V=(x3 - x3)/(t3 - t3) => x3=32-2.3=3, x2=22-2.2=0=>V2-3=(3-0)/(3-2) =3 olur.

b) 2. ve 3. Saniyedeki hız anlık bir hızdır ve o anda geçen süre 0’a yakındır. V2=∆x/∆t => ∆t’ nin 0’a yakın
olması bize sonsuz küçüklükteki değişimden bahsetmesi demektir. Ve bu küçüklük bizi türeve götürür. Deği-
şen şey t olduğu için türev t’ye göre alınır. x=t2 - 2t => x’=2t-2 olur. Buradaki x’ ifadesi dx/dt’dir. Bu da ∆x/∆t’
den gelmektedir. Ancak ∆x/∆t şeklini değerlerimiz büyük olduğunda kullanırız. ∆t’nin 0’a yakın olması bizi
türeve götürdüğü için dx/dt şeklinde yazarız. Sonuç olarak madem ki ∆x/∆t=dx/dt o zaman dx/dt=v deriz.
Bu durumda V2= ∆x/∆t =2t-2=> V2=2.2-2=2m/s, ve V3=3.2-2=4 m/s şeklinde anlık hızlar bulunabilir.
Türev mantığını biraz daha açarsak V= ∆x/∆t idi ve biz ∆t’yi 0’a yakın diye türev alıyoruz dedik. Bunu şim-
di de türev formüllerini kullanmadan yapalım. x(t)=(t2 – 2t)m denklemine göre hareket eden parçacık için 2.
Saniyedeki hızını arıyoruz. tilk=2, tson=2+h diyelim ve h burada 0’a yakın sonsuz küçüklükte bir değer. Pratikte
0 alınabilir. V= ∆x/∆t =(xs-xi)/(tson-tilk )=>Xs=(h+2)2-2(h+2)=h2+2h , xi=0, ts-ti=h+2-2=h,
V= (h2+2h)/h=h+2
h, 0’a yakın olduğu için h+2=2
şeklinde 2. Saniyede hız böyle de bulunabilir. Ama biz sonsuz küçüklükteki ifadelerin değişiminin türeve
gittiğini öngörerek kolaylıkla yapabiliriz.
İvme
Günlük hayatta sabit hızlı hareketlerle az karşılaşırız. Küçük de olsa
araçların hızında değişmeler olacaktır. Hiçbir şey olmasa bile hızımızı
düşüren sürtünme vardır. Hızımızı değiştiren bu sürtünme bazen de
sabit hıza sebep olabilir. Başka sebepleri de olsa da mesela karlar sür-
tünmeden dolayı limit hıza ulaşırlar ve sabit hızla inerler. Ama cisimler
bu limit hıza ulaşmadan önce hızlanmaları gerekmektedir. Cisimler
belli ivmeler hızlanır ya da yavaşlar ya da artan bişeydir. Ivme belli bir
süredeki hızdaki değişimdir. a=∆v/∆t formülünde ∆t eğer küçük bir
değer değilse(0’a yakın değilse) buradan çıkaracağımız a değeri o süre
içindeki ortalama ivme’ir.

ÖRNEK 2.4
x=3t2 denklemine göre hareket eden bir cismin;
a)4-5 saniye aralığındaki ivmesini bulunuz.
b)4.sn’deki ivmesini hesaplayınız.

ÇÖZÜM

a) a=DV/Dt= (V5-V4)/(t5-t4)=>V5 ve V4 hızları lazım.


V5 cismin 5. Saniyedeki hızı olup anlık hızdır. v= formülünde o anda ∆t 0’a yakındır. Bu yüzden de V5’i
türevle bulacağız. dx/dt=v=6t=>V5=6.5=30 , V4=6.4=24 > V=(30-24)/(5-4)=6m/s
b) a= DV/Dt (∆t 0’a yakın olduğu için türeve gidiyoruz)= dV/dt=v’=(6t)’=6m/s2
görüldüğü üzere ivmemiz bir denkleme bağlı çıkmadı. Yani ivme sabit. Sabit olduğu için de her an aynı
değerdedir.

Özetle toparlarsak; x konumumuzdur. Belirli bir sürede yapılan konum değişimi bize hızı verir.

V= ∆x/∆t yani “hız konumun zamana göre değişimi”


Değişim çok küçük olduğunda türeve dönüştüğü için önceki cümleyi aynen kullanarak değişim yerine türevi
koyabiliriz.
V= dx/dt yani “hız konumun zamana göre türevi”.

Konumla hız arasındaki ilişki hızla ivme arasında vardır.


a= ∆v/∆t yani “ivme hızın zamana göre değişimi”dir. Buradaki ∆t ifadesi 0’a yakın olacak şekilde küçüldüğü
zaman ivme anlık ivme olur ve ∆t 0’a yakın olduğu için bu küçük değişim türevle bulunur.
“İvme hızın zamana göre türevidir.” ifadesi rahatlıkla kullanılabilir.
Serbest Düşme Hareketi
Havada serbest bırakılan cisimlerin aşağı doğru düşmesi çevremizde sıklıkla gördüğü-
müz bir olaydır. Bu düşme hareketleri, cisimleri yerin merkezine doğru çeken bir kuv-
vetin varlığını gösterir. Bir cisme etki eden yerçekimi kuvveti o cismin ağırlığına eşittir.
Cismin G ağırlığı, G = mg bağıntısı ile bulunur. Burada g, yerçekimi ivmesidir. Yerin
çekim alanı da denilebilir. Yerçekim ivmesinin birimi, hareket ve dinamik konusunda
öğrendiğimiz ivme birimidir. SI birim sisteminde m/s2 ya da N/kg dır.
Havasız ortamda serbest bırakılan bir cisim yerçekimi etkisi ile aşağı doğru g ivmesi ile düşer.
Bu olaya serbest dusme denir. Serbest düşmeye bırakılan bir cisim sabit g yerçekim ivmesinin etkisi ile
aşağı doğru düzgün hızlanan hareket yapar. Her saniye hızı yerçekim ivmesi kadar artar. Yerçekim ivmesi,
g = 9,81 m/s2 dir.
Serbest düşen bir cisim her saniye bir öncekine göre daha fazla yol alır. 1 saniye sonra
aldığı yol h kadar ise, 2 saniye sonra 3h, 3 saniye sonra 5h ... dir.
Ayrıca her saniye yerçekim ivmesi kadar hızı artar.
1 saniye sonra hızı 9.81m/s, 2 saniye sonra 19,62 m/s,
3 saniye sonra hızı 29,43 m/s dir.
Bu durumu açıklarsak. Sabit ivme altındaki hareketli bir cismin hızı zamanla değişecek
ve hızın değişimi anlık yerdeğiştirmeyi de etkileyecektir. Yani birim zamandaki yerde-
ğiştirme de giderek artacaktır. Bu durumu grafiksel olarak ifade edersek konum-zaman
grafiği parabolik, hız-zaman grafiği eğimi olan bir doğru ve ivme-zaman grafiği sabit
bir doğru olacaktır.

ÖRNEK 2.5

Yerçekimi ivmesinin 10m/s2 olduğu sürtünmesiz bir ortamda m kütleli bir


cisim 20 m/s hızla şekildeki gibi yatay atıldığı an durmakta olan araba ok
yönünde düzgün hızlanmaya başlıyor.Cismin arabanın üzerine düşebil-
mesi için arabanın hızlanma ivmesi en az kaç m/sn olmalıdır?

ÇÖZÜM
m kütleli cismin 20 yükseklikten düşmesi için gerekli süre; h=0.5gt2 den t=2sn çıkar. Bu süre zarfında cisim
aynı zamanda Dx=v.t=(20m/s).2=40m yol alır. Geriye kalan 8m lik yolu araba 2 saniye boyunca sabit ivme ile
hızlanarak almalıdır ki cisme yetişsin.
Dx=8m=0.5at2 formölünden t yerine de 2 sn yazarsak; a =4 m/s2 bulunur.
Konumun-Hız-İvme Bağıntıları

Tek boyutlu doğrusal hareketlerin güzel tarafı ivmenin sabit olmasıdır. İvme sabit olduğundan ortalama
ivme ani ivmeye eşittir. Bu tür harekette hız hareketin başından sonuna kadar aynı oranda artar (veya azalır).
Bir boyutta sabit ivmeli hareket yapan bir cismin hareketinine ilişkin a, v, t, x niceliklerini veren ifadeleri
türetirsek:
, böylece ivme ; , ve hız-ivme ilişkisi: olur.

Burada ivme sabit olduğundan ortalama hızı ilk ve son hızların ortalaması şeklinde yazıp

yerdeğiştirme şeklinde yazarsak ivme ile yerdeğiştirme arasındaki ilişkiyi veren denklemimiz:

Ayrıca gerekli işlemler yapılınca yerdeğiştirme-hız arasındaki ilişkiyi veren denklemler elde eidilir:
Bölüm Sonu Soruları
1- A şehrinden B şehrine giden bir araba toplam yolun yarısını 30 km/saat diğer yarısını da 45 km/saat hızla
gidiyor. Bu yolculuk için arabanın ortalama sürati kaç km/saattir?
2- Bir cisim x-ekseni boyunca sabit a= +3m/s2 ivmesi ile hareket etmektedir. t = 0 anında xo = − 6m ve Vo =
+4m/s olduğuna göre t= 10s anındaki konumunu ve hızını bulunuz.
3- Doğrusal hareket yapan bir araba, hareketsiz iken 10s içinde düzgün bir şekilde konumu sıfıra gelecek
şekilde a=+6 m/s2 lik bir ivme hızlanarak harekete başlıyor. Hareketinden 10s süre sonunda sabit şiddette bir
hızla harekete devam ediyor. Başlangıçtan itibaren 15s sonra bu araba kaç m yol almıştır?
4- ilk başta s = 0 konumundan ilk hızsız olarak harekete başlayan
ve doğrusal hareket yapan bir motosikletin ivmesi konuma bağlı
olarak şekildeki gibi değişmektedir. s = 200 m iken motosikletin
hızını bulunuz.

5- Bir cisim düzgün hızlanarak A noktasını VA =10m/s hızı ile, 6s sonra B noktasını da VB = 70m/s hızla geçi-
yor. A ile B noktaları arasındaki mesafeyi ve cismin ortalama ivmesini bulunuz.
6- Bir otomobilin hız göstergesi km/h yerine m/sn olarak ayarlanmıştır. Otomobilin harekete başlamasın-
dan sonraki hızı için aşağıdaki okumalar yapılmıştır.
Zaman (sn) : 0, 2, 4, 6, 8, 10, 12, 14, 16
Hız (m/sn): 0, 0, 2, 5, 10, 15, 20, 22, 22
a) Otomobilin 2sn ara ile otalama ivmesini bulunuz. ivme sabit midir, neden?
b) Yukardaki verileri kullanarak bir hız zaman grafiği çiziniz. (Bunun için yatay eksende 2sn = 1cm ve
düşey eksende 5m/sn = 1cm alınız.)
c) Elde edilen noktalardan geçen düzgün bir eğri çizdikten sonra 1cm2 lik alan ne kadar yolu gösterir?
d) İlk 8sn de gidilen yol ne kadardır?

7- Otomobil şoförlerinin ortalama reaksiyon süreleri 0,7sn kadardır. 16m/sn2 ivme ile yavaşlayabilen bir
aracın seyir hızı 30km/h dir. Şöfür ‘DUR’ işaretini gördükten sonra frene basınca aracın duruncaya kadar
alacağı yol kaç km dir?

8- Bir top, binanın tepesinden düşey olarak V0=30m/sn ilk hızla aşağı doğru atılıyor.
a) Topun 2sn sonraki hızı ne olur?
b) 2sn sonra ne kadar yol alır?
c) Top binanın tepesinden 30m düştüğü zaman hızı ne olur?
d) Top yerden 120m yukarıda elden çıktığına göre ne kadar süre içinde yere çarpar?

9- Bir öğrenci yer çekimini bizzat incelemek için 300m yüksekliğinde bir gökdelene çıkarak elinde bir krono-
metre ile kendini serbest düşmeye bırakıyor.(ilk hızı sıfır) 5sn sonra yardımsever bir insan aynı yere gelerek
öğrenciyi kurtarmak için süngeri sermeye başlıyor. Yardımsever arkadaşın öğrenciyi kurtarabilmesi en geç
kaç saniye içinde süngeri sermesi lazım?

10- A ve B otomobilleri bir yolda komşu iki şeritten aynı yönde giderken bir trafik ışığında duruyorlar. Yeşil
ışık yanınca A otomobili 1m/sn2 lik sabit ivme hızlanıyor. 2sn sonra B otomobili hareket edip 1,3m/sn2 lik
ivme ile hızlanıyor.
a) B’nin A’ya ne zaman ve nerede yetişeceğini bulunuz.
b) Bu andaki otomobillerin hızlarını bulunuz.
3. BÖLÜM

İki Boyutta Hareket

3.1 Atış Hareketleri


3.2 Serbest Düşme
3.3 Bağıl Hız
Bölüm Sonu Soruları
Atış Hareketleri

Atış hareketlerinin şimdiye kadar gördüğümüz hareketten


farklı yönü iki boyutta gerçekleşebilmesidir. Her şey aynı
mantıkla ilerler. Dikkat edilmesi gereken noktalardan birisi
ivmemizin yer çekimi ivmesi olmasıdır. Diğer bir nokta ise
iki boyutta hareketin her boyutu ayrı incelenmelidir. Yatay-
daki mesafe yataydaki hızla, dikeydeki mesafe dikeydeki hızla
bulunmalı ve sürelerin ortak olduğu unutulmamalıdır. Ayrıca
boyutları belli etmek için x bileşenlerini “i” birim vektörü, y
bileşenlerini “j” birim vektörü ile göstereceğiz.

ÖRNEK 3.1
Bir araç x yönünde sabit 20m/s hızla giderken yokuş aşağı inmeye başlıyor ve x yönündeki ivmesi 2m/s2, y
yönündeki ivmesi -3m/s2 oluyor.
a) Bu aracın ivme-hız-konum vektörlerini bulalım
b) Yokuşun yüksekliği 24m ise yokuş bittiği andaki hızın doğrultusu ve büyüklüğünü bulalım

ÇÖZÜM
a) Öncelikle araç 20m/s’lik hızla giderken hızı sabittir ve zamandan bağımsızdır. O sırada hız vektörü
v=20i(m/s)’dir ve j bileşeni yoktur. Daha sonra aşağı doğru ve sağa doğru ivme kazanmıştır. İvme denklemini
yazarsak a=axi+ayj şeklinde olacaktır. Verilen örnek için
a=2i+(-3)j deriz (Buradaki – sadece yön belirtir). Biliyoruz ki ivmenin zamana göre integrali hızı verecektir.
c sabitlerini bulmak için t=0 anına bakalım çünkü t=0’ken v=20i
t=0=>=(2t+c1)i+(-3t+c2)j=c1i+c2j=20i> c1=20, c2=0
v=(20+2t)i-3tj
Not: Bunu Vson=Vilk+a.t den de bulabilirdik ama mümkün olduğunca az formül kullanmaya çalışalım.
ilk konum 0 ise c’ler 0 olur.
İntegral aldığımızda elimizdeki sabitler ilk hız, ilk konum gibi değerleri gösterir.

b) Yokuş aşağı doğru 24m ise konum denkleminin düşey yani j bileşeni -24 olduğunda araç yokuşun sonuna
varmış olacak. Bu durumda
-24=-3t2/2 olduğunda yokuş bitmiş olacak. Buradan t=4 çıkar.
t=4 ise v=28i-12j olur.

Serbest Düşme
Cisimler yerçekiminden dolayı dünyanın merkezine doğru çekilmektedir. Biz cismi istediğimiz kadar yüksek
bir hızla atalım yine de cisim serbest bırakıldığında yapmak istediği hareketi yine yapacaktır. Bu yüzden belli
bir hızla atmadan önce serbest bıraktığımızda nasıl bir yol izliyor onu göreceğiz.

Serbest bıraktığımız için ilk hızımız 0’dır ve aşağı doğru g ile ifade ettiğimiz yerçekimi
ivmesine sahibiz. a=g ise herhangi bir andaki hızımız v= olacaktır. V=gt.
Herhangi bir anda aldığımız yol ise x= =gt /2+c olur. Buradaki c sabiti ilk
2

konumu belirtmektedir ilk konumu 0 alırsak x=h= 0.5gt2 olur. Aşağı iniş süremiz ti olsun.
h= 0.5gti2 olur. Yönler işin içine girerse g=-9,81 alınmalıdır. Eğer g=9,81 alırsak formüllerde
–yi bizim koymamız gerekir.
Çıkış süresi tçıkış olmak üzere Vilk hızı ile atılan bir cisim tç sürede x1=Vi.tç kadar yo
alırdı. Ancak yukarıda söylediğimiz gibi cisim ne şekilde atılırsa atılsın dünyanın
yerçekiminden dolayı x2=0.5gt2 ifadesine göre bir düşüş yaşayacaktır. Cisim x1
kadar çıkarken x2 kadar düşecek ve h=x1-x2 olacaktır. Bu durumda
h=Vilktc- 0.5gtc2 şeklinde formülize ederiz. Göründüğü üzere formüle – yi biz
koyduk ancak bu formülde g=9,81m/s2 alınmaktadır. Eğer formülü daha aka-
demik kullanmak istersek h=vilk.tc + 0.5gtc2 deriz. Bu durumda g=-9,81m/s2
alınması gerekir. Ama – yi formülde kullanmak olayları daha rahat kavramamızı
sağlayabilir. Cisim en tepeye çıktığında duracaktır. Yani yukarı yönde hızı kalma-
dığı için en yüksek yer yukarı yöndeki hızının bittiği yer olacaktır.
Cisim yukarı çıkarken hızı g ivmesi ile azalacak, aşağı inerken g ivmesi ile artacak h mesafesi de değişmediği
için iniş ve çıkışta geçen süreler aynı süreler aynı olduğu için aynı hızada hız büyüklükleri de aynı olur. Aşağı
iniş süresine ti dersek hm mesafemiz;

hmax= 0.5gtilk2 şeklinde ifade edilir.

hmax’ ye yukarıda hmax=V0tcıkıs - 0.5gt2cıkıs demiştik. tcıkıs=tınıs olduğuna göre tcıkıs=tınıs= t0 diyelim

(hmax= 0.5gt02 , hmax=V0t0 - 0.5gt02) > 0.5gt02=V0t0 - 0.5gt02 => gt02=V0t0 => t0=V0/g elde edilir.

Aslında bakarsak başka bir taraftan bunu daha kolay elde edebiliriz. Cisim V0 hızı bitene kadar yukarı çıka-
cak. V0 hızı g ivmesi ile azalacak. Bu yüzden V0-0.5gt0=0 olduğunda hmax seviyesine çıkılmış olacak. Burdan
t0=V0/g olur. t0=tçıkış=tiniş demiştik. Bu durumda toplam hava kalma süremiz tucus=tinis + tcıkıs= 2t0= 2V0/g
olur. Şimdiye kadarki atışlarımız yukarı aşağı yöndeydi. Şimdi de eğik atışı inceleyelim.

-İki boyutlu hareketlerde ilk işimiz verilenleri bileşenlerine ayırıp soruyu tek boyuta çevirmek olmalı.
V0=Vx + Vy

-Diğer bir dikkat edeceğimiz hususa gelirsek aşağı yöndeki yer çekiminden başka etkiyen kuvvet yoktur.
Bu durumda yatayda hız değişmeyecektir.
Yukarı yöndeki Vy hızımız g’ lik ivme ile azalacak ve 0 olduğunda maximum yüksekliğe cisim çıkmış olacak-
tır.
Vy-gtcıkıs=0 => tcıkıs=Vy/g şeklinde çıkış süresini buluruz.

Maximum yüksekliği istersek tçıkış=tiniş => h= 0.5gti2 şeklinde bulur, istersek de Vy.tçıkış - 0.5gtç2 ‘ den elde ede-
biliriz. İkinci formül karışık dursa da atışın simetrik olmadığı durumlarda hesaplamaları burdan yapabiliriz.

hm= 0.5gti2 ‘ yi daha da basitleştirmek için ti=tç=Vy/g ise;


hm= gVy2/g2=Vy2/2g olur. Süreyi bulmadan mesafeye geçeriz.
Vy2=2ghm => Burda aslında zamansız hız formülü gelmektedir. İlk hızı Vi son hızı Vs olan bir hareketlinin
zamansız hız denklemi Vs2 – Vi2 = 2ax şeklindedir. Bu yüzden çıkılan yol direk 02-Vy2 = -2gh ‘tır.

Yatayda alınan yol X=Vx .tucus şeklinde bulunabilir.

Bunun yanında X =Vx.tucus = Vx.2tç=2Vx.Vy/g şeklinde ifade edilebilir.

Yataydaki alınan mesafenin en fazla olmasını istiyorsak cismi hangi açıyla almalıyız soruları olabilir. Burda
düşünmemiz gereken şey kesinlikle türevdir. Peki türevi buraya nasıl dahil edeceğiz öncelikle bilinmelidir
ki en fazla, en az sorularında türev direk aklımıza gelmelidir. Çünkü en fazla veya en az sorularında sorulan
değişkenin grafiğini düşündüğümüz zaman grafikte en fazla dediğimiz yerde bir tepe, en az dediğimiz yerde
bir çukur noktası alıncaktır . Tepe noktasında grafik bir anlık sabitlenir düz gider artma azalma yaşamaz.
Aynı şekilde çukur noktasında da bunlar olacaktır. Değişimin olmaması, türevin 0 olması demek. Yani biz X’
in türevini alıp 0’ a eşitlediğimizde bulduğumuz değer bize tepe noktasını verecektir.

X=2Vx.Vy/g bulmuştuk. X’ in maximum değeri için q ne olur onu merak ediyoruz. Bu yüzden denklemi q’
ya bağlı yazmalıyız.
tan(q) =Vy/Vx => Vy=Vxtan(q)
X=2Vx2.tan(q)/g olur. Ancak bu denklemde atladığımız bir nokta var o da en başta yaptığımız kolaylık olsun
diye Vx =V0cosq, Vy = V0sinq dememiz. Yani Vx ‘in içinde q var. O yüzden
X=2VxVy/g=2V0cos(q).Vysin(q )/g şeklinde yazmalıyız.
Burda 2sin(q ).cos(q )=sin(2q) trigonometrik dönüşümünü kullanarak
X=V0sin(2q)/g deriz. Artık türevini alıp sıfıra eşitleyebiliriz.

(V0sin(2q)/g)’ = V0.cos(2q)/g = 0 => 2q = qp/2 buradan q’ları sadeleştirince p/2=45 bulunur.

Yani buradan anlıyoruz ki soruya göre değişmeyen bir değere sahibiz.

Cisim tA sürede A noktasında olsun. Normalde cisim A noktasına gitmek istiyordu. V0sin(q) yukarı bileşeni
onu h1 yüksekliğine çıkaracaktı.
h1=V0sin(q)tA dır. Ancak cisim serbest düşme yaparak hs= 0.5gtA2 kadar aşağı inmiş ve h2 yüksekliğinden
geçmiştir. h1-hs=h2 => V0sin(q).tA - gtA2=h2
ÖRNEK 3.2
Bir cisim yerden 1 metre yükseklikteten 25 m/s hızla düşey yukarı atıldığı anda
ikinci bir cisim aynı doğrultuda 101 m yüksklikten serbest bırakılıyor. Buna göre;
a) Bu iki cisim kaç saniye sonra çarpışır?
b) Çarpışma yerden kaç metre yükseklikte olur?
ÇÖZÜM
a) Çarpışma durumuna kadar birincinin alacağı yol x, ikincinin y olsun.
x+y=101=0.5gt2+Vo.t-0.5gt2+h burada 0.5gt2 ler birbiri götürünce Vo.t+ h=101 olur.
25.t+1=101 , t=4 saniye

b) y= Vo.t-0.5gt2 +h = 25.4 - (0,5).(9,81).(42)+1 = 22.52 metre bulunur.


Bağıl Hız

Daha önceki konularda hız konusunu işlemiştik. Ancak işlediğimiz


konularda referans noktasından sadece bir doğrultu üzerinde ha-
rek eden cisimleri ele aldık. Bu başlık altında da referans noktasının
konumunun da değişeceğini inceleyeceğiz. Nasıl ki aynı turnuva
grubunda yarışan iki futbol takımınından 28 puana sahip A takımı
ve 35 puana sahip B takımının göreceli puanından bahsediyorsak
cisimlerin hızlarını da bağıl hız olarak ifade ederiz. Burada B’nin A’ya
göre skoru 7 dir.
Düşünün ki belirli bir konumda hareketsiz duruyorsunuz ve önünüzden 50 m/s hızla bir araç geçiyor. O
anda hareketsiz olduğunuz için giden araçla aranızdaki mesafe tamamen aracın hızından kaynaklanıyor. An-
cak sizin de araca ters istikamette 20 m/s hızla gittiğinizi düşünün. Aracın tam yanınızdan geçtiğini düşünün.
Bundan sonraki arada kalan mesafe birim zamanda daha çok artacaktır. Bir önceki durumda saniye başına
50m iken şimdi 50+20= 70m olacaktır. İşte bağıl hızı bu noktada şöyle tarif ediyoruz;
Burada anaızda açılan mesafe saniye başına 70m ve siz giden araca baktığınızda kendinizi duruyor kabul
ediyorsunuz ve aracın size göre hızı bağıl hız oluyor. Bagıl hız, gözlemcinin kendisini hareketin durgun refe-
rans noktası kabul edip gözlenen cisme baktığında ölçtüğü hızdır.

* Bağıl hareket sorularında genellikle gözlenen ve gözlemcinin hangisinden hangisine bakmak gerektiği
karıştırılır.
Unutmayalım ki bağıl hızı bulmak için gözlenenden gözlemciyi çıkarmalıyız. Bağıl hız vektörel olduğu için,
işlemler vektörlerin özel­liklerine göre yapılır.Bağıntıya göre, bağıl hız bulunurken, gözlediğimiz cis­min hızı
aynen alınıp, gözlemcinin hızı ters çevrilerek vektörel olarak toplanır. Bu toplam vektör bağıl hızı ve­rir. Ya
da bağıl hız bulunurken, gözlenen cisim hız vek­törü ile gözlemcinin hız vektörü çakıştırılır. Gözlemci­nin hız
vektörünün ucundan, gözlenenin hız vektörü­nün ucuna çizilen vektör bağıl hızı verir.
ÖRNEK 3.3

Şekildeki gibi aralarında 600 lik açı bulunan farklı doğrultudaki yolda
olan K aracının şöfürü L aracını hangi hızla kendisine yaklaşıyormuş gibi
görür?

ÇÖZÜM
Bağıl hız formölünü kullanmadan önce şunu bilmemeiz
gerekir ki: K aracı gözlemci L aracı da gözenendir.

VB = VKL= VL - VK olur.
Üçgende kosinüs teoremi kullanarak:

VB2 =(VL)2 + ( - VK)2-2VL .VK.cos(120)


VB = 36,056 m/s bulunur.

Nehir Problemleri

Daha önce nehir problemleri ile karşılaşmışsınızdır. Bu tarz problemlerde önemli olan nokta nehir suyunun
akıntı hızıdır. Her soruda akıntı hızının etkisi iyi düşünülmelidir.

ÖRNEK 3.2
Bir nehirde kayıkçı insanları karşıya geçirmektedir.
Kayıkçının suya göre hızı 6 m/dk , akıntı hızı ise 8
m/dk dir. Nehrin genişliği 60m olduuna göre;
a) Kayıkçı karşı kıyıya varıncaya kadar kaç metre
yol alır?
b) Kayıkçının yerdeki bir gözlemciye göre hızı ne
kadardır?

ÇÖZÜM

Öncelikle suya göre hız kavramını tartışalım suya göre hız kayıkçının akıntı
hızından etkilenmiş hali midir yoksa etkilenmemiş hali mi? Eğer suya göre
hızı akıntı hızından etkilenmiş hali olarak düşünürsek şunu anlamış oluruz.
Biz soruda neye göre derse gözlemci hızından o hızı çıkarıyoruz. Yai suyun
akıntı hızı çıkarılmıştır. Kayığın suya göre hızı demek aslında kendi öz hızı
demektir. Bu hız 6 m/dk ise 60m lik nehir genişliğini (60m) / (6m/dk) = 10dk
da alacaktır. Bü süreçte kyığa akıntı da etki edecektir.
a) Akıntı kayığı bu süreçte (8m/dk).(10dk)= 80m sürükleyecek ve kayık karşı kıyıya varana kadar
100m=(602 +802)1/2 yol alacaktır.
b) Yerden bakan bir gözlemci kayığı hem karşı kıyıya hem de akıntı yönüne gidiyormuş gibi görecektir. Bu
hız bağıl hızdır. Toplam alınan yol 100m yi geçen süreye bölersek 100m/10dk= 10m/dk olarak buluruz.
Bölüm Sonu Soruları

1- Yatayla 370 lik açı yapacak şekilde yerden atılan bir taşın ilk hızı V0 =10 m/s’ dir. Bu taşın hareketini iki
boyutta çizerek taşın havada kalma süresini: tuçuş , çıkabileceği maksimum yüksekliği : hmax, menzilini: xyatay
bulunuz.( tuçuş , hmax , xyatay = ?)

2- Rüzgarın doğuya doğru 40km/h hızla estiği bir günde yerden 1 km yükseklikte 150 km/h sabit hızlı bir
helikopter güneye doğru uçuyor. Helikopterden serbest bırakılan bir cismin yere çarpmadan hemen önceki
hızı kaç m/s olur? (Rüzgarın sadece cismin yönünü değiştireceğini dikkate alınız)

3- Bir cisim Vx= +20m/s ve Vy= -15m/s bileşenli ilk hızla t=0 anında başlangıç noktasında harekete geçmek-
tedir. Cisim sadece x doğrultusunda ax=+4 m/s2 ivme ile xy düzleminde hızlanıyor. Cismin t=5s deki hızının
büyüklüğünü ve x ekseni ile yaptığı açıyı bulunuz.

4- Askeri kargo uçağı bir grup askere yardım kumanyası bırakacaktır. Uçağın yerden yüksekliği 100m ve
yatay hızı 40 m/s olduğuna göre paketin istenilen noktaya düşürülmesi için paket hedef noktasından önce
bırakılmalıdır. Uçağın paketi bırakmaya başladığı yer ile asker grubu arasındaki bu yatay mesafe kaç m/s dir?

5- Akıntı hızının doğuya doğru Va = 5 km/h olduğu nehirde bir tekne kuzeydeki karşı kıyıya varmak istiyor.
Teknenin suya göre hızı 10km/h ise yerden bakan bir gözlemciye göre hızını ve nehre göre açısını bulunuz.

6- Bir tüfekle yatay olarak 30 m uzaklıktaki bir hedefe nişan alınıyor. Mermi hedeften 8 cm aşağı saplandığına
göre merminin namludan çıkış hızı nedir?

7- Doğuya doğru 4m/s hızla giden A aracının önündeki kavşağa kuzayden gelen B aracının hızı 3m/s dir.
Buna göre B aracının hızı A aracındaki şöfüre göre hangi yönde kaç m/s dir?

8- Bir merminin namludan çıkış hızı 350 m/s dir. Bu mermi sabit hızla ilerlerken yatay doğrultuda ve 200m
ilerideki bir hedefi vurabilmesi için namlu ile yatay konum arasındaki açı ne olmalıdır?

9- Doğuya 60 km/h’ lik hızla giden bir tren kuzey-doğu doğrultusundaki bir yol üzerinden üstgeçit ile geçi-
yor. Tren yolu ile otoyol arasındaki açı 450 dir. A otomobili 45 km/saat’lik hızla yol boyunca üstgeçite doğru
ilerlediğine göre trenin otomobile göre hızı nedir?

10- Akıntı hızının 4 m/s olan bir ırmakta, bir kayık akıntıya dik doğrultuda ve 6 m/s’lik hızla hareket başlıyor.
Irmağın genişliği 76 m olduğuna göre, kayık kaç saniye sonra ve yatayda nekadar kayarak karşı kıyıya varır?
4. BÖLÜM

Newton Yasalarının Uygulanması

4.1 Kuvvet ve Etkileşimleri


4.2 Newton’ un Birinci Yasası
4.3 Newton’ un İkinci Yasası
4.4 Newton’ un Üçüncü Yasası
4.5 Denge ve Serbest Cisim Diyagramı
4.6 Sürtünme Kuvveti
Bölüm Sonu Soruları
Kuvvet ve Etkileşimleri

Önceki konularda kuvvet etkisi sonucu hareket ettirilmiş cismin kine-


matiğini ele aldık. Şimdiye kadar hareketin sebebiyle değil sonucuyla
ilgilendik. Giden bir arabanın hızı verildiğinde alacağı yolu nasıl buluruz
onu gördük. Ancak bu arabayı harekete geçiren onu hızlandıran ona ivme
kazandıran etki nedir onu görmedik. Biraz düşünürsek bir cisme etki varsa
değişim olması beklenir. Bir cisim sabit hızla yoluna devam ediyor, kimse-
ye karışmadan sakin sakin ilerliyorsa demek ki ona bir etki yoktur deriz.
Ama bir cisim ki hızlanıp yavaşlama yapıyorsa onun üstünde bir etki var
demektir. Hızlanıp yavaşlama yani hızdaki değişim ivme demektir.
Demek ki ivmeyle etkinin bir ilişkisi var. Ayrıca yapılan etkinin ne kadar te-
sir edeceği cismin kütlesiyle de ilgisi olmalı. Mesela bir masayı itmemizle bir
arabayı itmemiz aynı şekilde olmaz. Arabaya daha büyük etki gerekli. Etki diye
bahsettiğimiz kuvvettir. Bir cisme belli bir kuvvet uygulandığında cisim a‘lık bir
ivme ile harekete geçer. a = => Bu durumda a(ivme) dediğimiz şey birim
kütleye uygulanan kuvvet yani bir kütlenin çok sayıda birim parçaya ayrıldığını
düşünürsek her bir parçanın ivmesi a olursa ordaki a ‘ların toplamı bize F ‘i ve-
recektir. Burda çok küçük parçalardan bahsettiğimiz için olayın içine Türev-İn-
tegral girecektir. a ‘ların toplamı dediğimiz için de integral gerekir. a ‘nın m ‘ye
göre integrali F ‘i vermeli.

F= Burada a m ‘e bağlı olmadığı için sabittir.

F=a => F=m.a

Cisimlere etkiyen net kuvvet 0 olduğunda cisimler dengededir deriz. Ancak cisimlerin dengede
olması onların durması demek değildir. Net kuvvet 0 olsa da cisimler hareket edebilirler. Ancak
sabit hızlıdırlar. Burada dikkat edilmesi gereken nokta ivmenin F’e bağlı olduğu Net kuvvet 0’sa
ivmenin de 0 olması gerektiğidir.

ÖRNEK 4.1
Şekildeki gibi M=5kg lik arabaya L=0.4m uzunluğundaki ağırlığı
ihmal edilen iple bağlanmış m=3kg cisim sürtünmesiz yatay düz-
lemde başlangıçda durgun vaziyettedir. Sistem F = 40N luk sabit
kuvvetle çekilince;
a) Sistemin ortak ivmesini bulunuz
b) İpin düşey eksenle yaptığı q açısını bulunuz.

Öcelikle sisteme dışardan etkiyen kuvvetleri incelersek yatayda çeken F=40N


ve düşeydeki Mg ve mg ağırlık kuvvetleri mevcuttur. Mg yüzeyde tepki olarak
bir sürtünme kuvvetine sebep olmadığı için işleme katmıyacağız.
ÇÖZÜM

a) Fnet=Mtoplam.asistem genel formölünde bilineneleri yazınca:


40N=(5kg+3kg). asistem asistem = 5 m/s2 bulunur.

b) T.cosq = m.g= (3kg).(9,81m/s2) = 29,43 N


T.sinq = m.a = (3kg).(5 m/s2) = 15 N

tanq = 29,43N/15 N =1,962


q = arctan(1.962) = 630
Newton’ un Birinci Yasası
“Bir cisme dışardan bir etki olmadıkça cisim tepki vermeyecektir. Durumunu koruyarak ne yapıyorsa ona
devam edecek, durgunsa durgun kalacak, hareket halindeyse sabit hızla hareketine devam edecektir.”
Hareket eden veya sabit duran cisim hızını değiştirmemek için bir direnç gösterir. Bu dirence eylemsizlik
denir. Bu durumda Newton ‘un birinci yasasına eylemsizlik yasası da denilebilir.
Bir otobüste giderken otobüsün hızı neyse bizim hızımız da o olacaktır. Düşü-
nelim ki otobüs sabit hızla gidiyor ve biz de ayakta dikiliyoruz. Otobüs gider-
ken bir kedinin yola atlamasından dolayı aniden frene bastığını düşünelim.
Bu durumda otobüste bulunan bizler ileri doğru düşmek suretiyle gidebiliriz.
İşte bunun sebebi eylemsizliktir. Çünkü sabit hızla otobüsle beraber giderken
otobüs frene bassa da bizim aynı hızla devam etme isteğinden ileri doğru bir
yönelmemiz olacak. Başka bir örnek olarak gondolu düşünebiliriz. Gondolun
en ucuna oturalım ama olayı daha rahat anlamak için. Gondol bizim bulun-
duğumuz tarafın maximum yüksekliğine çıkıp aşağı doğru ivmelendiğinde biz
aşağı gittiğimiz halde bir hafiflik hissederiz. Sebebi gondol aşağı doğru a ivmesi
ile hızlanırken bizim a ivmesi ile eylemsizlik uygulamamızdır. Ayrıca biz bu eylemsizliğe sahip olsak bile
gondola tutunup yerimizde durabiliriz. Ama midemizdeki yiyecekler bizim kadar şanslı olmadığı için biraz
yukarı çıkabilirler. Bu da mide bulantısına sebep olabilir.

Newton’ un İkinci Yasası


Newton’un2. yasasına göre dengelenmemiş kuvvetlerin etkisindeki
parçacık ivmeli hareket yapar. Kinetik, bu dengelenmemiş kuvvetle
onun onun yol açtığı hareket ve bu hareketteki değişim arasında bağ
kurar.

Temel formül:
Eğer bir parçacığın üzerine etkiyen bileşke kuvvet sıfırdan farklı ise,
parçacık bileşke kuvvetin şiddet ile orantılı ve bu kuvvetin yönün-
debir ivmeye sahip olur. Toplam kuvvetin ivmeye oranı parçacığın
kütlesine eşittir.

ÖRNEK 4.2
Yandaki düzenekte görüldüğü gibi iki cisim aralarındaki iple birbirine
bağlanmıştır. Cisimler serbest bırakılınca sistemin ortak ivmesini ve
ipteki gerilmeyi bulunuz.

ÇÖZÜM
Sisteme etkiyen net kuvvet eğik düzlemdeki cismin ağırlığından gelen kuvvet bileşenidir.
Fnet = m.gsinq = mtoplam . asistem

m.gsinq = (5kg).(9,81m/s2) =49,05 N 49,05N =(3+5 kg). asistem asistem = 6,13 m/s2
İpteki gerilme küçük kütleli cismin eylemsizliğinden gelen kuvvet olacaktır
Tip = m.asistem = (3kg). (6,13 m/s2) = 18,4N
Newton’un Üçüncü Yasası

İki arabayı zincirle bağlayıp ikisini farklı yönlere doğru gaz verdiğimizi ve ikisinin de birbirine üstün geleme-
yip dengede kaldığını düşünelim. Bu durumda birinci araba ikinci arabayı 50kN’luk bir kuvvetle çekiyorsa
ikinci araba da birinci arabayı 50kN’luk bir kuvvetle çekiyor demektir. Bu kuvvetler büyüklük olarak aynı
olsa da yön olarak zıttır. İkinci arabanın yerine sabit bir yere birinci arabayı bağlasak ve yine 50kN’luk bir
çekme kuvveti uygulayacak şekilde gaza bastığımızda araba hareket edemiyorsa ilk durumdan bir farkı olma-
ması gerekir.
Yani 50kN’luk bir çekme kuvveti bağlı olduğu yerden
arabaya etki etmektedir. Eğer sabitlenen nokta yeterince
dayanıklı ise biz 100kN’la çekersek o yine 100kN’luk bir
kuvvetle cevap verecektir. Yani biz ne kadar etki edi-
yorsak o da o kadar tepki veriyor. Bu yüzden bu yasaya
etki-tepki yasası diyoruz.

Denge ve Serbest Cisim Diyagramı


İki boyutlu olarak ele alınan sistemlerde cisimler hareketsiz denge halınde duruyor ya da net kuvvetin
sıfırolduğu durumda sabit hızlı hareket ediyorsa sistem üzerine dıştan etkiyen tüm kuvvetleri çizmek isteriz.
Cisim üzerine etkiyen kuvvetleri özellikle yönleriyle ve şiddetleriyle belirterek cismin o anlık durumunu
çizdiğimizde serbest cisim diyagramı çizmiş oluruz.
Serbest Cisim Diyagramı (SCD):
Serbest Cisim Diyagramı çizimi tüm denge ve
kinetik problemlerinin çözümünde temel bir
noktadır.
SCD oluşturmak için özetle;
• Diyagramı çizilecek olan eleman (cisim) veya
elemanlar izole edilir ve ana hatları ile çizilir.
• Tüm dış kuvvetler ve momentler bu çizim
üzerinde gösterilir. Burada; cismi izole ederken
kaldırılan mesnetlere karşılık yerleştirilen tepki
kuvvet ve momentleri de dış kuvvetler olarak
adlandırılır.
• Denge denklemlerini uygularken kullanılacak
noktalar ve boyutlar çizimde gösterilir.
• Bir koordinat eksen takımı tanımlanır.

ÖRNEK 4.1

Şekildeki gibi tavana iplerle asılmış sistem dengededir. T2 ipindeki gerilme


20 Newton olduğuna göre T1 ve T3 çekme kuvvetlerini bulunuz.

ÇÖZÜM
Sistem dengede ise etkiye karşı tepki verilebilmiş
demektir. Sağa çeken kuvvetler sola çekenlere,
aşağı olanlar da yukarı olanlara eşit olmalıdır.
Sürtünme Kuvveti
Şimdiye kadar anlattığımız kısımların çağunda sürtünme kuvvetini ihmal ederek işlemler yaptık. Gerçek
hayatta sürtünme kuvvetinin etkileri ve vazgeçilmez sonuçlarıyla sürekli karşılaşırız. Günlük hayatta sür-
tünmenin büyük bir önemi vardır. Öyleki yürüyüp koşmamızı, koştuktan sonra tekrar durmamızı sağlar.
Sürtünme kuvveti cismin harektli halinde ve durgun halinde ayrı nitelik kazanır. Bunu açıklayacak olursak,
sürtünmeli bir yüzeyde bir masayı itmeye çalıştığımızı düşünelim. İlk başta masa durgun iken önce kuvveti
kademeli arttıracak şekilde itelim. Masayı yavaş yavaş iterken cisim cisim durgun olduğundan tepki kuvveti
statik sürtünme kuvveti olacaktır. Masayı itmeye çalıştığımızda ilk başta zorlanırız ama yerinden oynar oy-
namaz statik sürtünme maximum değerine ulaşı ve bundan sonra daha az kuvetle iterek hareket ettireceğiz.
Bu andan itibaren cisim hareket ediyordur ve uygulanan kuvvet bir önceki duruma göre daha az olur. Cisim
hareket halindeyken üzerine etkiyen sürtünme kuvvetine de kinetik sürtünme kuvveti denir.

Statik sürtünme fsmax=ms.N ile ifade edilir.


N(ormal force) o yüzeye etki eden dik kuvvettir. Burada fsmax yazmamızın sebebi ise statik sürtünme kuvveti
direk bu değeri almaz. Mesela cismin max sürtünmesi 10N olsun. Biz bu cisme 1,2,3,….,9,10 N gibi kuvvetler
uyguladığımızda cisim yaptığımız etkiye tepki olacak şekilde bir kuvvetle karşılık verir. Aksi taktirde biz sta-
tik sürtünmesi 10 olarak hesapladığımız bir cisme 3 N’la ittiğimizde direk 10N karşılık verirse cismin itilen
yönün tersine hareket etmesi gerekir. Böyle bir şey düşünülemez. Sürtünme kuvveti o ittiğimiz cismi hareket
ettirici yönde değil hareketini engelleyici yöndedir. Ancak etki 10N’u geçince cisme yeterli tepki veremez.
Cisim harekete geçer ve artık statik sürtünme de etkili olmaz.
Bu durumda cisme kinetik sürtünme fk=mk.N hakim olur.
Kinetik sürtünme her zaman statik sürtünmeden daha düşük olacaktır. Yani problemlerde ilk yapılacak ham-
le statik sürtünmeyi aşacak etki var mı? Eğer yoksa ne kadar etki varsa o kadar tepki vardır deriz. Eğer maxi-
mum sürtünmenin aşıldığını görürsek kinetik sürtünmeyi kullanacağız.
ÖRNEK 4.2
Eğim açısı a olan eğik düzlemde m kütleli cisim serbest bırakılınca aşağı doğru
hızlanmaktadır. Cisimle yüzey arasındaki statik sürtünme katsayısı ms, kinetik
sürtünme katsayısı mk olmak üzere cismin ivmesini bulunuz.

ÇÖZÜM
Fnet=m.a dır. a ve F’nin yönü aynı olmalıdır. Cisim eğik düzleme paralel bir şekilde hızlanacak demekki etki
etki eden kuvvetlerin bize eğik düzleme paralel yöndeki bileşenleri gerekli. Cisme dışardan bir kuvvet gözük-
memektedir. Bu durumda cisme etki eden tek kuvvet cismin ağırlığı diyebiliriz. Ancak bir noktayı atlamış
oluruz o da sürtünme kuvveti. Çünkü sürtünmeli yüzeyde hareket halinde harekete ters yönde bir sürtünme
kuvveti etki edecektir. Sürtünmenin cismin hareket yönüne ters olduğu unutulmamalıdır.

Bu durumda Fnet=mgsina –mk.mgcosa=ma


a=g(sina-mk.cosa)
Bölüm Sonu Soruları
1- Eğik düzlemde kaymakta olan çekicinin ağırlığı 4W, römorkun yük
ile beraber ağırlığı W dir. Çekici ile zemin arasındaki sürtünme katsayısı
μ =0.8 olduğuna göre çekicinin hızlanırken sahip olabileceği en büyük
ivmenin yerçekimi ivmesine oranı kaçtır? (α=300 , g=9.81 m/s2)

2- Şekildeki gibi yatay düzlemde durmakta olan 5kg kütleli bir cisme sabit
T=40N luk bir çekme kuvveti uygulanarak hareket ettiriliyor. Cisim ile
yüzey arasındaki kinetik sürtünme katsayısı μ =0.6 olduğuna göre cismin
hareketinden 4s sonraki hızını ve yerdeğiştirmesini bulunuz.

3- Kütlesi 0.3 kg olan bir hokey diski sürtünmesiz bir yüzey


üzerinde şekildeki gibi F1=5N ve F2=8N’luk iki kuvvete maruz
kalmaktadır. Diskin ivmesinin büyüklüğünü ve yönünü bulunuz.

4- Kütlesi 75kg olan bir adam asansörde, bir tartının üzerinde duruyor. Asansöre kablolar yardımı ile 3sn.
süre boyunca 8300Nluk yukarı çekme kuvveti uygulanıyor. (g=9.81 m/s2)
a-) Tartıda okunan değer kaç kg’dır?
b-) 2sn sonra asansörün hızı kaç m/s olur? (Asansör ,tartı ve adamın toplam kütlesi 750kg olarak alınız.)

5- 200kg kütlesindeki bir kontrol teleferiği tepesindeki sabit bir kablo


üzerinde A noktasından bağlı bir kablo yardımıyla hareket etmektedir.
A’ daki kablo yatay olarak T = 2.4kN luk bir gerilme uygulandığı anda
teleferiğin ivmesini ve onu destekleyen kablonun teleferiği destekleyen
makaralara uyguladığı kuvveti bulunuz.(g=9.81 m/s2)

6-
Şekildeki blok A noktasından geçerken V1=20m/s ve B
noktasından geçerken V2=10m/s hıza sahiptir. x=75m
, θ=15o için eğik düzlem ile blok arasındaki µk kinetik
sürtünme katsayısını hesaplayınız.

7- Başlangıçta hareketsiz olan 10kg’ lık arabaya P kuvveti uygulanmaktadır.


P1 ve P2 kuvvet durumlarına göre t=5s’ deki hızı ve yerdeğiştirmesi nedir?

8- A ve B elemanları rijit hafif bir çubuk ile


birbirine bağlanmışlar ve yatay düzlemdeki
sürtünmesiz kanallardahareket etmektedirler.
Görülen konum için, A’ nın hızı sağa doğru
VA=0.4m/s ise her bir elemanın ivmesi ile
çubuktaki kuvveti hesaplayınız.
5. BÖLÜM

Newton Yasalarının Uygulanması

5.1 Yörüngeli Hareket


5.2 Dairesel Hareketin Dinamiği
5.3 Düzgün Dairesel Hareket
5.4 Düzgün olmayan Dairesel Hareket
Bölüm Sonu Soruları
Yörüngeli Hareket

Daha önceki hareket konularında cismin doğrusal bir yolda


hareket ettiğini yani hızın yönünün zamanla değişmediğini
ele almıştık. Biliyoruz ki hız vektörel bir büyüklüktür, sürat
gibi sadece şiddeti değil aynı zamanda yönü de önemlidir.
Yörüngeli hareketlerde hızın şiddeti değişse de değişmese
de bizim için önemli olan yönünün değişmesidir. Burada
cismin çhareket boyunca çizdiği yola yörünge bu hareket
şekline de yörüngeli hareket diyeceğiz. Bilinen en genel
yörüngeli hareket dairresel harekettir. Diğer yörüngeli hare-
ketlere örnek harmonik, spiral ve sikloidal hareket verilebi-
lir.

Dünyanın Güneş etrafında dönmesi gibi Ay da Dünya ve Güneş etrafında


yörüngesel hareket halindedir. Ay aynı zamanda kendi etrafında da döner.
Bunu daha da genişletirsek güneş de Samanyolu Galaksisinde yörüngesel
hareket ederek döner. Galaksi de kendi etrafında ve daha büyük bir sistem
ertrafında döner. Belki de o sistem de kendi etrafında dönüyordur hatta
Evrenin kendisi de kendi etrafında dönüyordur. Ne hikmettir ki doğada
bulunan ve insan faktörünün olmadığı tüm bu dönen sistemler ve sistem
içindeki alt sistemler atomlar hatta atom etrafında dönen elektronlar sa-
atin tersi yönünde hareket eder. Bundan dolayı döngüsel hareketlerde saat
yönünün teri pozitif(+) işaret kabul edilecektir.

Şunu da söylemeden geçmek istemiyorum. Belki bazılarınız


biliyordur ama benim gibi önceden Güneş sistemindeki geze-
genlerin aynı düzlemde yuvarlak bir tepi gibi hareket ettikle-
rini zannedenler olmuştur. Ama gezegen yörüngelerinin hem
yarıçapları farklı hemde dolanım düzlemleri farklıdır. Ben de
daha sonra öğrenmiştim bunu. Nerden bilebilirsiniz ki ilkokul-
da anlatılınca çocuk aklınımızla öyle hayal etmiştik.

Yukarda bahsettiğim Evrende herşeyin yörrüngeli hareket ettiğini en güzel anlatan resimlerden biri. Burada
Güneş etrafında dolanan gezegenler Güneş ile birlikte başka bir yörüngede de hareket ederler. Bunu dahada
genişletenilirsiniz hayal gücünüzün elverdiği son sınıra kadar.
Dairesel Hareketin Dinamiği

Hız değişimi hem şiddetin azalıp artması hem de yönünün değişmesi şeklinde olabilir. Hızın zamanla deği-
şiminin ivmeyi verdiğini biliyoruz. Şekilde gördüğünüz dairesel bir pistte sabit süratle hareket eden cismin
zamanla hızı değişir. Bu değişimin sebebi hız vektörünün değişmesidir yani yönünün değişmesidir. İki farklı
sürede anlık hız vektörleri arasındaki değişimi geçen süreye böldüğümüzde a=DV/Dt förmülüne göre radyal
ivmeyi buluruz. Burada radyal ivme ifadesi ivmenin yarıçap(radius) doğrultusunda etkimesidir. Yani sanki
görünmeyen bir kuvvet cismi merkeze doğru çekerek cismin sürekli yönünün değişmesine sebep olmasıdır.
Teğetsel ivmeminin değişmesi çizgisel hız yani süratin değişmesine bağlı olur. Burda teğetsel ivme de varsa
cismin toplam ivmesi radyal ivme ile teğetsel ivmenin vektörel toplamı olacaktır.

Cisim üzerinde net bir kuvvet olunca ivmeli hareket edeceğini biliyoruz.
Yörüngeli harekette cismin hareket doğrultusunu değiştiren bir kuvvet ola-
caktır. Bu kuvvetin yönü genelde hız vektörüne diktir. Bu kuvvete merkez
kaç kuvveti denir. Cisim yürünge üzerinde hareket edince merkezden
kaçmak istediği için bu isim verilmiştir.

Tam olarak çembersel bir yörüngede dolanmakta olan sabit hızlı cismin bir tur atınca alacağı yol çemberin
çevresi x=2pr kadar olur. Bir tam tur atınca geçen süreye hareketin periyodu diyoruz ve bunu T ile gösteriyo-
ruz. Şimdi bilinen x=V.t fomlünde yerine yazarsak 2pr=V.T denklemini elde ederiz. Bu denklemin önemliliği
kadar F=m.a dan esinlenerek elde edeceeğimiz Fmerkezkaç = m. ar yazdığımızda ve ar yerine de Dv/Dt yazınca
Fmerkezkaç= m.V2/r elde ederiz. Burada radyal ivmeyi çekince:

Dairesel hareket konusu günlük hayatta bir çok problemin çözümü için bize kolaylık sağlayacak biligler içe-
rir. Örneğin karayolunda giden bir aracın virajlarda savrulmamamsı için sahip olması minnimun hız, dönen
cisimlerin tork ve periyot analizleri gibi çoğu alanda dairesel hareketin förmüllerini kullanırız.

Cisim dairesel pistte bir tam tur atınca geçen süre dolanım periyo-
du olur. Aslında dairesel harekette tambir tur atmak demek 3600
=2p açı taramak demektir. döngüsel hareket yapan cisimler birim
zamanda taradıkları açıya açısal hız denir ve w ile gösterilir. Tambir
turun sonunda tranan 2p lik açıyı geçen süre yani dolanım periyo-
du T ye bölersek açısal hızı elde ederiz.
Düzgün Dairesel Hareket

Düzgun dairesel harekette çizgisel hızın şiddeti sabittir. Hız vektörünün yönü değişse de büyüklüğü de-
ğişmez. Herhangi bir kütle çembersel yörünge üzerinde eşit zaman dilimlerinde, eşit miktarda yol alır. Bu
yüzden cismin hareketine düzgün dairesel hareket denir.

* Cismin çembersel yörünge üzerinde bir tam tur yapması için geçen süreye periyot denir. T harfi ile göste-
rilen bu büyüklüğün birimi saniyedir.

*Düzgün dairesel hareket yapan cismin bir saniyedeki tur sayısına frekans denir. Frekansı f harfi ile gösteri-
riz. Frekansın birimi s-1 dir ve buna Hertz in kısaltılışı Hz denir.
Frekans aslında açısal hız ile karıştırılabilir. Yani siz eğer bir cismin açısal hızından bahsediyorsanız aslında
frekansı hakkında bilgi vermiş olursunuz. Açısal hız birim saniyede taranan açı olduğuna göre tam turu(2p)
alması için geçen süre normal frekanstır. Açısal hızın birimi rad/s dır.

ÖRNEK 5.1

Bir cisim O noktası etrafında r=15 m yarıçapıyla düzgün daire-


sel hareket yapıyor. Hareketin periyodu T=5 s olduğuna göre;

a) Cismin merkezcil ivmesi kaç m/s2 dir?


b) KL arası ortalama ivmesi kaç m/s2 dir? ( p=3.14 alınız.)

ÇÖZÜM

a) Cismin merkezcil ivmesi a=V2/r ye eşittir. r yi biliyoruz ama V yi bulmak için T yi ve r yi kullanacağız.
x=V.t den 2pr = V.T olur. V=2pr/T =(2 x 3,14 x 15)/5 = 18,84 m/s olur. Böylece;
a= V2/r den , a=18,842/15= 1,256 m /s2 bulunur.

b) K ve L noktaları arası çemberin üçte biri olduğu için geçen süre periyodun da
üçte biri olacaktır. t = T/3 = 5/3. KL arası ortalama ivme:
aKL = DV/Dt = 18,84/ (5/3) =11,088 m/s2 bulunur.
Düzgün olmayan Dairesel Hareket

Dairesel bir yörüngede süratini değişerek hareket eden parçacı­ğın


merkezcil ivmesinin yanında dV/dt büyüklüğünde bir teğetsel ivme-
sinin de olabileceğini söylemiştik. Teğetsel ivme cismin dahada hız-
lanmasına yada yavaşlamasına sebep olacaktır. Böylece parçacığa etki
eden kuvvetin hem merkezcil hem teğetsel bileşeni olmalıdır. Toplam
kuvvet bu iki bileşenin vektörel toplamı olacakır. F = Fr + Ft . Kuvvetin
etkisiyle aynı doğrultularda ivmeler doğarken çizgisel ve açısal hızlar
da bu sebeple değiştiği için biz buna düzgün olmayan dairesel hare-
ket diyoruz. Yani, toplam ivme a = ar + at olduğundan, parçacığa etki
eden toplam kuvvet ve toplam ivme karmaşıktır.

ÖRNEK 5.2
Bir sporcu 2m uzunluğundakı halatın ucunda bulunan
5kg kütleli demir gülleyi at=3m/s2 ivme ile 7sn sallıyarak
bırakacaktır. Buna göre;
a) Güllenin halattan ayrılırkenki çizgisel hızı kaç m/s
dir?
b) Gülle ayrılmadan hemen önce halattaki tepki kuvveti
nekadardır?
c) Gülle yatay düzlem ile yerden 0.6m yükseklikten 300
lik eğik atış yaptığına göre güllenin menzilini bulunuz?

ÇÖZÜM
a) DVT=at.Dt
DVT=(at=3m/s2).(7sn)=21m/s

b) Fmerkezkaç = Thalat olacaktır.

F= mV2/r den T= 5x212/2 = 1102,5N bulunur.

c) Gülle o.6 m yükseklikte V=21 m/s lik hızla atılırkendüşey bileşeni Vy=V.sin30= 10.5 m/s yatay bileşeni de
Vx=V.cos30 = 18.18 m/s olur. Gülle tepe noktasına
t1=Vy/g = 10.5/9.81=1.07 saniyede çıkar. Böylece
y= Vy.t1 - 0.5g.t12 = (10,5x1,07)-(0,5x9,81x1,072)= 5,62m yükselir.
Bu süre boyunca x1= Vx.t1 = 19,47m yol alır.
Gülle tepe noktasında iken yerden 5,62+0,6=6,22m yüksektedir. Aşağı iniş süresi h=0.5gt22 den
t2 = 2(y+0.6)/g
t2 =2x6,62/9,81= 1,35 s bulnur.
Toplam menzil : X= x1+Vx.t2 =

X=19.47+18,18x1,35= 44,0065 m bulunur.


Bölüm Sonu Soruları
1- Birbirlerine paralel olan demiryolu ile karayolunda hareket eden tren
ve otomobilin hız-zaman grafikleri şekildeki gibidir. Tren ve otomobilin
kat ettikleri yolun eşit olabilmesi için geçen süre kaç T dir?

2- Yarıçap r = 1 m olan şekildeki makaraya sarılı ipin A ucu


3 m/s2 sabit ivme ile çekiliyor. t = 0 anında B noktasının
toplam ivmesi aB = 5 m/s2 olduğuna göre hızı VB kaç m/s
dir ? A
3- Bir maddesel nokta bir doğru üzerinde a=-0.2V2 ivme –hız bağıntısı ile hareket ediyor. t=0 anında
konum s=0 ve hız V= 20m/s olduğuna göre t=2 deki konumu, hızı ve ivmeyi hesaplayınız.

4- Bir OABC dikdörtgen levhası t anında xy düzleminde D ekseni


etrafında ω = 10 rad/s sabit açısal hızı ile dönmektedir. Bu an için
C noktasının hız ve ivme vektörlerini ve D eksenine olan uzaklığını
bulunuz.

5-
Bir bovling topu, bir bottan gölün yüzeyine V= 8 m/s’lik hızla çarpacak
şekilde bırakılıyor. Su içindeyken topun aşağıya doğru hız cinsinden
a=3-0,1V2 ivmesine sahip olduğunu kabul ederek topun gölün tabanına
vurduğundaki hızını bulunuz. (g= 9.81 m/s2)
30m

6- Yandaki şekilde gösterildiği gibi sürtnünme katsayısının 0.4 olduğu yüzeyde


80N’ luk kuvvet yatay düzlem ile 370 açı yapacak şekilde durgun haldeki bir
bloğa etkiyor. Kütlesi 5kg olan bu tahta blok 6 saniye sonra kaç metre yol
alır? (g= 9.81 m/s2)

7- Şekildeki eğik düzlem üzerine 7 kg’ lık cisim bırakılıyor. Eğik düzlem
ile cismin tabanı arasındaki sürtünme katsayısı 0.4 olduğuna göre cismin
kaymadan durabilmesi için q açısı en fazla kaç derece olmalıdır?
(g= 9.81 m/s2)

8- A bileziği ve B bloğu şekilde gösterildiği gibi C, D ve E gibi üç maka-


radan geçen bir kabloyla bağlanmışlardır. C ve E makaraları sabit iken
D makarası 5 m/s’ lik sabit bir hızla aşağıya doğru çekilen bir bileziğe
tutturulmuştur. A bileziği K yerinden itibaren sabit bir ivmeyle ve ilk
m hızsız olarak aşağıya doğru hareket etmeye başlamaktadır. A bileziği
L noktasından geçerken hızı 4 m/s olduğu bilindiğine göre tam bu
sırada B bloğunun yüksekliğindeki değişim, hız ve ivmesini bulunuz.

9- Aralarında 900 açı yapacak şekilde iki ip ile tavana asılan


bir cismin G ağırlığı ve T2 ipinde meydana getirdiği gerilme
kaç N dur?
10- Bir mühendis arabaların savrulmadan gidebileceği bir karayolu virajı
tasarlamak istiyor. Yarıçapı R=50m olan bu virajda araçlar en fazla V= 15
m/s hızla gidebildiğine göre virajın eğim açısı kaç derece olmalıdır?
(g=9,81 m/s2 ve lastik asfalt arası kinetik sürtünme katsayısı mk=0,3 alınız.)

11- Kütlesi 7 kg olan demir bilye 1 m uzunuğundaki ipin ucuna bağlanarak


düşey düzlemde döndürülüyor. Bu ip en fazla 100 N luk kuvvete kadar dayanıklı
olduğuna göre topun dönebileceği en büyük hız kaç m/s olur?

12- Yarıçapı R=15m olan bir dönme dolapta dönen bir yolcunun
konumu en altta iken çizgisel hızı 25 m/s dir. Yolcunun bu konumdaki
ivmesinin büyüklüğünü ve yönünün bulunuz.

13-
Şekildeki gibi yarıçapı R=0.7m olan içi boş silindirin çeperine iç taraftan 2 kg lik
bircisim aşağı kaymadan durabilmesi için silindirin açısal hızı kaç rad/s olmalıdır?
(g=9,81 m/s2 ve cisim ile silindir arasındaki sürtünme katsayısı mk=0,5 alınız.)
6. BÖLÜM

İş-Güç-Enerji

6.1 İş-Enerji
6.2 Fizik için Matematik: İntegral
6.3 Kinetik Enerji ve İş-Enerji İlişkisi
6.4 Değişken Kuvvetlerle İş-Enerji
6.5 Güç, Verim
Bölüm Sonu Sorularıs
İş-Enerji
Etrafımıza baktığımızda görürüz ki her şey iş-enerji dediğimiz iki kavram etrafında
dönmektedir. Bu iki kavram birbiriyle doğrudan ilişkilidir. İşçiler çalıştığı işlerde
enerji harcarlar ve patronlar aslında insanlara harcadığı enerjinin parasını öderler.
Çalışanlar yerine makineler kullanıldığında onlara iş yaptırmak için enerji kaynağı
sağlamak gerekir. Bu enerji için insanlar birbiriyle savaşa girmekte, birbirlerini öldür-
mekte. Çünkü ne kadar çok enerji o kadar çok güç demektir. Enerji fizikte iş yapa-
bilme yetisi olarak bilinse de günlük hayatta farklı tanımlayabiliyoruz. Ama fizikteki
karşılığı bir işi oluşu gerçekleştirmek için gerekli yetidir.İş “W”(work) sembolü ile
gösterilir. Yapılan iş aslında tüketilen enerjidir. Ne kadar iş yapıldıysa o kadar enerji
harcanır. Bu yüzden birimi de J’dür(W=E).

Duran bir cismi bir miktar itsek o cisim üzerine bir iş yapmış oluruz. Bir hamal yükleri bir yerden başka bir
yere taşıyarak yine bir iş yapmış olur ve bu işin karşılığı olarak bir ücret alır. Eğer hamal yükleri alıp biraz do-
laştırıp aynı yerine geri getirse bu hamala kimse ücret vermez. Çünkü hamalın yaptığının fizikteki karşılığı iş
değildir. Fizikte iş yapabilmek için cismin yerdeğiştirmesi gerekir. Eğer net yerdeğişimi yoksa o cisim üzerine
yapılan net bir iş de yok deriz. Bu durumda iş yerdeğiştirme ile doğru orantılıdır. Ayrıca bir cisim üzerinde
iş yapmak için bildiğiniz gibi o cismi harekete geçirecek bir kuvvet de gereklidir. Bu yüzden iş kuvvetle de
orantılıdır. W=F.∆x şeklinde ifade edilir.
Burada dikkat edilmesi gereken F ve ∆x’in aynı doğrultuda olması
gerektiğidir. Çünkü burdaki W, F kuvvetinin yaptığı iştir. F kuvveti
de anca kendi doğrultusunda iş yapabilir. Eğer hareket 3 boyuttaysa
yapılan işi bulmak için F ve ∆x’i skaler çarpmalıyız. Çünkü ska-
ler çarpımda aynı yönlü çarpım vardır(vektörler konusunda daha
ayrınıtılı incelenebilir). Peki ya F yoldan bağımsız değilse, yani F x’e
bağlı bir denkleme göre değişiyorsa işi nasıl bulacağız. Bunu yapma-
dan önce biraz integrali inceleyelim.

Burada önemli bir ayrıntıya daha değinmek istiyorum. Hatırlarsanız kuvvet ile yolu çarpınca aynı zamanda
döndürme etkisi olan momenti de buluyorduk. Burda da kuvvet ile mesafeyi çarpınca yapılan işi ya harcanan
enerjiyi elde ediyoruz. Peki iş ve momenti nasıl birbirinden ayırt edeceğiz? Cevabı şöyle:

* Üç boyutlu olarak düşündüğünüzde, kuvveti cismin hareket doğrultusunda


etki ettiğinizde ve cisim ile kuvvet aynı doğrultuda ise iş yapılmış olur. Eğer
üzerinde kuvvet uygulanan cisim yerdeğiştirmeyip dönme hareketi yapıyorsa
elde edilen türetilmiş büyüklük moment olur. Yani en etkili iş kuvvet ile hare-
ket yönü arasındaki açı sıfır olduğu durum (skaler çarpım), en etkili moment
de kuvvet ile kuvvet kolu arasındaki açının dik olduğu ( vektörel çarpım )
durumdur.
ÖRNEK 6.1

Bir nakliye işçisi 50 kg’lık bir sandığı düz bir zeminde, bir sabit hızla, üzerinde
yatay bir kuvvet kullanarak sandığı 7m itiyor. Sandık ile zemin arasındaki kinetik
sürtünme katsayısı mk=0.3 ‘e eşittir. Bu kuvvet sandık üzerinde ne kadar iş yapar?
(g=9.81m/s2)
ÇÖZÜM
Sandık üzerine etkiyen toplam kuvvetler nakliyecinin uyguladığı ve sandığın ağırlığının oluşturdurduğu
zemindeki sürtünme kuvvetleridir. Cisim sabit hızla ilerlediğine göre ivme yoktur ve net kuvvet sıfırdır. De-
mekki nakliyecinin uyguladığı kuvvet sürütnme kuvvetne eşittir. Fn = -fs = mk .mg = (0,3).(50kg).(9,81m/s2)
Fn= 147,15N
Yapılan iş kuvvet ile yerdeğiştirmenın skaler çarpımı olduğu için W = Fn .Dx =(147,15N).(7m)= 1030,05 Joule
Fizik için Matematik: İntegral

Dilimize İngilizce ya da Fransızcadan geçmiş olan integral sözcüğü tümlemek


anlamına gelmektedir. İntegral Latince toplam(Summa) kelimesinin baş harfi olan
S’nin biraz değiştirilmiş hali sembolize edilir. Geldiği kökenden de anlaşıla-
cağı üzere integral, entegre etmek birleştirmek, tüme varmak, toplamak anlamında-
dır. Ama normal toplama işleminden farklı olarak çok küçük olan şeyleri toplama-
da kullanılır. Ayrıca nasıl toplama çıkarmanın tersi ise integral de türevin tersidir.
Şimdi türevde verdiğimiz çember-daire örneğini tersten inceleyelim.
Büyük dairenin içindeki seçtiğimiz dilimin alanını bulmak için dilimi
açtığımızı düşünelim
Bu seçtiğimiz dilim daire üzerinde her yerde olabileceğinden dolayı r değişkendir ve 0’dan R’ye kadar deği-
şebilir. Seçtiğimiz dilimin alanı yaklaşık olarak 2pr∆r olur. Türevde de söylediğimiz gibi ∆r çok küçük oldu-
ğunda dr şeklinde ifade etmemiz daha doğru olacaktır. Şimdi r’nin artışına göre seçilen bu dilimlerin r=0’dan
r=R’ye kadar sonsuz tane olduğunu söyleyebiliriz. Bu şekilde sonsuz küçüklerin toplanması integrali devreye
sokacaktır. Toplam 0’dan R’ye sonsuz tane 2prdr nin matematiksel ifadesi:

= 2pr2/2=[pr2]R0= pR2- p02= pR2


>> demek ki sonsuz tane sıralı çemberi yarıçapı 0’dan R’ye değişmek üzere birleştirirsek yarıçapı R olan bir
daire elde ediyoruz.
Başka bir örnek olarak üçgenin alanını bulalım:

Şimdi bunu integralle nasıl hesaplarız onu düşünelim.


Öncelikle toplamını bulmak istediğimiz şeyin içinde
küçük bir parça almalıyız ve bu aldığımız parçanın belli
bir değerden yine başka bir belli değere sonsuz tanesi
yan yana gelsin ve asıl istediğimiz şekli vermeli. Burada
bu işlemi gerçekleştirecek olan yani üçgeni oluşturacak
olan boyu devamlı değişen 0’dan başlayan ve 6’ya kadar
yan yana dizilen çubuklar olacaktır. Seçilen küçük alanı
hesaplarsak:

Taralı alanı üçgenin alanından hesaplarsak;


Taban*yükseklik/2=6.4/2=12
Bu küçük parçadan 0’la 6 arasında sonsuz tane olduğu için 0’dan 6’ya y(x)dx değerlerini toplamalıyız. Bu
şekilde denkleme bağlı olarak değişen değerlerin toplamını integral yardımıyla hesapladığımız için
= =x2/3> 62/3-02/3=36/3=12
Şimdi konumuza dönersek Kuvvet yola bağlı değiştiğinde ne yapmamız gerekiyor. Bunu bir grafiğe dökersek
daha iyi anlaşılabilir. x1’den x2’ye kadar yapılan işi bulmak istersek şunu görürüz ki direk F.(x2-x1) yapamıyo-
ruz. Çünkü (x2-x1) yolu boyunca her noktada aynı F değerine sahip değiliz. Bu yüzden yolu sonsuz parçaya
ayırıp herhangi bir noktada yapılan işi temsil edecek fonk-
siyonu bulup x1 den x2’ye integral alırsak toplam işe ulaşırız.
Taralı Alan=F(x) .(xs-xi)=F(x).∆x olur. ∆x çok küçük olduğu
için TA=F(x).dx deriz ve bu bize birim aralıktaki işi verir.
Herhangi bir x konumundaki iş F(x)dx ise x1’den x2’ye F(x)dx
değerlerini toplayarak bütün işi bulabiliriz. Bu durumda
förmülüne ulaşırız.
Bu arada F-x grafiğinin alanı da aynı yolla bulunacağın-
dan alan işi vermektedir.
Kinetik Enerji ve İş-Enerji İlişkisi
Kinetik enerji cismin hareketinden dolayı sahip olduğu bir ener-
ji çeşididir. Cisim hareketli ise kinetik enejisi var , hareketsiz ise
kinetik enerjisi yoktur.
-Öteleme hareketi
-Dönme hareketi
-Yuvarlanma hareketi
-Harmonik hareket
-Sikloid hareket
gibi farklı hareketler olunca bunlara bağlı farklı kinnetik enerji
çeşitleri de mevcut olacaktır.
Kinetik enerji skaler bir nicelik olup, iş ile aynı birime sahiptir.
Kinetik enerji cismin durgun haldeki kütlesi ve hızın karesi ile
doğru orantılıdır. Burada önemli olan kinetik enerjinin cismin
karesiyle doğru orantılı olduğudur. Bir parçacığa etki­leyen net
sabit bir ∑F kuvveti tarafından parçacık üzerinde yapılan iş, ki­
netik enerjisindeki değişime eşittir.

“Dengelenmemiş kuvvetler altında hareket eden cisimler ivmeli


hareket yapar.” Bu ifade Newton fiziğinin temel ilkesidir. Kinetik
enerji kavramına uyarlarsak ivmeli hareket eden cisimlere aktarı-
lan enerji değişeceği anlamına gelir. Yani dengelenmemiş kuvvet-
ler altındaki bir cismin hareketinden dolayı sahip olacağı enerji
ki, biz buna kinetik enerji diyoruz, ya azalır ya da artar. Yanda
göreceğiniz formöller bunun ispatı niteliğindedir. Adım adım
izlersek net kuvvet ivmeyi oluşturacak, bu ivmeden dolayı hız de-
ğişecek ve değişen hızdan dolayı da hareket enerjisi olan kinetik
enerji değişecektir.

ÖRNEK 6.2

Şekildeki gibi yatay sürtünmeli düzlemde K ve L cisimlerine iki


farklı kuvvet uygulanarak gösterilen mesafeler kadar hareket
ettiriliyor. Bu iki cismin kazanacağı bilirtilen mesafeleri aldıktan
sonra sahip olacakları kinetik enerjileri oranı WK/WL kaçtır?

ÇÖZÜM
Öncelikle cisimler üzerine etkiyen net kuvvetleri elde edelim.
K cismine sağa doğru 3F, sola dogru da F kadar sürtünme kuvveti etki ediyor. Net kuvvet FK=(3F-F)= 2F
L cismine sağa doğru 7F, sola dogru da 2F kadar sürtünme kuvveti etki ediyor. Net kuvvet FL=(7F-2F)= 5F

Yapılan iş kinetik enerjiye eşit olduğundan WK= 2F.5x= 10F.x WL= 5F.3x=15F.x WK/WL=2/3
Değişken Kuvvetlerle İş-Enerji

Uzerine değişken kuvvetler etkiyen maddesel cisimler farklı ivmelerle farklı hızlarda hareket yaparlar. Bu
tür hareketli cisimlerin enerjilerini hesaplamak tüm süre boyunca etkiyen kuvvet ile birim zamanda kate-
dilen mesafeyi çarpmakle elde ederiz (DW=F. Dx). Kuvvetlerin, maddesel noktanın yer değiştirmesine bağlı
olarak değiştiği problemlerin enerji hesapları en uygun yolu integral alma yoluyla olur. Kuvvetlerin yer değiş-
tirmeye göre integralinin alınması, bu bölümünde konusu olan, iş ve enerji denklemlerini verir.

*İşin bu tanımı göre, kuvvetin yer değiştirmeye dik yöndeki bileşeni Fn = F sinα ’nin iş yapmadığına
dikkat etmek gerekir.

Maddesel nokta, uygulanan kuvvet sonucu izlediği yol boyunca sonlu bir yer değiştirme yapıyorsa, kuvvetin
yaptığı iş kuvvet-yol grafiğinin altında kalan alan integre edilerek bulunan değerdir.

ÖRNEK 6.3
Şekildeki gibi 100N lık kuvvet 8kg lık cisme etki ederek 4m
yatayda sürükleyip h= 0.65m yüksekliğe çıkarıyor. Buna göre:
a) Kuvvetin cisim üzerine yaptığı yataydaki işin dikeydeki işe
oranını Wy/Wd ,
x = 4m b) Kuvvetin radyal bölgede yatay bileşeninin yaptığı işi bulunuz.
ÇÖZÜM
a) Yataydaki bileşen Wy = F.dx =(100N).(4m) = 400J
Düşeydeki bileşenin yaptığı iş: Wd = F.h = (100N).(0.65m) = 65J
Wy/Wd =400/65
b) Kuvvetin yatay bileşeni F.cosa olur. Öyleyse a=90-q nın sınırlarını bulmalı-
yız.
a= r-h=150-65= 85cm q= arccos(85/150)=55.50 , a = 350 ;
aradaki mesafe d/r=arcsinq , d= 1.23m bulunur.

= [(100.sina).(1.23m)]270 325.5 = 53.33Joule bulunur.


Güç

Günlük hayatta güç terimini sıklıkla kullanırız. a şası B şahsından daha güçlü ya da X arabasının gücü
120beygir gibi ifadeleri kullanınca acaba fiziksel olarak ne anlama geldiğini biliyormuyuz? Çalışan bir sis-
tem iş yapınca harcadığı enerji bizim için önemlidir. Peki bir makinenein enerji tüketimini nasıl hesaplarız?
Makineler üzerinde güç birimi W(Watt) veya hp(horsepower) şeklindeki birimlerle etiketlenirler. Bu onların
birimzamanda tükettiği enerji mıktarının ölçüsüdür. Bir makinenin kapasitesi ya da gücü zamana bağlı iş
yapabilme veya enerji aktarabilme hızı ya da kapasitesi ile ölçülür. Ne kadar küçük olursa olsun, bir motor
yeterli zaman verildiğinde büyük miktarda enerji verebileceği için toplam iş ya da enerji çıkışı bu kapasitenin
yani gücün bir ölçüsü değildir. Öte yandan, kısa bir süre içerisinde büyük miktarlarda enerji vermek büyük
ve güçlü bir makine gereklidir. Bu nedenle bir makinenin gücü birim zamanda yaptıgı iş yada iş yapabil-
me hızı olarak tanımlanır. Fizikte güç İngilizce “power” kelimesinin baş harfi P ile sembolize edilir. Birimi
Watt’tır (1W = 1 J/s ). Bazen beygir gücü “hp” şeklinde de ifade edilir (1 hp = 746 W = 0.746 kW).

Verim
Sanayileşmenin getirdiği aşırı doğal kaynak tüketimi enerjinin önemini artırınca kullanılan enerjini boşa
harcanmaması için aletlerin verimliliği çok önemlidir. Bir işi yapmak için sarfettiğimiz enerjiye kıyasla o
iş sonucunda nekadar faydalı sonuç ya da kazanç elde edeceğimiz verimlilikle alakalıdır. Enerji tüketerek
çalışan bir makinenin aynı zaman aralığında yaptığı işin ona yapılan işe oranına, makinenin mekanik
verimi adı verilir “em”. Tüm makineler bir miktar enerji kaybı ile çalıştıklarından ve makinenin içerisinde
ilave bir enerji üretimi oluşturulmadığından dolayı verim her zaman birden küçüktür(0<em<1).
Hareketli parçalar içeren mekanik cihazlarda, sürtünme kuvvetleri nedeniyle her zaman bir miktar enerji
kaybı olacaktır. Bu sürtünme işi, ısı enerjisine dönüşür ve ortama yayılır. Herhangi bir andaki mekanik ve-
rim:

ÖRNEK 6.4
Yandaki şekilde görüldüğü gibi 300 lik açılı eğik düzlemdeki 20kg
lık cisim elektrik motoru yardmıyla yukarı çekilmek isteniyor.
Motorun verimi em=0.85 olduğuna göre cismi V=12 m/s lik sabit
hızla yukarı çekmesi için halattaki gerilme kuvveti T= 45N oluyor.
buna göre motorun verimi kaç Watt olamlıdır?
ÇÖZÜM
P = DW/Dt DW= F.Dx ,
DW yerine F.Dx yazarsak Pçıkış = F.Dx/Dt = F. V olur

Pçıkış = T.V = (45N).(12m/s) =540 Watt Pçıkış/ Pgiriş = 0.85 ise

Pgiriş = 540/0.85 = 635.3 Watt bulunur.


Bölüm Sonu Soruları
1- Bir cismin üzerine etkiyen kuvvet şekilde görüldüğü gibi x ile değişmektedir.
Cisim x=0’ dan x=8 m’ye hareket ettiğinde kuvvetin yaptığı iş kaç Joule’dur?

2- Başlangıçta durgun olan 6 kg’lık bir blok, 12 N’luk sabit, yatay bir kuvvetle yatay
sürtünmesiz bir yüzey boyunca çekilmektedir. Blok 3m’lik bir uzaklığa hareket
ettikten sonra hızını bulunuz.
Şekildeki 2 tonluk yarış arabası pistte giderken rüzgarın sürükleme diren-
3-
cine maruz kalmaktadır. Rüzgarın direnç kuvveti hız cinsinden FD= 1.2V2
bağıntısıyla verildiğine göre otomobilin 50 m/s hızla gitmesi için motorun
üretmesi gereken güç kaç watt’tır?

4- Şekildeki sistemin yatay yüzü sürtünmeli ve sürtünme katsayısı k=0,2 dir.


Buna göre sistemin ivmesini ve ipteki gerilme kuvvetini bulunuz.

5- Bir işçi 30 kg kütleli 20 torbayı her bir katı 3.5m olan bir apartmanın giriş katından 5.katına çıkarıyor. İşçi
bu işi 40 dakikada bitirdiğine göre yapılan toplam işi ve işçinin gücünü bulunuz.

6- Şekildeki sabit eksen etrafında dönebilen tekerlek etrafına sarılı ipin çekilmesi ile
döndürülüyor. İp çekilince tekerlek üzerinde 2 N.m lik net tork oluşuyor. A noktası
toplam durgun halden belli bir süre sonra 1200 lik açı taradığına göre yapılan net
işi ve tekerleğin bu andaki açısal hızını bulunuz.

7- Gücü 30000 W olan bir motor, dakikada 6000 devir yapması için döndürme momenti kaç N.m olmalıdır?

8- Sabiti k=200N/m olan bir yayın üzerine 17 kg kütleli cisim bırakılınca h kadar
sıkışıyor.
a) Sıkışma mikatrını bulunuz.
b) Yayda depolanan enerjinin cismin potansiyel enerji değişimine eşit olup oladığını
karşılaştırınız.

Kinetik sürtünme katsayısı 0,2 olan bir yüzeydeki blok şekildeki


9-
gibi yay ile 50 cm sıkıştırılıyor.
a) Blok yaydan ayrılırkenki hızı kaç m/s ‘ dir?
b) Yay serbest bırakıldığında blok ne kadar yol alır?

10- Düşey çalışan bir asansör, ağırlığı 4 ton olan durağan yükü 40
metre yukarıya sabit kuvvet ile 6 saniyede çekebiliyor. Asansörün
güç verimi %86 olduğuna göre bu işi yapmak için asansörün gücü
en az kaç kWatt olmalıdır?
7. BÖLÜM

Potansiyel Enerji ve Enerjinin Korunumu

7.1 Yerçekim Potansiyel Enerjisi


7.2 Esneklik Potansiyel Enerjisi
7.3 Korunumlu ve Korunumsuz Kuvvetler
7.4 Enerji Diagramları
Bölüm Sonu Soruları
Yerçekim Potansiyel Enerjisi
Enerjinin farklı türleri vardır. Yenilenebilir ve yenilenemez kay-
naklardan elde edilen bu enerji türleri “evrende yok olmaz ve
yoktan da var edilemez” olarak kabul edilir. Burada söyleyeceklerim
belki tartışma konusu olabilir ama enerjinin yoktan var edildiği
veya yok olabileceği hakkında merakınız varsa kendinizce belki
bir kanıya varmak için şu konuları araştırabilirsiniz: Evrenin oluşu
hakkındaki Büyük Patlama Teorisi (Big Bang), enerjinin gittikçe
kaybolduğu kuramı hakkında olan “exerji” ( Exergy and Energy
Decaying) ve enerjinin başka türe dünüşümü için Einsteigne’nin
Modern Fizik teorisindeki enerji-kütle dünüşümü.

Yerçekimi potansiyel enerjisi adından da anlaşılacağı gibi yerin çekim ivmesinden


doğan enerji çeşididir. Kütlesi olan cisimler yeryüzünden daha yüksek bir noktaya gö-
türülünce yerin çekim kuvvetine karşı iş yapılmış olur. Yerçekim ivmesi aslında kütesi
olan her cismin oluşturduğu kütle çekim ivmesinden doğuyor. Üzerinde bulunduğu-
muz Dünya’mız kütlesinin oluşturduğu bu ivme g=9.81 m/s2 değeri kdardır. Yerin
bu çekim kuvveti Newton’un 2. kanununa binaen F=m.a gibi W=m.g olur. Kuvvetin
dogrultusunda bir cisim hareket edilince biliyoruz ki enerji harcanarak iş yapılmış
olur, E=F.Dx . Yanda gösterilen m kütleli cismi h kadar yüksekliğe çıkarınca kazanılan
potansiyel enerjiye U dersek;

formölünü elde ederiz.

Potansiyel enerji cismin bünyesinde saklı durur, kararsız durumlarda cismi o konumda tutan engel
kalkınca cisim enerjisi bitene kadar aşağı iner. Potansiyel enerjiyi daha iyi anlamak için Newton’un
bizzat gözlemlediği örneği inceleyelim. Daldaki bir elmanın potansiyel enerjisi vardır.
Elma dalda durmaktadır ama sapının kopması halinde harekete geçeceği ve aşağı düşeceğini
biliriz. (enerji var ama henüz harekete geçmemiştir.) Daldaki elma, havadaki uçak, barajlardaki su,
dağlardaki kar gibi yerden belirli bir yüksekliğe sahip tüm cisimlerin bir çekim potansiyeli vardır.

ÖRNEK 7.1
Şekilde ebatları 10x30cm olan 50N ağırlıklı tuğlalardan bir duvar yapılacaktır.
Duvarın eni 1.4 m ve boyu 0.8 m olduğuna göre usta bu tuğlaları yerden teker teker
kaldırıp duvarı bitirince yerçekimine karşı kaç Joule iş yapar?

ÇÖZÜM
Yerçekimine karşı yapılan iş bir cismin kazandığı potansiyel enerjiye eşittir.
Her bir tuğlanın ağırlığı W=50 N ise tuğlaların yüksekliği lazım. Sakın her
toplam (1.4 x 0.8)/(0.1 x 0.3) = 37,33 bir tuğlanın ayrı ayrı duvardaki konumunu bulmaya çalışmayın :), gerek
tane tuğla gerekli yok çünkü toplam duvar ağırlığı ve duvarın kütle merkezinin yerden yük-
Duvar ağırlığı Pduvar= 37.33x50=1866N sekliği yeterli olacak. Duvar ağırlığı= Tuğla sayısı x Tuğlanın ağırlığı
Duvarın ağırlık merkezi tam ortada olacak h= 0.4m
Toplam iş : U=Pduvar x h = 1866N x 0.4m = 746 Joule iş yapılmış olur.
Esneklik Potansiyel Enerjisi
Bir cisim üzerinde kuvvet uygulandığında bu cismin şekli, hacmi ve boyu değişebilir. Eğer
uygulan kuvvet ortadan kalkınca cisim eski haline dönüyorsa bu cisme esnek cisim denir.
Esneklik potansiyel enerjisi, esnek cisimlerde şekil değişikliği oluşturulması sırasında
depolanan enerjidir. Bir yayı yatay olarak bir yere bağladığımızı düşünelim. Sonra yaya
doğru hızlı bir cismin geldiğini ve yayın ucuna çarrparak yayı sıkıştırdığını hayal edelim.
Hızlı gelen bu cisim kinetik enerjiye sahiptir. Cizim hızla yaya çarpınca yayı sıkıştırmaya
başlayacaktır. Cisim ise yavaşlayacaktır. Bu durumda cisim hızından dolayı sahip olduğu
kinetik enerjiyi yaya verecek yay da esneklik potansiyel enerjisi olarak kinetik enerjiyi
absorbe edecektir.
Yine bir atlama sporcusu koşarak kinetik enerji kazanır ve elindeki çubuğu yere
yaslayarak zıplar. Kinetik enerjisi çubuğa esneklik potansiyel enerjisi olarak aktarılır
ve çubuk esneyerek sporcuyu yukarı fırlatır. İşte sporcuyu yukarıya fırlatan bu ener-
ji çubuğun esneklik potansiyel enerjisidir. Daha sorna sporcu yere düşme sırasında
potansiyel enerjisini kinetik enerjiye dönüştürür. Hızla yere düşen sporcu minderin
üstüne çakılır. Burada sporcunun sahip olduğu kinetik enerji mindere çarpması ile
mindere esneklik potansiyel enerjisi olarak aktarılır ve bu sırada minder esnekliği
ile kinetik enerjiyi alarak sporcunun yumuşakça düşmesini sağlar. Hatta minder bir
miktar ısınarak esneklik potansiyel enerjisini ısı enerjisine çevirmiş olur.

Burada k sabiti, yayın birim sıkışması için gerekli olan kuvvete eşittir ve birimi de N/m idi. Yay, x kadar sıkış-
tırılırken F=k.x kuvveti bir iş yapar. Kuvveti uygulayan yay olduğu için işi yapan da yaydır. Yapılan iş yayda
depolanan esneklik potansiyel enerjidir. Bu enerji cismin kinetik enerjisinden gelmektedir. F=k.x kuvveti x’e
bağlı olarak değişen bir kuvvet olduğundan yaptığı işi F-x grafiğinin altındaki alandan bulabiliriz. Grafikteki
gibidir. Taralı alan, F=k.x kuvvetinin yaptığı işi dolayısıyla yayda depolanan enerjiyi verir. Buradaki x uzunlu-
ğu, sıkışma da olsa potansiyel enerji yine Eyay = kx²/2 dir.

Uygulamalarda farklı sabitlere sahip yaylar birleştirilerek


mekanik sistemler oluşturulabilir. Yanda gördüğünüz ilk
başta tek yaylı bir sistemde cisim yayı çekerken tüm enerjiyi
ona depolar. Eğer biz ikinci bir yayı parallel bağlarsak birim
uzama yapmak için daha büyük bir kuvvet uygulamamız
gerekir. Yani yeni durumda tıoplam yay sabiti büyüyerek
daha sert bir yaymış gibi hissederiz. Eşdeğer yay sabiti
sabitlerin cebirsel toplamıdır. İkinci durumda yayları seri
bağlayınca daha kolay çekeceğimizden eşdeğer yay sabiti iki
yayın sabitinden de küçük olacak. Eşdeğer yay sabiti yayla-
rın harmonik ortalaması olacaktır.
Korunumlu ve Korunumsuz Kuvvetler
Sürtünmenin olamadığı ideal sistemlerde yapılmak istenen iş ile
gerekli enerji tam olarak birbirine eşittir. Enerjinin korunumu
yasası ve sürtünmeden dolayı enerji kayıpları kaynak kuvvetlerin
korunumlu olup olmadığının göstergesidir. Kuvvetin korunumlu
olması enerjini korunumlu olması ya da olmamasıyla ilgilidir.
Eğer cisme uyguladığımız kuvvetin enerjisini daha sonra tek-
rardan herhangi bir kuvvet uygulamadan alabiliyorsak o zaman
kuvvet korunumludur. Diğer yandan uyguladığımız kuvvet ka-
dar büyüklükteki bir kuvveti tekrardan alamıyorsak etkisi başka
enerjiye dönüşmüş ve korunumsuz olmuştur. Yerçekimine karşı kaldırdığımız bir cismi serbest bıraktığımız-
da cisim yere düşmeden hemen önceki kinetik enerjisi tepe
noktasındaki potansiyel enerjiye eşitse kuvvet korunumludur.
Burda yerçekimi kuvvetinin korunumlu olması cismin tekrar
yere düşme isteği ile ilgilidir ancak enerjinin eşit olup olmaması
ortamdaki hava sürtünmesi ile ilgilidir. Korunumlu kuvvetlere
örnek verirsek kütle çekimi (özel hali yer çekimi) kuvveti ve yay
kuvveti ve diğer temel kuvvetler korunumlu kuvvetlere örnektir.
Korunumlu kuvvetlerin özellikleri:
1. Korunumlu kuvvetlerin yaptığı iş, cismin izlediği yoldan bağımsızdır. Sadece başlangıç ve bitiş noktalarına
bağlıdır.
2. Kapalı bir yol boyunca (başlangıç ve bitiş noktaları eşit) korunumlu kuvvetin yaptığı iş sıfırdır.
3. Korunumlu kuvvetler sadece konumun bir fonksiyonudur.
4. Korunumlu kuvvetler bir potansiyel enerji fonksiyonu ile ilişkilendirilebilirler.
5. Korunumlu kuvvetlerin yaptığı iş tekrar kazanılabilir. ( K kinetik enerji ve U potansiyel enerji arasında
karşılıklı dönüşüme izin veren kuvvetlerdir). Korunumlu kuvvetlerden kaynaklanan enerji de korunumludur.

Korunumlu olmayan kuvvetlerin asıl sebebi sürtünme kuvvetidir. Çün-


kü sürtünme kuvvetiyle ısıya dönüşen enerjiyi bir daha geri getiremeyiz.
Korunumsuz kuvetlerin yaptığı iş yola bağlıdır ve bu kuvvetler bir potan-
siyel enerji fonksiyonu ile ilişkilendirilemezler. Mekanik enerjinin (yani
kinetik enerji + potansiyel enerji) azalmasına neden olan korunumsuz
kuvvetlere yitirimli kuvvet de denir. Bu bakımdan korunumsuz kuvvete
en güzel örnek sürtünme kuvvetidir. Mekanik enerjiyi artıran korunum-
suz kuvvetler de vardır. Örneğin, oksijenin barutla kimyasal reaksiyona
girmesi sonucu patlayan havai fişek parçaları büyük bir kinetik enerji ile
havaya fırlar. Bu olay sonucunda serbest kalan kuvvetler korunumsuz-
dur, çünkü reaksiyon geriye döndürülemez.

ÖRNEK 7.2

Şekildeki gibi sürtünmeli yüzeyde cisimler serbest bırakılınca sabit hızla hareket
ediyorlar. Buna göre yatay düzlemin sürtünme katsayısını bulunuz?(g=9.81m/s2)

ÇÖZÜM
Sistem sabit hizla hareket ettiğine göre net kuvvet sıfırdır. Yani 3kg cismin ağırlığından dolayı oluşan sürtün-
me kuvveti 1kg lık cismin ağırlığıa eşit olmalıdır. Fs = m1g , sürtünme kuvveti Fs = m.m3g

m1g = m.m3g , m = m1/m3 m = 1/3


Enerji Diagramları

Sürtünmesiz ideal ortamlarda toplam enerji herzaman korunur. Mekanik sistemlerde toplam kinetik ener-
ji ile toplam potansiyel enerjinin cebirsel toplamına mekanik enerji diyoruz. Yerçekiminin etkisiyle aşağı
doğru yuvarlanan top maksimum yükseklikte iken hızı sıfır ve kinetik enerjisi sıfır olur. Cisim aşağı yuvarla-
nınca yükseklik olmadığı için tüm enerji kinetik enerjiye dünüşmüştür. Arada bir yerde cismi ele aldığımızda
potansiyel enerji ile kinetik enerjinin toplamı (mekanik enerji) yine aynı olcaktır.

ÖRNEK 7.3
Şekildeki sistemde sadece yatay düzlem sürtünmelidir.
m=3 kg lık cisim k=300 N/m lik yaya asılınca yay den-
ge konumundan x=6,5cm geriliyor. Buna göre;
a) Cisim ile yay arasındaki ip kopunca cisim kayarak
sürtünme katsayısı m=0.2 lik yüzeyde d=1m ötede iken
hızını bulunuz.
b) Eğik düzlem açısı a kaç derece olduğunu bulunuz.

ÇÖZÜM
a) Yükseklik potansiyel enerjisini aşağıya kayınca tüketen cisim bu enerjiyi yatay düzlemdeki sürtünmeye
birmiktar kaybettirecektir. Epot= Ekin + Esürt , m.g.h=1/2m.V2 +(m.m.g).d , m leri sadeleştirip Vyi yanlız bıra-
kırsak: V= kök(g.h-m.d) bağıntısını elde ederiz. değerleri yerine yazınca V= 4,4 m/s bulunur.

b) Sistem dengede durgun olduğu ilk durumda cismin eğik düzleme parallel ağırlık bileşeni yayın germe
kuvvetine eşittir. Fyay = m.g.cosa , Fyay = k.x , x= 0.065m olduğuna göre, k.x = m.g.cosa ve
a= arccos(kx/mg)= arccos(300x0.065/3x9.81), a = 48.50
Bölüm Sonu Soruları
1- Bir oyuncak tüfeğin atış işleyim (mekanizma) şekilde gösterildiği gibi kuvvet
sabiti bilinmeyen bir yaydan oluşmuştur. Yay 0,0120 m sıkıştırıldığında tüfek
düşey olarak ateşlendiğinde 35 g’lık bir mermiyi ateşleme öncesi konumunun
üzerinde 20 m’lik bir maksimum yüksekliğe fırlatabilmektedir. Tüm direniş
kuvvetlerini ihmal ederek yay sabitini bulunuz.

2- 70kg lık bir dağcı 106 m yüksekliğindeki bir yokuşu tırmanırken sırtındaki
15kg’ lık çantayı 30m çıktıktan sonra fazla enerji kaybetmemek için bırakıyor.
Dağcı yokuşun tepesine çıkınca toplam kaç Joule luk enerji harcamıştır?

3- Bir yayın serbest haldeki uzunluğu 300 cm’ dir. Bu yayı 10cm sıkıştırmak için 12.0 J’luk iş yapmak gerekti-
ğine göre yay sabiti kaç N/m’ dir? Ayrıca bu işi yapmak için gerekli kuvveti bulunuz.Örnek

4- Yandaki şekilde h=15m yükseklikten serbest bırakılan 4kg kütleli cisim h/3 seviyesine
geldiğinde kinetik enerjisi kaç Joule olur?( g=10m/s2)

5- Yay sabiti 40 N/m olan bir yayın önüne şekildeki gibi 5 kg lık cisim yayın
denge konumundan itibaren 25 cm sıkıştırılarak serbest bırakılıyor. Yatay
zemin sürtünmesiz olduğuna göre cismin yaydan ayrılınıcaki hızı kaç m/s 5 kg
olur?

6- Bir inşaat alanında işçi 65.0 kg beton dolu bir kova, hafif ama güçlü bir
halat ve bu halatın içinden geçtiği hafif ve sürtünmesiz makarayla yatay
bir zeminde duran 80.0 kg lık kutuya bağlıdır. Halat kutuya yatay olarak
bağlıdır ve kutunun üzerinde 50.0 kg’lık içi çakıl dolu bir çuval durmak-
tadır. Kutu ile zemin arasındaki kinetik sürtünme sabiti μk=0.400, satatik
sürtünme sabiti ise μs=0.700’dür.
a) Çuval ile kutu arasındaki sürtünme kuvvetini bulunuz.
b) İşçi bir anda çakıl dolu çuvalı kaldırıyor. Enerjinin korunumunu göz
önüne alarak beton dolu kovanın hareketsiz olduğu noktadan 2.00 m
aşağı indiğindeki süratini bulunuz.

7- 2.00 kg’lık bir blok, ihmal edilebilir kütleli ve kuvvet sabiti


k=400 N/m olan yayı 0.220 m sıkıştıracak şekilde ittiriliyor.
Blok serbest kaldığında önce sürtünmesiz yatay bir düzlemde
yol alıp daha sonra eğimi 37.0° olan sürtünmesiz bir düzleme
tırmanıyor.
a) Bloğun yaydan fırlatıldıktan sonra yatay düzlemde kayarken
sürati nedir?
b) Blok geriye kaymaya başlamadan önce eğik düzlem üzerinde
ne kadar yükselebilir?

8- Şekildeki m kütleli kutu q açılı sürtünmesiz eğik düzlem üzerinde h yüksekli-


ğinde serbest bırakılıyor. Kutu aşağı doğru hızlanarak kaydığına göre;
a) Eğik düzleme etkiyen normal kuvveti bulunuz.
b) Kutunun hareketi sırasında sahip olduğu ivme nekadardır?
h
c) Kutu aşağı inince nekadarlık hıza sahip olur? ( g = 9.81 m/s2 )
8. BÖLÜM

İtme-Momentum ve Çarpışmalar

8.1 Momentum ve İtme


8.2 Momentumun Korunumu
8.3 Çarpışmalar
8.4 Çarpışmalarda Enerji Korunumu
8.5 Ortak Kütle Merkezi
8.6 Momentumun Uygulamaları
Bölüm Sonu Soruları
İtme (İmpuls) ve Momentum

Bu bölümde kinematiğin bir konusu olan momentum hakkında konuşur-


ken öncelikle başlıkta da geçtiği gibi doğrusal (lineer) momentum bizim
odak noktamız olacaktır.
Momentum fiziksel olarak çarpma etkisi yada çarpma şiddeti olarak
bilinir. Formülsel olarak da kütle ile hızın çarpımı olarak yazılır. Hız
vektörel, kütle ise skaler bir büyüklük olduğu için vektörel bir büyüklü-
ğün skaler bir sayıyla çarpımı bize vektörel bir nicelik verecektir. Yönü ve
şiddeti önemli olan momentum aslında hareketin yönüne bağlı bir çarpma
etkisidir diye biliriz. Düşünün ki aynı hıza sahip bir futbol topu ile bir
tenis topu size çarpıyor. Hangisinin size vereceği çarpma etkisi daha fazla
oluyorsa onun momentumu daha büyüktür.

Doğrusal Momentum ve Korunumu

Kuvet-ivme ilişksinin bu konuda temel prensip olduğunu unutmamak gerekir. Newton’un 3. yasası
olan bu ilişki F=m.a dan F=m.(dV/dt), F.Dt=m.DV=DP olur. Eşitliğin sağ tarafı (F.Dt) yani kuvvetin
zamanla etkisine İTME(İmpuls) denir ve I ile gösterilir. Birimi Newton.saniye [N.s] dir. Bu etki cismin
momentum değişimine (DP ) eşit olur. Bu durum aslında kuvvetin cismi belirli bir zaman aralığında
itme miktarıdır. Gölle fırlatan sporcular gölleye ne kadar çok kuvvet uygulayarak (belirli bir zaman
aralığında) itmeyi denerse gölle o kadar büyük momentuma sahip olur ve o kadar uzağa gider. Durgun
bir duvara çarpan topun sekip zıt yönde gitmesi duvarın topa uyguladığı itme ile ilgilidir. Biliyoruz ki
momentum hıza bağlı vektörel bir büyüklüktür. Topun hız yönü(veya miktarı) değiştiği için momentu-
munda da bir değişme olacaktır. Momentumdaki bu değişiklik topun duvara temas ettiği kısacık süre
zarfında duvar tepki kuvveti ile o sürenin çarpımına yani itmesine eşit olacaktır.
Newton’ un yasalarından olan net kuvvet ivme ilişkisi F=m.a aslında lineer(doğrusal) momentumun
temel ilkesi gibidir. İvme yerine hızın zamanla değişimini yazarsak ve bu ifadeyi kütle ile çarpınca
momentumun zamana göre değişimi olur. Zaman differansiyelini (dt) kuffet ile çarpıp her iki tarafın
integralini alınca eşitliğin sağ tarafındaki terim bize itmeyi verir. Eşitliğin sol tarafı da momentum
değişimine eşit olur. Fiziksel olarak açıklaması; kuvvetin zaman aralığında bir cisim üzerindeki etki-
si cismin momentum değişimine sebep olur.
a=dV/dt
F=m.(dV/dt)
F.dt=m.dV= dP
F. Dt = DP

Newton der ki: Bir parçacığa etkiyen net kuvvet o parçacığın ivmeli hareketine veya hız değişimi-
ne sebep olur. Aslında bu hız ile kütle çarpımından momentum değişimine sebep olur. Düşünün ki
doğrusal yolda sabit hızla giden bir oyuncak arabaya arkadan bir kuuvet belirli bir sürede etkiyor ve
hızı iki katına çıkıyor. Momentumun formülünden P=m.V den ilk duruma göre momentumu da iki
katına çıkar.

P=m.V
m: kütle (kg)
V: hız (m/s)
P: momentum (kg.m/s)

Parçacıklar Sisteminde Momentum Korunumu

Doğrusal momentum korunumu Newton’un 3. yasasıyla ilişkilidir. Bu yasa etki-tepki prensibi ola-
rak bilinir ve bu ilişkideki kuvvetlerin yönleri birbirine zıttır ama şiddetleri aynıdır . Momentumları
farklı iki cismin çarpışmasında çarpışmadan önce sahip oldukları momentumlarının vektörel toplamı
dışardan sisteme bir etki olmadığı için çarpışmadan sonra sahip olacakları momentumları toplamına
eşit olacaktır.
P1,ilk + P2,ilk = P1,son + P2,son

m1.V1,ilk + m2.V2,ilk = m1.V1,son + m2.V2,son

F1 = dP1/dt
F2 = dP2/dt
dP1/dt + dP2/dt = (d/dt).(P1+P2) = 0
Çarpışmalar
Momentum uygulamarından birisi de cisimleri bir biri ile çarpışmasıdır. Bu çarpışmalar doğrusal, düz-
lemsel yada üçboyutlu gerçekleşebilir. Çarpışmlarda önemli kriter cisimlerin çarpışma öncesi ve çarpışma
sonrası toplam momentumlarının korunuyor olduğudur.

Tek Boyutta Esnek Çarpışma


Bu sistem örneğinde sadece bir boyutlu hareketleri ve çarpışmaları ele alcağız. Doğrusal yolda hareket
eden cisimler aynı doğrultuda momentuma sahiptirler. Çarpışma sonucunda da momentum aynı doğrultu
üzerinde olacaktır. Çarpışmanın esnekolması demek çarpışma sırasında ısıya veya başka bir forma enerji
dönüşmeden toplam kinetik enerjinin korunduğudur. Bu durumda ilk toplam momentum son toplam mo-
mentuma ve ilk kinetik enerji toplamı son kinetik enerji toplamına eşitlenir.
P1,ilk + P2,ilk = P1,son + P2,son

m1.V1,ilk + m2.V2,ilk =m1.V1,son + m2.V2,son

1/2m1V1,i2 + 1/2m2V2,i2 = 1/2m1V1,s2 + 1/2m2V2,s2

Denklemler düenlenip gerekli işlemler yapıldıktan sonra ; V1,ilk + V1,son = V2,ilk + V2,son

Özel denklemi bulunur.


Cisimlerin son hızları

V1,son = ((m1-m2)/(m1+m2)). V1,ilk + ((2m2)/(m1+m2)). V2,ilk

V2,son = ((2m2)/(m1+m2)) . V1,ilk + ((m2-m1)/(m1+m2)). V2,ilk olarak bulunur.


İki Boyutta Esnek Çarpışmalar
Daha önce iki parçacıklı sistemde momentumun doğrusal olarak korunduğunu söylemiştik. Bu sonuç
iki parçacıklı çarpışmalar için x, y, z doğrularının her birinde momentumun korunacağını ifade eder.
Kartezyen koordinat sisteminde sadece x ve y doğrultularında hareket varsa bu dözlemsel harekettir ve
momentum incelenmesi de düzlemsel olacaktır. Düzlemsel çarpşmalarda sistem dışardan izole edil-
diği için çarpışmadan önceki momentum bileşenleri (x ve y bileşenleri) toplamı aynı doğrultudaki son
momentum bileşenlerinin toplamına eşit olacaktır. Esnek çarpışma durumunda cisimlerin ilk kinetik
enerjileri toplamı da son kinetik enerji toplamlarına eşit olacaktır.

Pxi = Pxs Pyi = Pys

m1.V1xi +m2.V2xi = m1.V1xs +m2.V2xs m1.V1yi + m2.V2yi = m1.V1ys + m2.V2ys

m1V1cos(a)+m2V2cos(b)=m1V1’cos(t)+m2V2’cos(g) m1V1sin(a)+m2V2sin(b)=m1V1’sin(t)+m2V2’sin(g)
Bu çarpışma türü esnek olduğundan ve bileşke hız x-y bileşenlerinin vektörel toplamı şeklinde yazıldığından
kinetik enerji korunumunu tek bir denklemle gösterebiliriz
(1/2)m1V12 + (1/2)m2V22 = (1/2)m1V1’2 +(1/2)m2V2’2

Çarpışma Katsayısı, e
İki boyuttaki çarpışmalarda çarpışmanın esneklik dercesini veren ölçüte çarpışma katsayısı yada geri sıçrra-
ma katsayısı diyoruz. Bu ifadeyi “e” ile göterince asıl anlamı İngilizce’ den gelen “elastic buoyancy” ifadesinin
baş harfi olduğunu söyleyelim. Çarpışmanın derecesini ölçerken bibzim için önemli eksen çarpışma ekseni
olacaktır. Yani bu ifadeyi değerlendirirken momentumun değiştiği eksende çarpışma katsatyısı hesaplamaları
yapılır. Bu katsayı çarpışma sonrası bağıl hızların çarpışma öncesi bağıl hızlara oranı olarak açıklanır.

e=1 : tam elastik çarpışma


e=0 : tam plastik çarpışma
0<e<1 : elastoplastik çarpışma
e=1 : hem momentum hem de
enerji korunur.
e<1 : sadece momentum korunur.
Esnek Olmayan Çarpışmalar

Bu tür çarpışmalarda momentum korunurken çarpışma esnasında cisimler ısınır ve kinetik enerjiden
olur. Böylece toplam enerjiden azalma olur ve çarpışma sonrasında kinetik enerji ilk durumdan azdır.

(1/2)m1V12 + (1/2)m2V22 = (1/2)m1V1’2 + (1/2)m2V2’2 + Ekayıp


Çarpışma Sorularında;
1- İlk önce kağıt düzlemine koordinat eksenleri çizilir. (Genelde yatay kısım-x, düşey kısım-y alınır.)
2- Koordinat eksenleri doğrultusunda hızın bileşenleri Vx, Vy diye yazılır.
3- Çarpışmadan önceki eksenler doğrultusundaki momentumları hesaplanır.(Px=m.Vx, Py=m.Vy)
4- Çarpışma öncesi herbir eksen doğrultusudaki momentum korunum denklemleriyle çarpışma sonrası o
doğrultudaki herbir cismin hız bileşeni bulunur.
5- Esnek olan çarpışmalarda kinetik enerji korunum denklemi yazılır.
Kütle Merkezi

Hareketsiz cisimlerin kütle merkezleri bulunurken cismin önce şekline ve homojenliğine bakılıyordu. Eğer
cisim homojen ve simetrik geometrideyse kütle merkezi geometrik merkez ile çakışıktır. Ama cisimlerin
düzgün ve homojen olmadığı durumlarda kütle merkezi bulunurken her bir parçacığın kütle merkezine
olan moment katkıları parçacığın kütlesi ile kütle merkezine olan mesafenin çarpımıdır. Bu parçacıkların
oluşturduğu toplam momentleri toplarsak denge şartını elde ederiz. Yani kütle merkezine göre parçacıkların
momentleri toplamı sıfırdır. Aslında toplam kütleyi oluşturan parçacıkların koordinat düzleminde herhangi
bir noktaya göre oluşturdukları momentleri toplamına eşit büyüklükte moment oluşturan bir merkez vardır.
Bu momenti temsil eden merkez noktası toplam kütle burda toplanmış gibidir. İşte bu noktaya kütle merkezi
veya ağırlık merkezi denir.
*Orjine göre moment alınır; tüm kütlelerin orijine göre
momentlerinin toplamı kütle merkezinin orjine göre
momentine eşittir. Yani tüm parçacıkların ayrı ayrı
kütlelerini aslında kütle merkezi temsil eder.

XKM= (2kg.1m + 1kg.2 +3kg.3m)/(2kg+1kg+3kg)=2,17

YKM=(2kg.2m+1kg.1m+3kg.3m)/(2kg+1kg+3kg)=2,33

Dinamik yani hareketli cisimlerin anlık kütle merkezleri her zaman parçacıkların arasında bi
yerdedir. Bu nokta o parçacıkların çarpışmadan önceki kütle merkeziyle aynıdır.
Parçacık Sisteminin Hareketi

Parçacıkların hareketi toplam kütlenin kütle merkezinin hareketidir. Kütle merkezinin momentumu toplam
kütle ile kütle merkezinin hızının çarpımıdır. Buna göre momentum korunum denklemini yazarsak;

VKM= dXKM/dt = (1/M).(toplam(mi.dr/dt))=toplam(Vi.mi)/M

M.VKM = Ptop
Sisteme dıştan etki eden net kuvvet sistemin toplam kütlesi ile sistemin ivmesinin çarpımına eşit olduğuna
göre;

F=m.a=m.(dV/dt)= dP Fnet=DP

İç Patlamalar

Başlangıçta durgun olan bir cisim patlayınca ayrılan parçalar etrafa


savrulur. Bu parçalar patlamadan sonra momentuma sahip olur.
Başlangıçta durgun ilk hızsız sağlam cismin momentumu sıfır ise
sondurumdaki parçalarının toplam momentumları da sıfır yani ilk
duruma eşit olmalıdır.
Eğer patlama oncesi cisim sıfırdan farklı bir hızla hareket ediyorsa sahip olduğu momentum patlama
sonrası parçalarının da momentumlaının vektörel toplami ilk duruma eşit olur. Her iki durumda da
momentum korunur.

ÖRNEK:
Bir nişancı M=3 kg kütleli bir tüfeği gevşek olarak
yani tüfek geri tepebilecek şekilde tutmaktadır.
Kütlesi m=5 gr. Bir mermiyi yere göre Vm=300 m/s
hızla ateşliyor. Tüfeğin geri tepme sürati Vt’nin de-
ğeri nedir? Tüfeğin ve merminin son momentumu
ve kinetik enerjisi nedir?

ÇÖZÜM: Pt + Pm = 0 (ilk durum) = Pson = 0

0.005kg x 300m/s + 3kg x Vs = 0

Vs = 0.5 m/s
Roket Fırlatımı
Momentum korunumu ilkesine dayanarak gerçek hayatta mühendislik uygulamalrından biri de roketlerdir.
Uyduları yörüngelere taşımak için fırlatma rampalarında yerden kalkan roketler yerçekimini yenmek için
momentuma sahip olması gerekir. Roket fırlatımına start verilince yakıt tankından yanarak püskürttülen
maddenin momentumuna zıt yönde kalan kısım yukarı doğru hareket eder. Roketin dibinden püskürttü-
len madde yere bir itki sağlarken aynı zamanda roketin kütlesinde de azalmaya sebep olur ve faydalı yükü
taşıyan roket ucu ivmeli hareket ederek hızlanır. Burdaki temel prensip momentumun korunumudur.
Reaksiyon sonucu püskürttülen yakıt artıkları çok yüksek hıza sahiptirler. Hareketin herhangi bir anında
püskürtülen artıkların momentumu roketin kalan kısmının momentumuna eşittir.

Proket = Pgaz

mgaz.Vgaz = mroket.Vroket

ÖRNEK:
Bir roketin toplam kütlesi 1500 kg’dır. Roketten 2000 m/s hızla 300 kg’lık yakıt atıldığında roket kaç m/s’lik
hız kazanır?

ÇÖZÜM:
Bölüm Sonu Soruları
1- Kütlesi 5kg olan bir top 5 m/s hızla duvara çarparak ilk kinetik enerjisinin
yarısını kaybederek geri dönüyor. Buna göre;
a) Topun duvara çarptıktan sonra geri dönüş hızı kaç m/s dir?
b) Duvarın topa uyguladığı itme nekadardır?
c) Topun duvar ile temas süresi 0.035s ise duvarın uygalıdığı kuvvet nekadardır?

2- Sürtünmesiz yatay düzlemdeki cisimlerden ilkinin hızı 16 m/s kütlesi 3kg dır. Diğer cisminkütlesi 5kg ve
hızı sıfırdır. Hareketli cisim durgun olana çarptıktan sonra kenetlenip birlikte hareket ediyorlar. Çarpışma
anında sistemin enerjisinin %12,5 i kaybolduğuna göre sondurumdaki ortak hızlarını bulunuz.

3- Şekildeki baristik sarkaç deneyinde 400m/s hıza sahip 50 gramlık


mermi durgun L=1m lik iple asılı 10kg kütleli takoza saplanıp takozu
en fazla h kadar yükseltebiliyor. Çarpışma anında enerji kaybının
olamdığını düşünerek(g=10 m/s2);
a) Maksimum yüksekliği, h, bulunuz.
b) Maksimum q açısını bulunuz.

4- Şekildeki gibi m1= 4kg , m2= 5kg kütleli iki cisimlerden ilkinin hızı
+x yönüne doğru +24 m/s iken diğerinin hızı -20 m/s ‘dir. Cisimler
merkezi esnek çarpışma yaptıklarına göre çarpışmadan sonraki son
hızlarını ve çarpışma katsayısı, e’yi bulunuz.

5- 3. sorudaki sistemin aynısını göz önüne alarak mermi takozu delip geçtiğini düşünün. Takoz ve mermi
h/2 kadar yükseldükten sonra mermi sistemi 10 m/s hızla terk ettiğine göre ilk enerjinin yüzde kaçı çarpışma
sırasında kaybolduğunu bulunuz.
6- Bir golf topu h1 = 80cm yükseklikten serbestçe bırakılıyor. Top rijit yüzeye çarptıktan sonra h2 = 25cm
yukarıya zıplıyor. Golf topu ile yüzey arasındaki çarpışma katsayısı kaçtır? g=10 m/s2 , e=?

7- Bir cisim A noktasından v0 =36m/s hızı ile yatay doğrultuda fırlatıldıktan sonra
sırasıyla B, C ve D’ye çarpıyor. Duvarlar arası mesafe 18m ve çarpışma katsayısı
e = 1/3 olduğuna göre h kaç metredir?(g=10 m/s2)

8-
Şekildeki 175kg lık kayık üzerinde bulunan 75 kg kütleli adam suya atlamak
istiyor. Adam kayık yüzeyü ile 370 açı yapacak şekilde atlıyor. Adamın kayıktan
ayrılma hızı 15 m/s olduğuna göre kayığın ters istikamette gidiş hızını bulunuz.

9- Şekildeki gibi 11kg kütleli bomba iç patlama


sonucu üç parçaya ayrılıyor. Patlama sonucu 1.
ve 3. parçanın hızları ile kütleleri yanda verildiği
gibi ise 2. parçanın hızını iki boyutlu bileşenleri
halinde bulunuz.

10- 1500 kg kütleli bir roketi uzaya fırlatınca yerçekimine karşı koyabilmesi için 450 m/s hızla
gitmesi lazım. Roketin bu hıza ulaşabilmesi için ne kadar debili gaz püskürtmesi lazım?
9. BÖLÜM

Katı Cisimlerin Dönme Hareketi

9.1 Açısal Hız ve Açısal İvme


9.2 Sabit Açısal İvme ile Dönen Cisimler
9.3 Açısal ve Teğetsel Değişkenler
9.4 Döngüsel Harekette Enerji
9.5 Parallel Eksenler Teoremi
9.6 Atalet Momentinin Hesaplanması
Bölüm Sonu Soruları
Bu bölümde sabit bir eksen etrafında dönen cisimleri inceleyeceğiz. Ele aldığımız cisimlerin katı olması
gerekir. Maddenin katı hali dönme hareketi sonucu şeklinin bozulmamasına olanak verir.
Katı cisim: Cismi oluşturan moleköller sabit konumlarda ve zamanla cismin şekli bozulmadan içindeki
parçacıkların sadece titreşim hareketi yaptığı maddde fazıdır.

Açısal Hız-Çizgisel Hız ve İvme

Katı cisimler öteleme hareketi yapmadan sabit bir eksen etrafında sabit hızla
dönünce düzgün dönme hareketi yaparlar. Eşit zaman aralıklarla eşit açı tarama
hızına sabit açısal hız denir ve 'w’ ile gösterilir. Açısal hızın birimi 360 dereceyi 2p
radyana çevirince ‘rad/s’ (radyan/saniye’) yada ‘rpm’ (devir/dakida) alınır. Açısal
hızı sabit olan bir cisim üzerinde dönme merkezinden kenarlara kadar olan tüm
parçacıkların açısal hızı aynıdır. Çünkü cisim bir devir attıktan sonra cismi oluş-
turan bütün parçalarının birbirlerine göre konumları değişmez. Parçacıklar dışa
doğru savrulmak isteseler bile madde katı olduğu için birbirinden ayrılamazlar.

Çizgisel hız: Herhangi bir cismin aldığı mesafenin uzunluk ölçüsü cinsinden geçen süreye oranıdır. Hızın
tanımı zamanla yerdeğiştirme olduğuna göre dönen cisim üzerindeki bir noktanın çizgisel hızı süre içinde
aldığı mesafedir. Dairesel harekette çizgisel hız yarıçap ve açısal hız ile doğru orantılıdır. Çizgisel hıza aynı
zamanda teğetsel hız da denir. [V= w.r] formölü ile bulunan çizgisel hızın birimi normal hız gibi ‘m/s’ (met-
re/saniye) dir. Dönen cismin merkezindeki noktanın çizgisel hızı sıfırdır. Yani bu nokta sadece kendi etrafın-
da döner zamanla yol almaz. Ama merkezden uzaklaştıkça tüm parçalar aynı süre içinde aynı tur attıklarına
göre yarıçapı dolayısıyla çevresi fazla olan halka üzerindeki parçacık daha çok yol alır ve daha büyük çizgisel
hıza sahiptir. Demek çizgisel hız hem açısal hıza hem de yarıpa bağlıdır (V= r. w).

Çizgisel hız vektörel bir büyüklük olduğu için zamanla şiddeti değişmezken yönü
çember üzerinde sürekli değişir. İvme hızın zamana göre değişimi olduğundan
dairesel harekette hızı sürekli merkeze çeken bir kuvvet vardır ve bu kuvvet iv-
meyi oluşturur. Bu ivmeye merkezcil (radyal) ivme denir.
* Merkezcil ivme ile çizgisel hız vektörü her zaman birbirine dik yönlüdür.

Sabit bir eksen etrafında dönerken katı cisim


üzerindeki her parçacığın açısal hızı aynı olur.
Diğer yandan çizgisel ivme çizgisel hızın zama-
na göre değişimine eşit olan bir vektörel büyük-
lüktür.

olur ki buna çizgisel ivme denir.


* Bu ivme herzaman çizgisel hız ile aynı doğrul-
tudadır ve yarıçap vektörüne diktir
Sabit Açısal İvme ile Dönme Kinematiği

Doğrusal harekette olduğu gibi düzgün dairesel harekette de sabit ivme ile dönme hareketi incelenecektir.
Yani cisim ilkin durgun iken dönmeye başlayacak ve ivme ile giderek hızlı dönecektir.

2 2

Açısal ve Teğetsel Değişkenler

Dairesel harekette cismin anlık ivme ve hız bileşenlerini incelersek, ismin


çizgisel hızının miktarını değiştiren ve sürekli hareketin olduğu çembere teğet
olan ivmeye çizgisel (teğetsel) ivme denir. Teğetsel ivme herzaman çizgisel
hızın doğrultusundadır.
teğetsel ivme

Cismin çizgi üzerinde ve hareket yönündeki hıza teğetsel (çizgisel) hız demiştik.
Şimdi de bu hızı değiştiren ivmeleri ele alalım. Cismin çizgisel hızının yönünü
değiştiren ivmeye merkezcil(radyal) ivme denir. Bu ivme herzaman hız vektörüne
diktir.
merkezcil ivme

* Cismin toplam ivmesi teğetsel ivme ile merkezcil ivmenin vektörel toplamıdır.

aq= teğetsel ivme


* cismin hareketinin çizgisel hızın-
daki değişmeyi sağlar

ar= radyal ivme


* cismin hareketinin yönündeki
değişmeyi sağlar
ÖRNEK 9.1

Şekildeki gibi, 0.5 m uzunluğunda bir ipin ucuna bağlanan bir


top, yerçekiminin etkisi altında düşey bir daire çerçevesinde
salınmaktadır. Cisim, düşeyle θ=20o’lik açı yaptığı zaman top 1.5
m/s’lik hıza sahiptir.
a) İvmenin bu andaki radyal (ar) bileşenini bulunuz
b) θ=20o olduğu zaman teğetsel ivmenin (at)büyüklüğü nedir?
c) θ=20o’de toplam ivmenin (a) büyüklüğünü ve doğrultusu
nedir?
ÇÖZÜM:

θ=20o V=1.5 m/s


a) Merkezcil ivme: ar=V2/r=(1.5m/s)2/(0.5m)=4.5 m/s2
b) Teğetsel ivme: at=g.sinθ=(9.8m/s2).sin(20o)=3.4 m/s2
c) a=ar+at İvmenin büyüklüğü: |a|= (ar2+at2) 1/2 = [(4.5m/s2)2+(3.4m/s2)2] 1/2 =5.6 m/s2
yönü: φ = arctan(at/ar) = arctan[(3.4m/s2)/(4.5 m/s2)]=37o bulunur.

ÖRNEK 9.2
Şekilde görüldüğü gibi 0.5 km kütleli bir top, 1.5 m
uzunluğunda kablonun ucuna bağlanmıştır. Top, yatay
düzlemde dairesel yörüngede sabit hızla döndürülüyor.
Kablo 50 N’luk maksimum gerilmeye dayanabiliyor.
Kablo kopmadan önce topun sahip olabileceği maksi-
mum çizgisel hız nedir?

ÇÖZÜM :

∑T = m.ar = m.V2/r
T = m.V2/r => V = (r.T/m)1/2
Vmax = (r.Tmax/m)1/2 Vmax = [(1.5m).(50N)/(0.5kg)]1/2 = 12.2 m/s

ÖRNEK 9.3
Şekilde görüldüğü gibi 1500 kg kütleli bir araba düz bir yolda
35 m yarıçaplı bir virajda hareket etmektedir. Kuru zemin
için yol ile tekerlekler arasındaki statik sürtünme katsayısı
ks= 0.5 ise arabanın emniyetli olarak dönebilmesi için sahip
olabileceği maksimum hızı kaç m/s dir?

ÇÖZÜM :

Fr=m.ar= m.V2/r ,
Fs,max = ks. N , N = m.g
Vmax = (r.Fs,max/m)1/2 = (ks.m.g.r/m)1/2 = (ks.g.r)1/2
Vmax=[(35m).(0.5).(9.8m/s2)] 1/2=13.1 m/s
Dönme Enerjisi

Doğrusal yolda hareket eden cisimlerin kinetik enerjiya sahip oldukları gibi bir eksen etrafında dönen
cisimler de dönme kinetik enerjisine sahiptirler. Bu enerji aslında cismi oluşturan alt parçacıkların sahip
olduğu çizgisel hızlarından oluşan doğrusal kinetik enerjileri toplamıdır.

Cismin atalet(tembellik) momentidir.


Harekete karşı koyduğu eylemsizlik mo-
mentidir. Birmi [kg.m2]

*Cismin dönme kinetik enerjisi dönme (açısal) hızına bağlı olduğu gibi cismin şekline de bağlıdır.
Dönmede açısal hız sabit iken parçacıkların dönme eksenine göre konumu önemli bir kriterdir. Atalet
momenti merkezden uzaklığın karesiyle doğru orantılı olduğuna göre kütleli parçacıklar merkezden
uzaklaştıkça eylemsizliği artar ve hareketi sırasında dönme kinetik enerjisi büyük olur.

Eylemsizlik Momentinin Hesabı

Eylemsizlik momenti(I) ne cismi dönmeye karşı olan direnci ya da dönmeme ataleti demiştik. Bu ata-
let cismi oluşturan bütün parçacıkların kütlesi ile konumlarının karesiyle çarpımıdır. Yani kütle cismin
dönme merkezine nekadar uzak ise dönmeye karşı direnci de o kadar fazladır. Cisim homojen olursa kütle
hesabını birim yoğunluk ile hacmin çarpımı şeklinde yazılabilir. Burda eylemsizlik momentinin hacim-
sel olarak hesaplamanın vereceği kolaylık hacim hesaplanırken konum değerleri yani cismin boyutlarını
kullanabilmektir.

Paralel Eksenler Teoremi

Yandaki şekilde cismin kütle merkezinden geçen eksene


göre eylemsizlik momenti IKM olsun. Bucisim x-y koordinat
düzleminde şekildeki gibi yerleştirilmiş ise cismin y’ ekseni-
ne göre dönmesi sonucu daha büyük bir eylemsizliğe sahip
olası beklenir. Çünkü biz dönme eksenini y den y’ eksenine
taşıdık. Böylece tüm parçacıkların konumu dönme eksenine
göre daha uzakta kaldı. Yeni eylemsizlik momenti I’ olsun;
ilk durumdaki eylemsizlik momenti ile yeni durumdaki
uzaklık katkıları toplamı I’ yı vercektir.

eksene olan
uzaklık
cismin
yeni eksene kütle merkezinin toplam
göre eylemsizlik eylemsizlik kütlesi
momenti momenti
Farklı Geometrideki Cisimlerin Eylemsizlik Momentleri
Tork

Konumuz dönme hareketi ve dönmenin mekanik incelenmesi olduğundan anlık dönmeye sebep olan
etkiye değinmeden geçmek olmaz. durgun bir cismi doğrusal bir yolda hareket ettiren ve hız değişimine
sebep olan kuvvet nasıl önemli ise, dönme hareketinde dönmeye sebep olan etkiye de tork denir.

* Tork: Kuvvetin bir eksen etrafında dönebilen bir cisim üzerindeki döndürme etkisidir. Kapı kolları
kapının menteşelendiği kenara en uzak olacak şekilde yapılmasının sebebi küçük kuvvetlerle kapıyı ra-
hatlıkla açabilmektir. Buradan bir cismi döndürme etkisinde yani torkta döndüren kuvvet önemli olduğu
kadar kuvvetin etki ettiği yerin dönme noktasına olan uzaklığı da önemlidir.

* Tork döndürücü kuvvet ile dönme noktası uzaklılarının vektörel çarpımıdır.

*Kuvvet cismi saat yönünün tersine döndürürse tork pozitif(+), saat yönünde dödürürse negatif(-) işaret-
li olur.

Tork en iyi sağ el kuralı ile açıklanır. Dört parmak kuvvetin yönünü göstermek için doğrultulunca dönme
ekseni avuç içinde kalır ve baş parmak torkun yönünü gösterir. Yani cisim saat yönünün tersine döner ve
tork pozitif yönlü yukarı doğru olur.
Tork ve Açısal İvme Arasındaki Bağıntı

Doğrusal harekette cismin ivmeli hareketine neden olan net kuvvet olduğu gibi dönme hareketinde
de tork cismin ivmeli dönmesine sebep olur. Tork kuvvet ile kuvvetin döndürme ekenine olan uzaklığı
çarpımı olduğundan (t=F . d) cismin hareketi açısal bir ivmeye sahip olacaktır. Cismin döngüsel hareketi
torktan dolayı ivmeli hareket olacak ve bu ivme tork ile doğru orantılı artacaktır.

Açısal ivme ile tork arasındaki ilişki doğrusal harekette olduğu gibi bir orantı sabitine bağlıdır. doğrusal
harekette kuvvet ile ivme oranı cismin kütlesne eşit idi. Dönme hareketinde bu sabit cismin eylemsizlik mo-
mentidir. Cismin eylemmsizlik momenti cismin kütlesinin dönme eksenine göre dağılımını ifade ediyordu
ve dönmeye karşı gösterdiği eylemsizlik idi.
Dönme Hareketinme İş, Güç-Enerji

Fizikteki işin kuvvetle alınan yolun çarpımı olduğunu biliyoruz. Net kuvvetin oluşturduğu tork da
dönen cisimlere dönme enerjisi kazandırır. Doğrusal yolda kuvvetin yol ile çarpımı yerine burda kuvvet
zaten yarıçapla çarpılınca torku vermiş oluyor. Ama fizikteki işin kuvvet doğrultusunda alınması gerek-
tiği yani kuvvet ile alınan yolun paralel olması gerektiği unutlmamalıdır. Dönme hareketinde ise kuvvet
ve yarıçap vektörü birbirine dik olduğu için çarpımları işi vermeyecektir. Bunun yerine farklı bir boyutta
yapılan işlemler bize işi vercektir. Farklı bir boyut derken neyi kastettiğimizi açıklayalım. Tork nedeniyle
cisim öteleme hareketiyle yol almaz ama açısal olarak döndüğünden burda alınan yol yerine taranan açı
bizim için önemli olur . Yarıçap ne kadar açı tararsa kuvvet dalayısıyla tork okadar iş yapmış olur. Yani
tork ile açısal hızın çarpımı (t.w) bize dönme hareketinde birim zamanda yapılan işi (DW/Dt) verecek-
tir. Bu da güce eşittir (P= t.w).

Bir cisim dönerken cismi oluşturan tüm parçacıklar aslında hareket halindedir. Herbir parçacığın çizgisel
hızından dolayı sahip olduğu kinetik enerjileri cismin toplam dönme kinetik enerjisine eşittir. Dönme ki-
netik enerjisi cismin açısal hızına ve toplam kütlesine bağlıdır. Doğrusal hareketteki kinetik enerji formö-
lüne oldukça benzerdir. Dönme kinetik enerjisi:
ÖRNEK 9.4

Yandaki şekilde verilen diskin kütlesi 1.24 kg


ve yarıçapı ise 0.8 m’ dir. 15 N.m şiddetinde bir
moment cisme etkimektedir ve yayın rijitliği
k=10 N.m dir. Yay başlangıçta uzamamış durum-
da iken tekerlek durağan halden harekete geçiyor
ve kaymadan yuvarlanıyor. Silindirin merkezi
0.5 m ilerledikten sonra, tekerleğin açısal hızını
bulunuz.

ÇÖZÜM :

Yapılan iş: U1-2 = -1/2k[(x2)2–(x1)2] + M.( θ 2– θ 1) Dx = x2 - x1 = 1m (M noktası 0.5m ilerlerken A


x1 =0 , x2=2(0.5) noktası iki katı yani 1m yol alır.)

U1-2= -1/2(10)(12–0) + 15(0.5/0.8) = 4.375 N.m Dq = Dx /r

Kinematik İlişki: VM= r ω= 0.8ω


Kinetik Enerji: T1= 0(durağan halde)
T2= 1/2m (VM)2+ 1/2IM ω2
T2= 1/2(1.24kg)(0.8ω)2+ 1/2(1.24kg)(0.8)2ω2
T2= 0.7936 ω2

İş ve Enerji Prensibi: T1+ U1-2= T2


=0 + 4.375 = 0.7936 ω2
ω= 2.35 rad/s
ÖRNEK 9.5

Makaraya sarılı ipin A ucu sabit ivme ile çekilirken t = 0 anında B


noktasının toplam ivmesi 5 m/s2 ve hızı 2 m/s olduğuna göre ipin A
ucunun ivmesi kaç m/s2 dir? Yarıçap r = 1 m.

ÇÖZÜM :

ÖRNEK 9.6

Yarıçapı R, kütlesi M ve eylemsizlik momenti I olan bir


tekerlek sürtünmesiz yatay bir mil üzerine monte edilmiştir.
Tekerlek etrafına sarılı hafif bir ipin ucunda m kütleli bir
cisim vardır. Tekerleğin açısal ivmesini , asılı cismin çizgisel
ivmesini ve ipteki gerilmeyi bulunuz.
Bölüm Sonu Soruları

1- Yarıçap r olan makaraya sarılı ipin A ucu 7 m/s2 sabit ivme ile çekilirken t = 0 anında
B noktasının toplam ivmesi aB = 15m/s2 dir. Yarıçapı hesapladıktan sonra bulacağınız B
noktasının hızı VB kaç m/s dir?

2- Şekildeki gibi O noktasında menteşelenmiş sistem kurulmuştur. Çubuk


üzerinde serbestçe kayabilen1kg kütleli cisim başta o noktasındadır.
Çubuk sabit w= 3rad/s açısal hız ile dönmeye başlayınca cisim O nokta-
sından 0.2m uzaktayken sahip olacağı toplam hız kaç m/s olur?

3- Kütlesi 50 gram olan bir mermi şekildeki gibi 20kg loık 2m uzunluğundaki
çubuğun ortasına 400m/s hızla saplanıyor. Mermi ve çubuk birrlikte O noktası
etrafında döndüklerine göre merminin saplamasından hemen sonraki açısal hız
kaç rad/s olur? ( I = mL2/12 ve g = 9.81m/s2 alınız. )

4- Şekildeki gibi herbrinin uzunluğu 2L ve kütleleri m iki çubuktan


oluşmuş sistem B noktası etrafında serbestçe dönebilmektedir. Başlan-
gıçta durgun olan bu sisteme görüldüğü gibi AB nin tam ortasına 8P
lik kuvvet yukardan uygulanırsa çubukların açısal imeleri oranı a1/a2
kaç olur?

5- 1200 kg kütleli bir araba düz bir yolda, 45m yarıçaplı bir virajda hareket
etmektedir. Yol ile tekerlekler arasındaki statik sürtünme katsayısı kuru
zemin için 0,70 ise, arabanın emniyetli olarak dönebileceği maksimum hızı
bulunuz.

6-

Şekildeki düzenekte w = 12 rad/s açısal hızla döndörülen kol yardımıyla 30


cm uzunluktaki ipe bağlı m=2kg kütleli cisim döndürülüyor. Sürtünmesiz
olan bu sistamde r1/r2 =1.5 olduğuna göre T ip gerilmesi kaç Newton dur?

7- Bir mühendis, arabaların sürtünmeye güvenmeksizin savrulmadan dönebilecekleri eğimli bir otoyol virajı
yapmak istiyor. Başka bir deyişle, yol buzlu olsa bile araba belirlenen hızla kaymadan virajı dönebilmelidir.
Bir arabanın böyle bir virajı 15 m/s’ lik hızla dönebileceğini varsayınız. Virajın yarıçapı da 60m olduğuna
göre yolun eğimi kaç derece olmalıdır?

8- Sürtünmesiz eğik düzlemin 12m yüksekliğinde serbest bıra-


kılan 2kg’lık, 10cm yarıçaplı disk serbest bırakılıyor. Disk yatay
zeminde V öteleme hızı ile w açısal hızlarla hareket edince sahip
olduğu toplam kinetik enerji kaç Joule’dur?

9-
Yarıçapı 0.5 m olan diskin ağırlığı 2N dur. Disk w =10 rad/s açısal hızla döndörülerek
yatay zemine düşürülüyor. Disk ile zemin arasındaki sürtünme katsayısı m=0.3 olduğuna
göre diskin zemine değdiği andan itibaren alacağı yol kaç metredir? (g = 9.81m/s2 )

10- Yandaki şekilde görüldüğü gibi kütleleri 3kg olan iki cisim 1m
uzunluktaki ağırlıksız çubuklarla G noktası etrafında döndürülünce
toplam atalet momenti kaç kg.m2 olur?
10. BÖLÜM

Döngüsel Harekette Açısal Momentum

10.1Yuvarlanma
10.2 Katı Bir Cismin Açısal Momentumu
10.3 Net Tork ve Açısal İvmenin Açısal Momentuma Katkısı
10.4 Açısal Momentum Korunumu
Bölüm Sonu Soruları
Düzgün dairesel harekette dönen cismin sabit bir dönme ekseni etrafında hareketini gerçekleştirdiğine
ve bu dönme ekseninin sabit olduğunu sölemiştik. Bu konuda ise katı bir cisim dönerken aynı zaman-
da ötelendiğini yani dönme ekseninin de hareketli olduğuna değineceğiz.

Yuvarlanma
Bir cisim hem dönüyor hem de ötelenniyorsa bu harekete yuvarlanma denir. Yuvarlanma hareketinde
cismin yer düzlemi ile sürtünmeden hareket edeceğini diğer bir ifadeyle kaymadan hareket ettiğini varsa-
yacağız. Yuvarlanan bir cisim olarak araba tekerleğini ele alalım. Tekerlek yuvarlanınca üzerindeki bir nok-
tayı incelediğimizde karmaşık bir hareket yapar. Noktanın izlediği yol çizilirse tepelere benzer bir görüntü
ortaya çıkar. Bu tür harekete sikloid hareket denir. Yuvarlanma hareketi yapan cismin dönme ekseni
kendi etrafında w açısal hızı ile dönecek ve V doğrusal hızı ile ilerleyecektir.

sikloid

Yandaki şekilde sağa doğru kaymadan yuvarlanan


bir tekerlek görülmektedir. Kayma olmadığı için
tekerleğin yer ile temas noktasının hızı yerdeki bir
gözlemciye göre sıfırdır. Tekerleğin merkez nok-
tasının yere göre hızı yerden yarıçap kadar yüksek
olduğundan ve tekerleğin açısal hızı da w oldu-
ğundan V= w.R olur. Tekerleğin en üst noktası ise
yerden 2R yükseklikte ve w açısal hızla döndüğün-
den hızı 2V= w.2R olur.

*Tekerlek hem dönerek hem de ilerleyerek hareket ettiğinden sahip olacağı toplam kinetik enerji
dönme kinetik enerjisi ve öteleme kinetik enerjisi toplamıdır.
Katı Bir Cismin Açısal Momentumu

Doğrusal yolda hareket eden cismin momentumunun cismin kütlesi ile hızının çarpımı olduğunu öğren-
miştik. Şimdi de aynı cisim dairesel bir pistte aynı hız ile düzgün dairesel hareket yapıyor olsun. Cisim sü-
rekli aynı pistte dolandığı için merkezden cisme olan yarıçap vektörü birim zaman da aynı açıyı tarar. Pist
üzerinde cismin çizgisel hızı sabit olduğundan dorusal momentumu sabit kalır ve bu momentum döngüsel
harekette dönme merkezine göre tork oluşturur. Doğrusal momentumun sabit dönme ekseni etrafındaki
bu etkiye açısal momentum denir. ve ‘L’ ile gösterilir. L=(m.V).r , L =P.r

Açısal momentum çizgisel momentumun


dönmesiyle oluşur. Açısal momentumun
büyüklüğü;
-dönme yarıçapıyla,
-dönme hızıyla,
- cismin kütlesiyle
doğru orantılıdır. L=m.V.r
Yani açısal momentum doğrusal momen-
tumun oluşturduğu tork kadardır.

Süreksiz tekil cisimlerin açısal momentumu doğrusal momen-


tumun oluşturacağı tork kadar iken sürekli yani dönme merke-
zinden itibaren kütleli cisimlerin açısal momentumu cismi oluş-
turan herbir parçacığın açısal momentumları toplamı kadardır.
Biraz uzun bir cümle oldu galiba:). Azıcık açıklayalım bu cümle-
yi. Yanda görüldüğü gibi her yerinde kütlesi düzenli dağılmış ho-
mojen bir diski ele alalım. Bu disk merkezinden dik geçen bir ek-
sen etrafında dönerken diski oluşturan eşit kütleli parçacıklardan
herbiri birer tekil cisim olsun. Bu parçacıkların açısal hızları aynı
fakat merkezden uzaklaştıkça çizgisel hızları artacak şekilde fark-
lı olur. Kütleleri aynı çizgisel hızları farklı olduğu için doğrusal
dolayısıyle açısal momentumları da farklı olur. Tüm diski oluştu-
ran bu parçacıkları düşündüğümüzde diskin kendisi sürekli yani
herhangi biryerinde boşluk olmayan bir cisim olur. Diskin top-
lam açısal momentumu da kendisini oluşturan bütün parçacık-
ların açısal momentumlarının toplamı kadardır. Yani cisim ayrık
ya da sürekli olsun toplam açısal momentum sistemi oluşturan
parçacıkların açısal momentumlarının toplamları kadardır.
Net Tork ve Açısal İvmenin Açısal Momentuma Katkısı

Dönen katı cisimlerin açısal momentumu cismin eylemsizlik momenti ile


açısal hızının çarpımı şeklinde yazılabilir. ( L= I. w). Açısal momentumun
yönü çizgisel momentumun dönme ekseni etrafında oluşturduğu moment-
tin yönündedir.Yani açısal momentumun yönü cismin dönme yönüne göre
değişir ve sağ el kuralı ile bulunur. Dönme ekseni avuç içinde kalacak şe-
kilde dört parmak dönme yönünde olunca baş parmak açısal momentum
yönünde olur. Cisim saat yönünün tersine dönerse açısal momentum yukarı
ve pozitif, saat yönünde dönerse açısal momentum aşağı ve negatif olur.

Açısal momentumdaki değişim kendisini oluşturan açısal hız ve eylemsizlik momentinin değişmesine
bağlıdır. Bu değişim dıştan bir etki ile gerçekleşir. Sistemin kendi içinde yapacağı değişimlerle açısal
momentum değişmez. Dıştan yapılacak etkiler cismin dönme hızını artıracak net tork ya da sistemi ya-
vaşlatan sürtünmeler olabilir. Bu etkiler sonucunda eylemsizlik momenti sabit kalır ve açısal momentum
dönme hızıyla aynı oranda değişir. Sistemin kendi içinde yapılacak etkiler cismi oluşturan alt parçacıkla-
rın yerlerinin değişmesidir ki bu da eylemsizlik momentinin değişmesidir.
Açısal Momentumun Korunumu

Eylemsizlik momentinin artışı dönme hızını yavaşlatırken azalışı dönme


hızını arttırır. (L= I. w). Demek ki dönme hızını artırmak için ya dışardan
net bir tork uygulanmalı yada eylemsizlik momentini artırmak için birşeyler
yapılmalıdır.
Peki eylemsizlik momenti nasıl değiştirilir? Basit bir cevap olarak kütle mik-
tarını değiştirmek akıllara gelebilir ama buna ek olarak kütleyi değiştirmeden
kütlenin dönme eksenine göre uzaklığı değiştirilince eylemsizlik momenti
değişir. Çünkü eylemsizlik momenti birim kütle ile o kütlenin eksene olan
uzaklığının karesi ile çarpımıydı (I= m.r2). Bu durumu açıklarken en gü-
zel örnek buz pateni yapan bir kişinin kollarını açıp yada kapatınca dönme
hızının değişmesi verilebilir. Kolları açıkken L açısal momentuma sahip olan
patenci I eylemsizlik momenti ve w dönme açısına sahip olsun.Kollarını ka-
patınca kütlesinin dönme mekezine olan uzaklığı azalır dolayısıyla eylemsiz-
lik momenti de azalır. Dışardan sisteme etki olmadığı için açısal momentum
sabit kalır. Yeni durumda eylemsizlik momenti diyelim ki I/2 olsun. Kollarını
kapatınca değişiklik sistemin içinden olduğundan yani dışardan sisteme bir
etki olmadığından açısal momentum sabit kalır. Böylece dönme hızı da arta-
rak iki katına çıkacaktır. L=(I/2).(2 w)
ÖRNEK 10.1

Sürtünmesiz yatay bir masada, bir hokey topu masanın ortasından geçirilen bir ipe bağlı olarak sabit
bir uzaklıkta (yarıçapta) düzgün dairesel hareket yapmaktadır. Eğer ipi çekip yarıçapı yarısına düşersek
topun açısal momentumu kaç katına çıkar?

ÇÖZÜM :

İp dönme merkezindeki bir delikten çekildiğinden tork yoktur, dolayısıyla açısal momentum korunur.
L1 = I1w1 = mR2w1 = L2 = I2w2 = m(R/2)2w1

mR2w1 = m(1/4)R2w2

w1 = (1/4)w2
w2 = 4w1

ÖRNEK 10.2

Şekildeki sistem başlangıçta hareketsiz olup Dt sani-


ye boyunca zincire uygulanan T=20 N’luk kuvvetin
etkisi altında 150 dev/dak açısal hıza ulaşmaktadır.
Dt’yi belirleyiniz. Sürtünmeyi ve 3 kg’lık dört adet
küre dışındaki kütleleri ihmal ediniz.

ÇÖZÜM :


v 
v t2   
∫t1
ΣM z dt = H z 2 − H z1

  2π  
 (
20 0.1) t = (4 ) (
3) (
0.4 )150   0. 4 
T r  
mküre rlink  60  
 makara 
v
vküre


v t = 15.08 s
z
ÖRNEK 10.3
Şekildeki sarkaç iki adet 3.2 kg’lık topaklanmış kütle ve
hafif rijit çubuktan oluşmaktadır. 300 m/s hızla giden 50 g
kütleli mermi alttaki kütleye çarpıp saplandığında sarkaç
saat yönünde w=6 rad/s hızla salınım hareketi yapmaktadır.
Çarpmadan hemen sonra sarkacın w’ açısal hızı ile sarkacın
maksimum q sapmasını hesaplayınız.

ÇÖZÜM :
Açısal momentumun korunumu ilkesine dayanarak ilk momen-
tumu son durumdaki momentuma eşitleyeceğiz.

1 v 1´
2
q
t      v2´ q
∫0
M O dt = H O2 − H O1 = 0 , H O1 = H O2 O

q v2

MO = 0 ⇒ (r × mv )1 = (r × mv )2 w
2 q
1
v1

0.050(300)(0.4 cos 20) − 3.2(0.2)2 (6) − 3.2(0.4)2 (6) = (0.050 + 3.2)(0.4)2 w ′ + 3.2(0.2)2 w ′

w ′ = 2.77 rad / s ( sity )

Çarpışmadan sonra enerji yaklaşımı; T1 + V g1 = T2 + V g 2


(referans O’da)

1
(0.05 + 3.2)(0.4 ⋅ 2.77 )2 + 1 (3.2)(0.2 ⋅ 2.77 )2 + (3.2)(0.2)(9.81) − (3.2 + 0.05)(0.4)(9.81)
2 2
= 0 + (3.2 )(0.2)(9.81) cos q − (3.2 + 0.05)(0.4 )(9.81) cos q
q = 52.1o
Bölüm Sonu Soruları

1- Katı bir silindir sürtünmesiz bir mil etrafında serbestçe dönebilecek şekildedir.
Dış yarıçapı R1= 1.5m olan silindir etrafına sarılı bir ipe, sağa yönelmiş F1=15N
kuvveti uygulanıyor. R2= 1m yarıçapı üzerindeki ipe de aşağı yönelmiş F2=20 N
kuvveti uygulanıyor.
a) O’dan geçen z eksenine göre silindire etki eden net tork nedir?
b) olduğunda net tork nedir?
2- Şekilde görüldüğü gibi bir bovling topu 25 devir/dak’lık açısal hızla döndüğüne göre açısal
momentumunun büyüklüğünü bulunuz

3- Şeklide görüldüğü gibi, m1 kütleli bir kütle ile m2 kütleli bir blok R yarıçaplı
bir makaradan geçen ince bir iple birbirine bağlıdır. Makaranın, kendi eksenine
göre eylemsizlik momenti I dır. Blok, sürtünmesiz yatay bir düzlem üzerinde
kaymaktadır. Açısal momentum ve tork kavramlarını kullanarak, bu iki cismin
doğrusal ivmesini m1, m2 ve R cinsinden bulunuz.

4- Dairesel disk şeklinde yatay bir tabla, sürtünmesiz, düşey bir mil etrafında yatay
düzlemde dönmektedir. Tabla M=80kg’lık bir kütleye ve R=1.5m lik yarıçapa sahiptir.
m=50 kg kütleli bir öğrenci, dönen tablanın kenarından merkezine doğru yavaşça
yürümektedir. Öğrenci, dönen tablanın kenarındayken sistemin açısal hızı 4rad/s ise,
öğrencinin merkezden r=0.5 m uzaklıktaki bir noktaya ulaştığındaki açısal hızı ne
olur?

5- Kütlesi 400gr olan bir kuş 10m/s yatay olarak dikkatsizce uçmaktadır. Bir çubuğa
tepesinden 25 cm yukarısında çarpıyor. Düzgün çubuk 0.75 m uzunluğunda 1.50
kg ağırlığında ve tavanından menteşelidir. Çarpışma kuşu sersemletiyor ve yere
düşüyor. Kuşun çarpışmadan hemen sonraki çubuğun sürati ne olur?

6- Uzunluğu L, kütlesi M olan düzgün bir çubuk, şelildeki gibi bir ucu etrafında sür-
tünmesiz dönebilecek durumdadır. Çubuk yatay durumda iken serbest bırakılıyor.
Çubuğun ilk açısal ivmesi ve açısal momentumunu L,M ve g cinsinden bulunuz.

Kütlesi M=2kg ve yarıçapı R=30cm olan bir silindir merkezinden geçen, etrafında serbest-
7-
çe dönebilen bir eksen ile duvara tutulmuştur. Silindir etrafına sarılı ipin çekilmesi sonucu
döndürülüyor. İpi çeken T=6N kuvveti sabit olduğuna göre silindir 2 tam tur attığında
sahip olduğu açısal momentumu bulunuz.

8- Yandaki şekilde verilen M=4kg kütleli silindirin dış yarıçapı R=15cm ve iç yarıçapı
r=10cm dir. Kütlesi m=2kg olan bircisim silindire sarılı bir ipe bağlı olarak serbest
bırakılıyor. Aşağı inen cismin kinetik enerjisi 49 Joule ulaştığında silindirin açısal mo-
mentumunu bulunuz.

9-
Tabure ilee birlikte 4 rad/s açısal hızla dönen adam ile taburenin toplam eylemsizlik mo-
menti I=160kg.m2 dir. Adam kollarını kapatınca toplam eylemsizlik momenti %75 ine
azaldığına göre bu işi yapması için kaç Joule’ luk enerji harcamalıdır?
11. BÖLÜM

Esneklik ve Denge Şartları

11.1 Statik Denge


11.2 Denge Halindeki Cismi İncelemek
11.3 Kuvvet Çifti
11.4 Maddelerin Mekanik Özellikleri
11.5 Esneklik ve Esnek Maddeler
11.6 Esneklik Mödülü/ Young Mödülü
Bölüm Sonu Soruları
Statik Denge

Çevremizde gördüğümüz uzun süre konumu ve dengesi değişme-


yen binalar, küprüler ve daha bir çok yapıların mühendislik açıdan
incelenmesi statik denge ve esneklik konusuyla ilgilidir. Masa-
nın üzerindeki bir cismin yıllarca aynı durumda kalması ve masa
ayaklarının bu ağırlığı taşıması statik denge ve esnekliğe güzel bir
örnektir.

Fizikte statik denge, hareketsiz olma veya durgunluk halini


koruma durumu diye de geçer. Denge durumundaki cismin
ağılığı ve ağırlığı taşıyan sistemin tepki kuvvatleri birbirine
eşit olmalıdır ki cisim hareketsiz kalabilsin. Her denge şartın-
daki bu önemli prensip düşünülmelidir. Bu tür durumlarda
durgun haldeki cismin ağırlığını veya ağırlık kuvvetinin
bileşenlerini karşılayacak tepki kuvvetleri olmalıdır. Denge
durumunda denklemler kurulurken hem kuvvetlerin eşitliği
hem de momentler eşitliği baz alınır. Yani düşeydeki toplam
kuvvetler birbirlerini, yataydaki kuvvetler de birbirlerini
karşılamalıdırlar. Aynı zamanda herhangi bir dönme noktasına göre bu kuvvetlerin döndürme etkileri de
birbirlerini karşılamak zorunda. Bunu matematiksel olarak ifade edersek bütün dikey yöndeki kuvvetler
toplamı sıfır, yatay yöndeki kuvvetler toplamı sıfır ve dönme noktasına göre bu kuvvetlerin döndürme etki-
leri de sıfır olmak zorundadır.
Denge Halindeki Cisimler

Statik dengedeki cisimler incelenirken şu adımlar dikkate alınır;

1. İncelenen cismin iki boyutlu kabataslak görüntüsü çizilir ve uygun koordinat sistemi belirlenir.
2. Cisim üzerinde etkiyen tüm kuvvetler belirlenir ve etkiledikleri noktalar tespit edilir.
*Bu iki adımdaki çizim ile aslında Serbest cisim diyagramı çizilir. (Cisim, üzerindeki kuvvetler ve bu kuvvet-
lerin etkime noktaları ile oluşturulan şekle serbest cisim dyagramı denir.)
3. Her kuvvetin doğrultusu ve yönü tekrar kontrol edilir.
4. Koordinat sistemindeki eksenlere göre kuvvetlerin bileşenleri bulunur.
5. Uygun koordinat sistemine göre aynı doğrultudaki kuvvetler arasında işlemler yapılır ve çözülen eştlikle
bilinmeyen kuvvetler bulunur.
6. Denge şartlarına göre ugun bir dönme noktası seçilir.
7. Eksenlere göre bileşenlerine ayrılan kuvvetlerin dönme noktasına göre momentleri alınır. Moment alınınca
saat yönünün tersine göre dönme pozitif(+), saat yönündeki dönme ise negatif(-) işaretli kabul edilir.
8. Elde edilen kuvvet denge (SFx = 0 , SFy = 0) ve moment denge (ST = 0) denklemleriyle bilinmeyenler
bulunur edilir. Bulunan kuvvet eksi(-) işaretli çıkıyorsa yönü yanlış alınmış demektir. Böylece kuvvet ters
çevrilir.

Kuvvet Çifti
Kuvvetler bir cisim üzerine etkiyince cisimde ya şekil değişikliğine sebep olurlar ya da cismi hareket ettirerek
enerji kazanmasını sağlarlar. Cisim denge halindeyse hareket etmez ve esneme özelliğine göre şekil değiştirir.
Eğer kuvvetler birbirlerini dengeleyemiyorsa cisim net kuvvetin etkisine oranla ivmeli hareket eder. Kuvvet
çifti denince eşit şiddette ve zıt yönlü iki kuvvetin bir cisim üzerindeki etkisi anlaşılmalıdır. Yani kuvvetlerin
büyüklükleri aynı olmalı ve aralarındaki açı 1800 olmalıdır. Eğer kuvetler aynı noktaya etkiyorsa cisim den-
gede kalır ne öteleme ne de yuvarlama hareketi yapar. Ama kuvvetler farklı noktalara etki ederse aralarındaki
mesafeye oranla dönme hareketi yaparlar (Dödürmeyi sağlayan moment zıt yönlü kuvvetler arasındaki me-
safeyle doğru orantılıdır.) Dikkat ederseniz öteleme hareketinden bahsetmedik çünkü kuvvetler miktarca eşit
büyüklükte ve zıt yönlü olduğundan bu doğrultu boyunca net kuvvet sıfır olur ve öteleme hareketi yapmazlar.
Ama aralarında mesafe olduğundan tam ortalarındaki noktaya göre dönme hareketi yaparlar.
ÖRNEK 11.1
500 N luk F kuvvetini A noktasına şekilde görüldüğü gibi
etkimektdir. Bu kuvvetin AC çubuğu doğrultusundaki
bileşenin şiddeti 300 N ve yönü A dan C ye doğru olduğuna
göre kuvvetin AB doğrultusundaki bileşenin şiddeti ve yatay
düzlem ile yaptığı θ açısı ne olur?

ÇÖZÜM : Anoktasında cosinus teoremi kullanılarak ;


Sistemin serbest cisim diyagramı (SCD)
F2 = FB2 +FC2 + 2.FB.FC.cos(60o-q+q+45o)

5002 = FB2 +3002 +2.FB.300.cos(105o)

FB = 485,12 N

q açısını bulmak için F kuvvetinin yatay bileşeni AB


ve AC çubuklarındaki kuvvetlerin yatay bileşenleri
toplamına eşitlemeliyiz.

F.cos(q) = FB.cos(60o) + FC.cos(45o)


500.cos(q) = 485,12. cos(60o) + 300.cos(45o)
500.cos(q) = 454,692 N
q = arccos(454,692/500)

q = 24,580
ÖRNEK 11.2
Şekildeki gibi 10 kN luk yük taşıyan, 6 kN
ağırlığındaki uniform kirişe bağlı kablodaki
ve A pimindeki kuvvetleri bulunuz.
ÇÖZÜM : SCD

SFx=0 SMA=0
Ax - T.cos(250) =0 T.cos(250).(0,25)+T.sin(250).(5-0,12) - 10.(5-1,5-0,12) - 6.(2,5-0,12) =0

SFy=0 Ax=19,04 kN Ay= 7,13 kN T= 21,0 kN


Ay + T.sin(250)=0 A2 = Ax2 + Ay2 A=20,3 kN
Maddelerin Mekanik Özellikleri

Yapıların tasarımında kullanılan malzemelerin


mekanik özellikleri çok önemlidir. Malzeme-
lerin mekanik yükler altındaki davranışları-
na “Mekanik özellikler” adı verilir. Mekanik
özellikler esas olarak atomlar arası bağ kuv-
vetlerinden kaynaklanır. Ancak bunun yanın-
da malzemenin iç yapısının (Mikroyapı) da
etkisi vardır. Bu sayede iç yapıyı değiştirerek
aynı malzemede farklı mekanik özellikler elde
etmek mümkün hale gelir. Metallerin mekanik
özellikleri çeşitli yükleme şartlarında, çeşitli
deney parçaları ile incelenir.
Başlıca mekanik özellikler:
– Çekme/basma (tensile /compression)
– Sertlik (hardness)
– Darbe (impact)
– Kırılma (fracture)
– Yorulma (fatigue)
– Sürünme (creep)

Uygulanan gerilmeler (yükleme) altında; katılarda önce elastik (geri dönüşümlü) deformasyon daha
sonra da ani gevrek kırılma, ya da bir plastik (kalıcı) deformasyonu takip eden sünek kırılma oluşur.
Seramikler ve camlar gevrek kırılma, metaller ve polimelerler sünek kırılma davranışı gösterirler.
Beton gevrektir ve esnemeden kırılır, onu sünek hale getirmek için çelik kullanılır. Çelik sünektir,
kırılmadan önce deforme olur. Bu durum gerilme/uzama eğrilerinde gözlemlenir. Süneklik kırılmaya
kadar olan plastik deformasyon bölgesini ifade eder.
Esneklik ve Esnek Maddeler
Şimdiye kadar denge şartlarında katı cisimlerin şekilleri bozulmadan dengede kalacaklarını varsaydık.
Gerçek hayatta ise yükleme altındaki her cismin az da olsa şekil değişikliğine (deformasyon) uğraya-
cağını bilmeliyiz. Deformasyonlar elastik ve plastik deformasyon diye ikiye ayrılır. Eğer yükleme kal-
dırılınca cisim eski şeklini koruyup aynı boyutta kalıyorsa cisim esnektir ve bu duruma elastik defor-
masyon denir. Diğer yandan eğer yükleme kaldırıldıktan sonra madde eski boyutuna gelmiyorsa yani
maddenin iç yapısında bir değişiklik olmuşsa bu bozunma kalıcıdır ve buna plastik deformasyon denir.

elastik deformasyon plastik deformasyon

Maddeler üzerinde yükleme işlemi çekme, bastırma, burkma ve burulma şeklinde farklı olabilir. Bizler
burda genel olarak çekme ve bastırmayla ilgileneceğiz. Fiziğin mukavement ve statik alt bölümleri bu tür
uygulamalarla ilgilenir. Genel olarak L uzunluğuna ve A kesit alanına sahip bir cismin p yükü altında esnek-
lik davranışını incelersek kriterlerimiz birim alana düsen kuvvet ve uzunluk basına birim uzama değerleri
olacaktır. İşlemlerde kolaylık sağlaması için uygulanan kuvveti değil de birim alana düşen dik kuvveti alma-
mız işimizi kolaylaştıracaktır. Birim alana düşen dik kuvvete gerilmedir. Buna aynı zamanda stress de denir
ve birimi basıncın birimi ile aynıdır(N/m2). Uzama miktarı ilk boyu ile doğru orantılı olduğundan uzanımı
bulmak için toplam uzama miktarını ilk boy ile oranlayacağızki birim uzunluk başına uzama miktrarını
elde edelim. Birim uzama miktarına ise gerinim denir ki bu aynı zamanda maddenin esneklik oranıdır. Ge-
rinimi bulurken uzunluğu uzunluğa oranladığımızda elde edeceğimiz büyüklük birimsiz gibi düşünülebilir.
Oysaki işlemlerde gerinimin birimi boyutsuz değil de mm/mm alınır.

gerinim

gerilme
Esneklik Mödülü/ Young Mödülü

Malzemeler farklı özellikteki atom ve bileşiklerle oluştuğundan herbirinin yük altında esneme oranları
farklılık gösterir. Esneklik ya da Young modölü dediğimiz bu özellik malzemenin cinsine bağlı olarak deği-
şen bir katsayıdır. Esneklik (Young) modölünü E ile gösteririz. Bu katsayı birim gerinim başına düşen birim
gerilmedir.
Laboratuvarlarda yapılan malzemenin mukavemet testleri sonucu deneysel veriler elde edilir. Elde edilen
çekme deneyi grafiğinin doğrusal arttığı bölge bu malzeme hakkında önemli bir bilgi verir. Bu bölgedeki gra-
fiğin eğimi malzemeye özel Young modölüne eşittir. Esneklik modölü maddenin yükleme altındaki esnek-
liğinin bir ölçüsüdür. Bu katsayı sadece elastik deformasyonunun olduğu bölgede geçelidir. Madde akmaya
başladığında plastik deformasyon olacağından esneme grafiği lineer özellik taşımaz ve sabit bir gerilme-geri-
nim eğrisi elde edilmez.

*Gelime-gerinim grafiğindeki çarpı işareti malzemenin boyun verdiği ve kopmaya başladığı anı temsil eder.
Maddenin üç farklı durumu için farklı esneklik sabitleri tanımlanır.
1) Malzemenin uzunluğundaki değişmeye karşı gösterdiği direnç için esneklik (Young) modölü,
2) Malzemenin atomik düzeydeki düzlemlerin kaymaya karşı gösterdiği dirence kesme (Makaslama) modölü,
3) Katı veya sıvılıarın hacimlerindeki değişikliğe karşı gösterdiği dirence ise hacim (Bulk ) modölü denir.
Bölüm Sonu Soruları
1- Şekildeki uçağın ağırlık merkezinin ön ve arka tekerleklere göre
konumu verilmiştir. Statik denge şartı denklemlerini kullanarak W/R
oranını bulunuz.

2- Şekilde verilen yükler altındaki kirişin A ve B destek noktalarındaki


tepki kuvvetlerini bulunuz.

A noktasında duvara mesnetlendirilmiş kirişe uygulanan


kuvvetlerin şiddeti şekilde görüldüğü gibidir. Bu kuvvetlerin
3- çubukta oluşturduğu kesme kuvveti- moment diyagramını
çizerek A noktasındaki tepki momentini bulunuz.

4- 150N’luk dik kuvvet O noktasından bağlanmış şafta A noktasından etkimektedir.


Buna göre;
a) Bu kuvvetin O noktasına göre momentini,
b) Aynı momenti oluşturacak, A noktasına uygulanacak yatay kuvveti,
c) Aynı momenti oluşturacak, A noktasına uygulanacak minnimum kuvveti ,
d) Aynı momenti oluşturacak 240N’luk düşey kuvvetin yerini bulunuz.

5- Aşağıdaki ifadelerin tanımlarını yaparak karşılık gelen förmülleri elde


ediniz.
-mühendislik gerilmesi, s
-mühendislik şekildeğiştirmesi, e

6-
Farklı yükleme altındaki cisimlerin deformasyona uğramaları
şekilde gösterilmiştir. Bu deformasyonları veren formölleri elde
ederek karşılık gelen gerilme ifadelerini açıklayınız. (e, q, f, s, t)

7- Bir çelik numunesinin çekme yükü altındaki gerilim-uzama grafiği


verilmiştir. E,X,Y,K, a ve b noktalarına karşılık gelen ifadeleri birer
cümle ile açıklayınız.

8- Şekilde verilen x ekseni boyunca A noktasından mesnetli pas-


lanmaz çelik çubuğun boyu L = 60 cm ve kesit alanı 10x5 cm2 dir.
T=150kN luk kuvvet ile ucundan çekilirse son durumdaki boyu
kaç m olur?

9- Şekilde görülen betonarme kolonda altı adet 20mm çaplı BÇIII Çeliği
bulunmaktadır. Malzeme özellikleri;
s bgüv =20MPa, s çgüv =200MPa, Eç=200GPa, Eb =20GPa
olarak verildiğine göre bu kolonun güvenlikle taşıyabileceği P basınç yükünü
hesaplayınız.
12. BÖLÜM

Akışkan Mekaniği

12.1 Akışkan Madde


12.2 Akışkan Basıncı
12.3 Cismin Akışkanda Yüzmesi
12.4 Akışkan Hareketi
12.5 Viskozite ve Türbülans
12.6 Bernoulli İlkesi
Bölüm Sonu Soruları
Akışkan Madde

Maddeleri sınıflandırırken katı, sıvı ve gaz olarak ayrıldığını bilmeyenimiz


yoktur. Ama akışkan denilince akla sadece sıvı fazlı maddeler geliyorsa bu
yanlış bir genelleme olur. Çünkü akışkan madde kütlesi değişmediği halde
şekli belki de hacmi değişebilen maddelerdir. Genelde akışkan olarak sıvı, gaz
ve plazma hali ele alınır. Ama burda ilgi odağımız sıvı ve gaz üzerinde olacak-
tır. Akışkanlar mekaniği konusu sıvı ve gazların hareket, kuvvet, hız ve basınç
gibi değerlerin akışkanın davranışına etkisini açıklar. Katı maddelerin belirli
bir şekli ve hacmi olur ama akışkan olan sıvının belirli hacmi varken belirli bir
şekli yoktur. Dahası, gaz olan bir maddenin ne belirli bir şekli ne de belirli bir
hacmi vardır. Örneğin sıkıştırma durumunda sıvının hacmi ihmal edilebilecek
şekilde azalırken gazların hacmi değişir.

Akıskan: Zayıf bağların etkisiyle molekülleri bir arada tutulan ve rastgele hareket edebilen maddelere akış-
kan madde denir. Akışkanlar mekaniğiyle durgun haldeki bir akışkanın yoğunluğu, statik basıncı ve basınç
kuvvetleri incelenirken hareketli akışkanlarda(dinamik akışkan) hız, dinamik basınç, dinamik vizkosite ve
toplam yük değerlerinin hesaplamaları ele alınır. Akışkanlar mekaniğinde en önemli ve asla unutulmaması
gereken nokta Bernaulli ilkesi ve denklemidir.

Akışkan Basıncı
Maddenin farklı fazlarının farklı dinamik ve mekanik davranışlar sergilediğini akışkanlar ile açıklamak
akılda daha çok kalıcı olacağını umuyoruz. Akışkanlarda basınç değerleri bulunurken maddenin fazına ve re-
ferans bir duruma göre işlem yapılır. Katı maddelerın yaptığı basıncı bulurken katının yüzeye uyguladığı dik
kuvvetin temas yüzeyine bölünmesiyle elde edilen değeri alacağız. Oysaki akışkanların basıncının tespitinde
akışkanın özkütlesi ve hızı önemlilik arzeder.

Sıvılarda basınç, sıvının yüksekliği ile yoğunluğunun çarpımının bi-


rim yüzeye yaptığı dik kuvvet etkisidir. Sınvının birim alan üzerindeki
ağırlığı o alan boyunca yükselen sıvının oluşturduğu ağırlıktır. Yani alan
ile yüksekliğin çarpımı hacim, hacim ile özkütlenin çarpımı kütle ve
son olarak kütle ile ortamın çekim ivmesi ile çarpımı bize ağırlık vere-
cektir. Bu ağırlık birim yüzeye dik etki ettiği için sıvının o noktadaki
statik basıncına eşittir. Sıvı hareketli iken bir yüzeye çarptığı durumda
ise yüzeyde oluşturacağı basınç toplam basınç olur. Bu statik basınç ile
dinamik basıncın toplamıdır.
Gazlarda basınç, gazı oluşturan moleküllerin sahip olacağı kinetik
enerjilerinden dolayı sürekli hareket etmeleri ve birbirleriyle çarpışmaları
sonucu birim yüzeye etkiledikleri kuvettir. Gazların basıncı gazın sıcaklı-
ğına, hacmine dolayısıyla yoğunluğuna bağlıdır.
Sabit hacimli kaplarda gazın basıncı (P) mutlak sıcaklık(T, Kelvin) ve
mölekül sayısı (N, moll) ile doğru orantılı iken değişken hacimli kaplarda
gazın basıncı kabın hacmi (V, Litre) ile ters orantılıdır. Örnek olarak sabit
hacimli bir gaz tankında iç basınç gazın sıcaklığı ile artar. Gazın mutlak
sıcaklığı artarken moleköllerin hareket enerjisi artacağından yüzeye daha
fazla çarparlar. Aynı etki sıcaklık sabit olunca tankın içerisine daha fazla
gaz koyduğumuzda yine birim yüzeye olan çarpma sayısı artar ve hissedi-
len basınç artmış olur. Değişken hacimli kaplara en uygun örnek hareketli sızdırmasız pistonlu kaplardır.
Pistonlu kabın hacmi azaltılınca birim hacimde moleköl sıklığı artar ve basınç artmış olur.
Atmosfer basıncı: Atmosfer içindeki gazların ağırlık ve kinetik enerjile-
rinden dolayı yaptığı basıncı ifade ederken 1 atmosferlik (1 atm) basınç
referans alınır.Yer yüzeyinde 1m2 lik alana etkiyen ve yaklaşık 104 kg
kütleye denk gelen gaz basıncına atmorfer asıncı denir. Torichelli deneyi
denilen yandaki düzenekte deniz seviyesinde 0 oC de 1 atm basınç boru
içindeki yoğunluğu 13.6 g/cm3 olan cıvayı(Hg) 76cm yükseltir. Bu nedenle
biz işlemlerimizde 1atm= 76cm-Hg alacağız.Bu basınç atmosfer içindeki
gazların durumuna göre değişen bir değerdir. Hava sıcaklığının ve rakı-
mın yüksek olduğu yerlerde basınç düşük olur.

N T N T

Gazların basıncından bahsederken bir diğer önemli nokta da gazın hareketli


olup olmadığıdır. Basınç ölçer cihazı durgun haldeki gazın basıncını ölçünce
buna ortam basıncı denir. Bu basınç atmosfer basıncı olabilirken mutfak tü-
pündeki gaz basıncı da olabilmektedir. Uçakların hızlarının belirlenmesinde
kullanılan Pitot cihazı durgun haldeki statik basınç ile burun kısmında cihaza
çarpan hareketli havanın basınçları fakını alarak seyir hızını tespit eder. Burada
hareketli havanın basıncının çarpma bölgesinde durgun haldeki gaz basıncın-
dan fazla olan kısıma dinamik basınç denir.
ÖRNEK 12.1
Yandaki şekilde yarıçapı r=3 cm ve uzunluğu L=125 cm
olan pistonlu silindirin içerisinde atmosfer basıncında su
vardır. Piston 1w,2cm ilerleyip suyu sıkıştırdığında
a) Yeni durumdaki basıncı ve yoğunluğu,
b) Yapılan işi bulnuz.

ÇÖZÜM :

ÖRNEK 12.2

Silindirik depo 50 mm yüksekliğinde su içermektedir. İçteki küçük


silindir depo h yüksekliğinde özgül ağırlığı 0.8 g/cm3 olan gaz yağı
içermektedir. Pa`nın ölçülen basıncı ve gaz yağının ölçülen yüksek-
liği nedir. (Gaz yağının deponun üstüne çıkmasının önlendiği kabul
edilecektir.) ( Pb = 13.80 kPa; Pc = 13.82 kPa )

ÇÖZÜM :

H= 50mm g= 9,81 m/s2

Pa = rs . g. H + Pc = - 1000.(9,81). (50.10-3) + 13,82.10-3

Pa = 13,329 kPa

Pa + rs . g. (H-h) + rg . g. h = Pb

h= 1,987. 10-3m ≅ 2mm


Cismin Akışkanda Yüzmesi
Günlük hayatta su ve hava gibi akışkan içinde yüzen cisimlerle sık-
lıkla karşılaşırız. Bu akışkanlar içerisinde yüzen cisimlere bir kal-
dırma kuvveti uygularlar . Uçuşan sinekler, kuşlar, balonlar, uçaklar
havanın bir kaldırma kuvveti olduğunu gösterir. Bunun yanında su
üzerinde yüzen balıklar ve gemiler de suyun bir kaldırma kuvveti
olduğunu gösterir. Sıvı içinde bulunan bir cismin alt ve üst yüzeyle-
rinde basınç farkı oluşur. Sıvının oluşturduğu bu basınç farkından
dolayı cisim sıvı içerisinde yukarıya doğru itilir. Bu itme kuvveti,
sıvının kaldırma kuvveti olup cismin sıvı içinde kapladığı hacim, sı-
vının yoğunluğu ve ortamın çekim ivmesi ile doğru orantılıdır. Aynı
zamanda cisme etkiyen kaldırma kuvveti sıvı içindeki cismin sebep
oluğu yer değişitiren sıvının ağırlığı kadarıdır.

Fkaldırma=Vbatanhacim.dsıvı.gortam

Sıvı içine bırakılan bir cisme aynı anda iki kuvvet etki eder. Bu kuv-
vetler cismin ağırlığı ile kaldırma kuvvetidir. Yüzen ve askıda kala
cisimlerde bu iki kuvvet birbirine eşittir.
Fkaldırma= Gcisim

G = Vcisim.dcisim.g
F = Vbatan.dsıvı.g

Vcisim.dcisim.g = Vbatan.dsıvı.g

Vcisim / Vbatan = dsıvı / dcisim

Batan cisimlerde ise cisme etkiyen üç tane kuvvet vardır. Cismin ağırlığı (G) aşağı doğru iken kaldır-
ma kuvveti (F) ve kabın tepki kuvveti (R) yukarı doğru olur. Denge denklemi ; G = F + R olur.
Viskozite
Bir akışkan akmaya çalışırken molekölleri birbiri üzerinde
kaymak ister. Akışkanın vizkozitesi akma kuvvetinin karşılaş-
tığı sürtünme direncinin bir ölçütüdür. Bu sürtünme direnci
moleköller arası bağ kuvvetleri moleküllerin ağırlığı ve şekille-
rine bağlı olur. Balı bir kaba dökmekle sütü dökmek birbirin-
den farklıdır. Bal sütten daha viskoz bir yapıdadır. Bir akışkan
bir yüzey üzerinden geçerek aktığı zaman, yüzeye komşu olan
akış tabakası durgun haldedir. Yüzeyden itibaren birbirini
izleyen tabakaların hızları giderek artar. Yani yüzeye yakın ta-
bakaların hızları düşük yüzeyden uzak olan tabakaların hızları
ise daha yüksektir.
Bir başka yaklaşım da bir akışkanın birbiri üzerinde hareket eden bileşik tabakalardan oluşmuş gibi hissedilir.
Bu yaklaşım bize akışın laminer olduğunu ifade eder. Örneğin; bir deste oyun (iskanbil) kağıdına bir kuv-
vet uygulandığında üsteki kart ilerlerken, bir alttaki kartı öteler en alttaki kart sabit kalır. Sıvıdaki durum da
aynıdır. En alttaki sıvı tabakası sabit kalırken, üstteki tabaka belli bir hız ile kayar, kayarken de bir alttakini
öteler. Buradaki öteleme kuvveti sıvı tabakaları arasındaki yüzeye gösterilen sürtünme kuvveti ile ilişkilidir.
Newtonun vizkoz akış (laminer veya tabakalı akış) kanununa göre sıvı içerisindeki iki komşu tabakanın bir-
birlerine göre hareketine direnç gösteren F sürtünme kuvveti A alanı ve dv/dx hız gradienti ile orantılıdır.

F: Sürtünme kuvveti
µ: Vizkozite katsayısı (Yunancada mu harfi)

Türbülans
Mühendislik uygulamalarında karşılaşılan akışların çoğu türbülanslıdır. Türbülanslı akış, çalkantıların
hakim olduğu karmaşık bir mekanizmadır ve türbülanslı akışın teorisi halen büyük ölçüde çözülememiştir.
Türbülanslı akış, girdap adı verilen dönen akışkan bölgelerinin akış boyunca rastgele ve hızlı çalkantıları ile
gözlenir. Bu değişimler momentum ve enerji geçişini kolaylaştırır. Türbülanslı akışta dönen girdaplar; kütle,
momentum ve enerjiyi akışın diğer bölgelerine moleküler difüzyondan daha hızlı biçimde taşır. Çünkü tür-
bülanslı akışta kütlenin, momentum ve ısının transferi büyük ölçüde artar. Sonuç olarak türbülanslı akış çok
daha yüksek sürtünme, ısı ve kütle geçişi katsayıları anlamına gelir.
Atmosferik türbülans, rüzgarların süratindeki ani ve
düzensiz değişimdir. Havadaki türbülans hareketi bir
dizi hamlelerden ibarettir. Hamle ise, bir dakikadan
daha az bir zamanda rüzgar şiddetindeki ani değişim-
dir.
Türbülansa yol açan faktörler:
a-) Termal faktörler
b-) Sürtünme veya mekanik faktörler.
ÖRNEK 12.3
Şekilde gösterilen Dm = 100 mm çapında
bir mil Dy = 100,25 mm çapında ve L = 300
mm genişliğinde bir yatak içerisinde eksenel
doğrultuda 10 kp’lık bir basınçla ancak 0,5
m/s hızla hareket ettirilebiliyor. Buna göre
mil ile yatak arasındaki yağlama yağının
viskozitesini bulunuz.

ÇÖZÜM :

ÖRNEK 12.4
Başlangıçta tamamen su ile dolu olan üstü atmosfere açık 2m
yüksekliğinde geniş bir tank, tabanında bulunan 10 cm çapın-
daki keskin köşeli bir delikten 100 m uzunluğundaki boru ile
atmosfere boşaltılmaktadır. Sistemdeki toplam tersinmez yük
kaybı (hL) 1.5 m olarak belirlendiğine göre tanktan boşalan
suyun ilk hızını (V) belirleyiniz.
V

ÇÖZÜM :
Önce z2=0, z1 = 2m,
P1 = P2 = Patm,
türbin ve pompa olmadığından htürbin=0, hp,f= 0 kabul edilir.

1 ve 2 noktaları arasında Bernoulli eşitliği yazılır. Buna göre,


Akışkan Hareketi

Kapalı bir dolaşım sistemi içinde bir akışkan akışı


oluşabilmesi için basınç farkı olmalıdır. Akışkan,
basıncın büyük olduğu yerden basıncın küçük oldu-
ğu yere doğru akar. Yandaki şekilde görüldüğü gibi
farklı kesit alanlı bir boru içinde sıvı hareket edrken
debi (birim zamanda geçen sıvı miktarı) her yerde
aynıdır. Bu nedenle kesit dar alanlı yerde akış hızı
fazlageniş alanlı yerde hız daha büyüktür.

A1 > A2 , V1 < V2
V1 < V2 , h1 > h2 sonucuna ulaşılır.

Bernoulli İlkesi

Genel olarak akışkan bir maddenein hareke-


ti sırasında sahip olduğu enerjinin korunumu
ilkesine dayanır. Kararlı durumda bulunan bir
sıvının içinde bir akış hattını ele alalım. Sıvının
bu bölgedeki enerjisi, yerçekiminden doğan
potansiyel enerji, kendi üzerinde bulunan sıvının
aşağı itmesinden doğan potansiyel enerji ve sıvı-
nın hızından dolayı sahip olduğu kinetik enerji
toplamlarından meydana gelmektedir. Bernoulli
ilkesine göre, akım çizgilerine sahip turbulanslı ve
viskoz olmayan bir akımda bu enerjilerin topla-
mı değişmez. Bu ilke ilk defa 1700’lerde İsviçreli Matematikçi Daniel Bernoulli tarafından ortaya konulmuş-
tur. Bernoulli ilkesi günlük hayattaki birçok olayı açıklamaktadır. Bernoulli ilkesi, akıskanın sürati artarsa
akıskanın basıncı azalır, şeklindedir. Şekildeki uçak kanadında kanadın şeklinden dolayı üst kısmında hava
alt kısmına göre daha süratli akar. Kanadın üstünde basınç düşer. Bu fark yukarı doğru bir kuvvet oluşturur.

Hızlı hareket halindeki bir trenden veya herhangi bir araçtan atlayan bir kimsenin genellikle o aracın altına
düşmesi bu sebeptendir. Oda içine doğru açılan bir pencerenin bir fırtınada kapanmaya çalışması ve camla-
rın dışa doğru patlaması Bernoulli ilkesinin bir göstergesidir.
Bölüm Sonu Soruları
1- Atmosfer basıncının 76 cm-Hg olduğu bir ortamda içi su dolu bir tankın çeperine
yapılan basınç ölçülecektir. A noktasında yarıçapı r= 3cm olan bir deliğe basınç-ölçer
cihazı konulmuştur. A noktası su yuzeyinden h= 45cm derinlikte olduğuna göre ;
(dsu= 1 g/cm3 , dcıva= 13.6 g/cm3 , g= 9.81 m/s2 )
a) A noktasındaki sıvı basıncı kaç Pa’ dır?
b) A noktasındaki toplam basınç kaç Pa’ dır?
c) Basınç-ölçer cihazının yuzeyinde hissedilen sıvı basınç kuvveti kaç N’ dur?
Çapı D= 10cm olan bir borudan akan suyun hızı V= 7m/s dir. Suyun sıcaklığı
2-
160C ve bu sıcaklıktaki viskozitesi m= 0.38 Ns/m2 , yoğunluğu r=0.998g/cm3
olduğuna göre;
a) Suyun akış debisini bulunuz.
b) Reynold sayısını hesaplayarak akışın laminer ya da türbülanslı olduğuna
karar veriniz. (Re = r.V.D/ m )

3- Taban alanı S=200cm2 olan yandaki deney kabına oda sıcaklı-


ğındaki 2 litre suya bir yumurta bırakılınca batarak kabın tabanına
değiyor ve su seviyesi kapta 4 cm yükseliyor. Suya 150 gram sofra
tuzu konulunca yumurta yüzmeye başlıyor ve su seviyesi 1 cm aşağı
iniyor. ( Toda= 250 , dsu= 1 g/cm3 , g= 9.81 m/s2)
a) Yumurtanın hacmini bulunuz
b) Yumurta ile kabın tabanı arasındaki tepki kuvveti kaç N’dur?
c) Tuzlu suyun yoğunluğunu bulunuz.

4- Silindir şeklindeki bir tank 5Mpa basınçlı hava ile doludur.


Bu tank en fazla 11 Mpa basınca dayanacak çelik malzeme ile
yapılmıştır. Tankı test etmek için içerisine pompa yardımıyla
su ilave edilince tankın iç çapı 2cm genişlediğine göre ilave
edilecek maksimum su miktarı kaç litre olur?

5- Şekildeki su pompası suyu yüksek yere takviye etmek için kullanıl-


maktadır. Bu pompanın giriş kesitindeki su basıncı Pgiriş=0.2 atm ve hızı
Vgiriş=3.4 m/s , çıkış kesitinde Pçıkış=3.2atm ve hızı Vçıkış=4.8m/s olduğuna
göre pompanın göcü kaç Watt’tır?

Şekildeki sistemde kare plaka V=0.3m/s hızla üzerinde e=0.4mm kalınlıkta


6-
yağ tabakası bulunan eğik düzlemde kaymaktadır. Viskozitenin sebep olacağı
sürtünme kuvveti plakanın yavaş kaymasına sebep olacaktır. Plaka ile yağ
tabakası arasında "t=F/A” ve "t= m.DV/e” şeklinde bir ilişki olduğu deneysel
olarak belirlenmiştir. Karenin yüzey alanı A=0.81m2 , eğik düzlem açısı a=300
ve yağın viskozitesi m=0.024 N/m2.s olduğuna göre plakanın ağırlığı kaç N’
dur?
7- Giriş çapı d1= 80mm ve çıkış çapı d2= 40mm olan sistemde suyun akış
debisi Q= 6 lt/s ‘ dir. Cıva yoğunluğu dcıva = 13.6g/cm3 olduğna göre
Bernoulli ilkesini kullanarak iki koldaki cıva seviyesi,h, farkını bulunuz.

8- Şekildeki depoda yağunluğu 850kg/m3 olan yağ bulunmakttadır.


Piston basıncı 0,7atm olduğuna göre H cıva yüksekliğini bulunuz.
(dcıva = 13.6g/cm3 )
13. BÖLÜM

Kütlesel Çekim Kanunları

13.1 Çekim İvmesi ve Kütle Çekim Kuvveti


13.2 Kepler Kanunu ve Gezegenlerin Hareketi
13.3 Kütlesel Çekim Alanı
13.4 Kütle Çekim-Potansiyel Enerjisi
13.5 Toplam Mekanik Enerji
13.6 Bağlanma Enerjisi
13.7 Kurtulma Hızı
13.8 Gel-Git, Medcezir
Bölüm Sonu Soruları
Çekim İvmesi ve Kütle Çekim Kuvveti

Evrendeki küçük parçacıklardan büyük gezegenlere kadar kütleye sahip her


varlık genel kütle çekim yasasından etkilenir.
Kütlesi olan her cisim diğer cisimlerle etkileşim halindedir. Kütle çekim
kuvveti bu cisimler arasındaki etkileşim sonucu oluşur ve iki cismi oluştu-
ran madde miktarıyla doğru orantılıdır. Kütlesi büyük cisimler küçük kütleli
olanlara bir çekim kuvveti uyguluyor gibi düşünülebilir ancak kuvvet etki
tepki prensinine göre küçük kütleli cisim tarafından da aynı büyüklükte
büyük cisme etkiyordur. Newton’ un ağaçtan düşen elmayı gözlemlemesi
olayına tekrar değinirsek elmanın daldan düşüp yere doğru hareketi elmaya
yerden etkiyen bir kuvvetin olduğunun göstergesidir. Aynı şekilde elma da
yerküreye eşit büyüklükte kuvvetle çekiyordur ama elma yerkürenin yanında
çok küçük olduğundan kendisi yere çekilir.

Büyük kütleli cisimlerin eylemsizliği fazla olduğu için sabit duruyor küçük cisim kendisine geliyormuş gibi
algılanır. Yerküre üzerindeki cisimlere göre sonsuz büyüklükte kütleye sahiptir ve büyük kütleli cisimleri bu-
yüzden küçük kütleli cisileri kendisine kütle çekim kuvvetiyle çeker. Bu kuvvet yerden cisme etkiyen çekim
ivmesi ile etkilenen cismin kütlesinin çarpımı kadardır(F=m.a). Çekim ivmesi aslında genel kütlesel çekim
kanunlarıyla açıklanır. Çekim ivmesi cisim etrafında bir çekim alanı oluşuturur ve şiddeti cismin merkezin-
den uzaklaştıkça uzakluğın karesiyle ters orantılı azalır.
Evrensel çekim yasasında kütle çekim sabiti denilen bir kat sayı vardır ve G ile gösterilir. Bu katsayı her geze-
gen için ayrı değer alır. Yerküremiz Dünya için G=6,673.10-11 N.m2/s2 alınır.

Büyük kütleli bir gezegen çevreinde olan bir


parçacığa etkiyen kuvvet aslında parçacığın o
gezegenin çekim alanındaki ağırlığına eşittir.
(m.g= G.(M.m)/r2 )
Burdan anlaşılır ki parçacığa göre sonsuz kütle-
deki gezegenin çekim ivmesi aslında gezegenin
kütlesiyle doğru orantılı ve aradaki mesafenin
karesiyle ters orantılıdır.
Kepler Kanunu ve Gezegenlerin Hareketi

Tarih boyunca astronomiye ilgi duyan ilim adamları yıldızların ve gökcisim-


lerinin hareketlerini inceleyip hesaplamalar yapmışlardır. Yılın belirli za-
manlarında belirli açılarla yıldızların ve gezegenlerin konumlarını çok hassas
ölçümlerle belirlemişledir. 12.yüzyılın alimlarinden El-Cezeri'nin çalışmaları
Avrupalı bilim adamlarının astronomi alanındaki fikirlerine ışık tutmuştur.

İsmailé Cıziri (1136-1206)


12.yüzyıl ilim adamlarının çalışmalarının üzerine ortaçağda astronomi ile ilgilenen
Galileo Galilei, Leonardo Davinci, Johannes Kepler gibi önderler günümüze kadar
kullanılacak çalışmalar miras bırakmışlardır.
Ünlü Alman düşünür Kepler uzun süren çalışmaları sonucu aşağıdaki kuramları
elde etmiştir.

Yörüngeler Kanunu: Bütün gezegenler sistemine bağlı oldukları yıldız etrafında


eliptik bir yörüngede hareket ederler. Dolandıkları bu eliptik yörüngenin geometrik
özellikleri daire gibi değil de iki tane odak noktası vardır. Bu odak noktlar elipsin
büyük çapı üzerinde bulunur ki bu aynı zamanda büyük asal eksendir. Büyük çap ile
Johannes Kepler dik kesişen küçük çapa küçük asal eksen de denir.
(1571-1630)

R1: ana eksen yarıçapı


R2: yardımcı eksen yarıçapı
F1,F2: odak noktalar

Elipsin eğrilik oranı e =R1/R2

0<= e <=1

Alanlar Kanunu: Eliptik yörüngede dolanan gezegenin elipsin mer-


keziyle çizdiği yarıçap eşit zaman aralığında eşit alan tarar.

Eşit alanlar kanunu bize gezegenin güneş etrafında dolanırken açı-


sal momentumunu koruduğunu kanıtlar. Gezegen, yarıçapın büyük
olduğu güneşten uzak konumlarda iken çizgisel hızı yavaş, yarıçapın
küçük güneşe yakın konumlarda iken çizgisel hızı büyük olacak
şekilde hareket eder.

t1 / t2 = S1/ S2

t1=t2 ise S1=S2 olur.


Periyotlar Kanunu: Aynı yıldız etrafında dolanan gezegenlerin yörüngelerinin ortalama yarıçapı ile dolan-
ma süreleri arasında bir ilişki vardır. Bu ilişki şöyle ifade edilir: yarıçaplarının küpünün dolanma periyotları-
nın karesine oranı aynı sistemdeki farklı yörüngelerde dolanan tüm gezegenler için sabittir. Bu kanun dairesel
yörüngede dolanan Mg kütleli gezegene Ms kütleli güneşin uyguladığı çekim kuvveti ve merkezkaç kuvveti-
nin dengelenmesiyle bulunmuştur. Newton' un F=m.a formölü kullanılarak;

Keplerin bu yasası (R3/T2) aslında aynı güneş sistemi içinde dolanan gezegenler arasında kararlı bir ilişki
olduğunu ifade eder. Yani ortak yıldız etrafındaki gezegenler aynı oranda çekim kuvveti ve çekim potansiyel
etkileşimlerine sahiptirler. Bu etkileşimler gezegenlerin yörüngelerinin ortalama yarıçapları ve dolanım peri-
yotları yıldıza özgü olacak şekilde kararlılığını korur. Bu oran yıldıza özgüdür ve her yıldız için ayrı bir değer
alır.
Kütlesel Çekim Alanı
Kütlesel çekim alanı gezegenlerin çevresindeki cisimlere gönderdiği, cisimleri kendine çeken kuvveti oluştu-
ran g çekim ivmesiyle oluşur. Gezegen, etrafına her tarafa etkiyecek şekilde bu ivmenin değeri kadar gezegen
çekim alanı oluşturur. Alanın şiddeti çekim ivmesine bağlıdır ve yeryüzünden uzaklaştıkça şiddeti azalır. M
kütleli bir gezegen kendisinden r kadar uzaklıktaki m kütleli bir cisme F=G.(M.m/r2) kuvveti uygular.
Cisme etkiyen kuvvet F=m.a=m.g den g=G.M/r2 olur. Bu durum bize çekim alanı veya çekim şiddeti geze-
genin kütlesi(M) ile doğru, mesaffenin karesi (r2) ile ters orantılıdır. Bu yüzden denilir ki bir cismin Dünya
daki ağırlığı Ay daki ağırlığının 6 katıdır. Sebebi Dünya nın kütlesinin Ay ın kütlesinin 6 katı olmasıdır.
Kütle Çekim-Potansiyel Enerjisi

Çekim potansiyel enerjisi cismin yeryüzeyinden olan yüksekliğinden


dolayı sahip olduğu potansiyel enerjidir. Bulunduğu gezegenden kendisine
etkiyen çekim kuvveti mg ile yüzeyden h kadar çekilince yapılan iş E= F.Dx
den U=m.g.h olur. h'ı r kadar büyük alırsak ve g yerine GM.m/r2 yazarsak
U= - (GM)/r olur. Formülün önüne eksi işareti koyulmasının sebebi cisim
yeryüzünden uzaklaştıkça formül değerce sıfıra yaklaşır ve potansiyel enerjisi
büyür. Gerçekte de yerden yükselen bir cismin potansiyel enerjisinin artığının
kanıtıdır.

r1 r2
Toplam Mekanik Enerji
Etrafında dolandığı gezegene göre çok küçük kütleye sahip bir uydu (M>>m) yörüngesinde V hızıyla
dolanınca yüksekliğinden dolayı potansiyel ve hareketinden dolayı kinetik enerjiye sahip olur. Uydunun
toplam mekanik enerjisi kinetik ve potansiyel enerjileri toplamıdır.

* Uydunun toplam enerjisi açısal momentum prensibiyle her zaman korunur. Uydu etrafında dolandığı geze-
gene yakın mesafede iken kinetik enerjisi büyük potansiyel enerjisi küçüktür. Uydu gezegenden uzaklaştıkça
potansiyel enerjisi artarken kinetik enerjisi azalır ama toplam mekanik enerji korunur. Uydunun gezegene
yakın mesafelerde hızlı hareket etmesinin sebebi hem potansiyel enerjinin çoğunun kinetiğe dönüşmüş ol-
ması hem de uydu gezegenin çekimine karşı yörüngede kalması için merkezkaç kuvvetinin (F=mV2/r) büyük
olması gerektiğidir.

Bağlanma Enerjisi
Yeryüzünden yörüngelere yerleştirilmek amacıyla fırlatılan uydular yerin çekim kuvvetine karşı koyabil-
mek için bir enerjiye sahip olmaları gerekir. Başlangıçta sadece kinetik enerjiye sahip bu uydular yörüngeye
gidene kadar bu enerjiyi potansiyele dönüştürerek en son radyal (yarıçap) yönünde sadece teğetsel hıza sahip
olurlar. Uyduları yeryüzünden yörüngeye oturtmak için gerekli enerjiye baglanma enerjisi denir.

Fmerkezkaç = Fçekim

(m.V2)/r = G.(M.m)/r2

uydunun kinetik enerjisi Ek = 1/2 m.V2 = 1/2G.M.m/r2 olur.

Cismin toplam enerjisini sıfırlayan enerji bağlanma enerjisi


olduğundan
Ebagkanma= - Etoplam

Etop = Ek + Ep = G.M.m/2r - G.M.m/r = -G.M.m/2r

Ebaglanma = G.M.m/2r
Kurtulma Hızı

Yeryüzündeki bir cismi gezegenin çekim alanından çıkarmak


için sonsuza doğru götürmemiz gerekir. Hatırlarsak isim
sonsuzda iken sahip olduğu toplam enerjisi mekanik enerji
oluyordu ve bu enreji kinetik ile potansiyel enerjilerin toplamı
kadardı.

Etop= - 0.5(GMm/r)

Cisme bu enerjiyi kazandırmak için yeryüzünden büyük bir


hızla fırlatılması lazım. Cisim fırlatılınca sahip olduğu kinetik
enerjiyi kurtulma enerisine eşitlersek;
1/2mV2= 1/2 GMm/r den

olur.

ÖRNEK 13.1
ÖRNEK 13.2
Bir gezegen etrafında dolanması için fırlatılmış m kutleli bir yer uydusunu 2R yarıçaplı bir dairesel
yörüngeden 5R yarıçaplı bir yörüngeye taşınıyor. Bu taşıma esnasında yapılması gerek işi hesaplayınız.

ÇÖZÜM :
Uydunun taşınması için gereken işi hesaplarken enerji metodunu
kullanacağız. Uydunun son durumdaki toplam enerjisinden ilk
durumdaki toplam enerjiyi çıkarınca bulacağımız enerji fakı
yapılan iş olacaktır.

W = DE = Eson - Eilk

Eilk = - G.M.m/2r1 = - GM.m/2(2r)


Eson = -G.M.m/2r2 = - GM.m/2(5r)
DE = Eson - Eilk = - GM.m/10r -(- GM.m/4r)

W = DE= (3GMm)/20r olarak bulunur.

ÖRNEK 13.3

ÖRNEK 13.4
Gel-Git, Medcezir Olayı
Ay, Dünya etrefında dolanırken kendisi ile Dünya arasındaki çekim kuvvetinden dolayı
bir etkileşim olur. Ay'ın yörüngesinin eliptik olması ve Dünya'nın kendi ekseni etrafın-
daki dönüşünün sebep olduğu bu etkileşimden dolayı günde iki defa okyanus sularında
yükselmeler- alçalmalar gözlemlenir.
Ay’ın farklı uzaklıklarında, yerin çekim şiddetindeki farklılığından dolayı Dünya' nın o andaki Ay'a bakan
yüzünde ve zıt yüzünde şişmeler (tidal bölgeler) gözlemlenmesine gel-git olayı ya da medcezir denir.
Üç farklı gel-git türü vardır;
Günlük gelgit; genelde tropik bölgelerde görülen, bir Ay gününde bir yükselme ve bir alçalma şeklinde olu-
şan gelgit türüdür.
Yarı-günlük gelgit; genellikle Kuzey Avrupa kıyılarında görülen, iki yükselme ve iki alçalma şeklinde oluşan
gelgit olayıdır.
Karışık gelgit; genellikle Kuzey Amerika kıyıları ve Avustralya kıyılarında görülen, iki yükselme ve iki alçal-
ma ile bütünleşik şekilde oluşan gelgit olayına verilen isimdir.
Galileo 1632'de yayımladığı "Gelgit Üzerine Diyalog" (Dialogue Concerning the Two Chief World Sys-
tems-Dialogue on the Tides) kitabında gelgit için "Denizdeki suların, Dünya'nın Güneş etrafında dönmesi
sonucu savrulmasıdır." diyerek yanılgıya düşmüştür. Gelgitin kütleçekim kuvveti sonucu oluştuğu 1687'de
Newton'ın Principia eserinde açıklanmıştır. 18.yüzyılda su yüksekliğini hesaplayacak tablolar geliştirilmiştir.
Günümüzde ise su yüksekliği, akıntılar, gelgitin oluşacağı zaman bilgisayarlarla hesaplanmaktadır.

Gel Gel-git, Güneş’in çekim kuvvetine de bağlıdır. Ancak Güneş Ay'a göre Dünya'dan epey uzakta olduğu
için güneş'in gel-git olayındaki etkisi Ay'ın etkisinin üçte biri kadardır. Ay, Dünya ile Güneş arasındayken gü-
neş etkisi çok; Ay Güneşe göre 90 derece farklı tarafta ise güneş etkisi azdır. Gelgit olayı ilk ve ikinci dördün
evrelerinde en düşük, yeni ay ve dolunay evrelerinde en büyük değeri alır. Bir yerde sular kabarırken Ay, o
yer için gökyüzünün en yüksek noktasındadır. Herhangi bir yerde gelgit olayı her gün aynı saatte olmaz. Bir
önceki günden 50 dakika daha geç oluşur. Buna "Liman Gecikmesi" denir. Bunun nedeni Güneş günü ile Ay
günü arasındaki 50 dakikalık farktır.Bir meridyen Ayın karşısına geldikten sonra dünya dönerek aynı alana
24 saatte gelir, fakat bu sırada Ay dünya çevresindeki yörüngesinde döndüğü için biraz ilerlemişdir, 50 dakika
sonra meridyen yeniden ay karşısına gelir.Böylece Ay günü 24saat 50 dakika olur.

günün ilk 12 saati günün son 12 saati


Gel-git olayı şu önemli olaylara sebep olur:
1- Ay Dünya'ya yaklaşınca okyanu sularını çeker ve kıyılarda sulaçekilir.
2- Sürtünmelerden dolayı Dünya'nın kendi etrafındaki dönüş hızında az da olsa azalma görülür. Bunun
sonucunda günler uzar. Ay'ın her yüzyılda Dünya’ dan 3,8 cm uzaklaşması bu sebeple olur.
3- Ay, Dünya'dan uzaklaşınca sular kıyılara geri gelir. Deniz sularındaki bu gidiş geliş olayı birçok bölgede
kıyı sorunları oluşturur.
4- Akarsu ağızlarında delta oluşumu engellenir.
5-Akarsu vadilerinin ağızlarının tıkanması önlenir.
6-Kıyı kirlenmesi önlenir.
7-Haliçler oluşur. Deniz yükseldiği zaman akarsuların ağız kısımlarına sokulur ve haliç şekli meydana gelir
Bu çeşit kıyılara haliç tipi kıyılar denir.
8-Watt kıyıları oluşur. Deniz alçalınca ortaya çıkan, deniz yükselince ortadan kalkan bu kıyılara watt kıyıları
denir.Deniz, belli aralıklarla alçalıp yükselince kıyı çizgisi değişir. Türkiye’nin çevresindeki denizler iç deniz
olduğu için gelgit genliği azdır. Bu nedenle, Türkiye kıyılarında gelgit’in etkisi hissedilmez.
Bölüm Sonu Soruları
1- Şekilde gösterildiği gibi aralarında r kadar mesafe bulunan m1 ve m2 kütleli
iki gök cisminin birbirine uyguladıkları kuvveti formülsel olarak elde ediniz.
( evrensel çekim sabiti G=6,673.10-11 N.m2/kg2 )

2- Aralarında belirli bir uzaklık bulunan iki kütle arasındaki çekim kuvveti 63000N’dur. Bu iki kütle arasında-
ki uzaklık 7 katına çıkarılırsa aralarındaki yeni çekim kuvveti kaç kat azalır?
3- Dünyanın yüzeyinden 11km kadar yüksekte veya dünyanın merkezinden d=Rdünya+11km uzaklıkta bulu-
nan m kütleli bir cisim üzerine etkiyen çekim ivmesi kaç m/s2 dir?
4- Güneş sistmeinde dolanmakta olan gezegenlerin ortalama yarıçapları ile
dolanma süreleri arasında bir ilişki vardır. Dünya'nın yarıçapı 150 mil-
yon kilometre, Mars'ın yarıçapı 228 milyon kilometredir. Dünya Güneş
etrafındaki bir turunu 1 yılda yani 365 günde tamamladığına göre Mars
bir turunu Dünya dai süreye göre kaç günde tamamlar?

5- Dünya' nın yıl içinde farklı iki tarıhlerdeki Güneşe göre konumu veril-
miştir. Gezegenin toplam enerjisi korunumu ileksini kullanarak Dünya'nın
Günberi'deki kinetik enerjisi U1, Günötede'ki kinetik enerjisi U2 olsun. Bu
enerjilerin oranını döngüsel momentum korunumu formölleri ile bulunuz.

6-
Yerçekim ivmesinin uzaklığa göre değişimi yanda grafiksel olarak verilmiş-
tir. Deniz seviyesindeki bir havaalanından kalkacak olan 130000 kg kütleli
bir uçağın yeryüzünden 11km yükseklikte uçuş yapması için harcaması
gereken enerjiyi ve bu yükseklikteki çekim ivmesini integral methoduyla
bulunuz.
7- M kütleli bir gezegen etrafında başlangıçta R yarıçaplı yörüngede
dolanmakta olan bir haberleşme uydusu 3R uzaklığındaki yeni bir yö-
rüngeye taşınmak isteniyor. Bu iş için gerekli enerjiyi ve iki durumdaki
hızları oranını V1/V2 yi bulunuz.

8- Ömrü tükenmiş Dünya'nın merkezinden 49 km uzaklıktaki yörüngede dolanmakta olan 1500kg kütleli bir
telekominikasyon uydusunu sonsuz uzaklıktaki uzay çöplüğüne atmak için gerekli kurtulma enrjisi kaç Joule
dur? ( Mdünya= 5.9722 × 1024 kg , G=6,673.10-11 N.m2/kg2 )

9- Dünya merkezinden 27km uzaklıktaki bir yörüngeye oturtmak için göderilecek 5000 kg
kütleli bir uydu için gerekli bağlanma enerjisini bulunuz.

10- Dünya ile Ay arasında ortalama merkezden merkeze uzaklık 384.403 km 'dir. Ay' ın kütlesi May= 7.3477 ×
1022 kg ve Dünya'nın kütlesi MDünya= 5.9722 × 1024 kg 'dır. Ay Dünya etrafındaki bir turunu 27,5 günde ta-
mamladığına göre;
a) Ay' ın ortalama çizgisel hızını,
b) Ay' ın toplam enerjisini,
c)Ay' ın kinetik enerjisini bulunuz.
14. BÖLÜM

Harmonik Hareketler

14.1 Basit Harmonik Hareket


14.2 Salınımın Tanımı
14.3 Harmonik Hareket Uygulamaları
14.4 Basit Sarkaç
14.5 Fiziksel Sarkaç
14.6 Harmonik Harekette Enerji
14.7 Sönümlenmiş Salınımlar
14.8 Kuvvetlendirilmiş Salınım ve Rezonans
Bölüm Sonu Soruları
14.1 Basit Harmonik Hareket
Mekanik titreşimler, periyodik salınımlar ve tekrarlı hare-
ketler fizikte harmonik hareketler adı altında incelenir.
Harmonik harekete örnek olarak sarkaçlı duvar saati, yaya
bağlı kütlenin salınımı, krank miline bağlı motor pistonu,
salıncak gibi daha birçok fiziksel örnekler hayatımızda
mevcuttur. Basit harmonik harekete aynı zamanda peri-
yodik hareket de denir. Çünkü belirli bir denge konumu
etrefında yapılan hareket sürekli tekrarlanır.

Harmonik harekette enerji korunum kanunları ve kuvvetlerin denge şartları sıkça kullanılır.Eğer harmonik
hareket süresince sürtünmeler önemsiz ve dış kuvvetler sisteme etki etmiyorsa enerji kaybı yoktur ve buna
sünümsüz harmonik hareket denir. Sünümlü harmonik harekette ise dıştan sisteme etki eden bir sü-
nümleyici etken vardır. Bu sürtünme kuvveti yada diğer dirençler olabilir. Sünümleyici etkenler her salınımda
hareketin süresi ve cismin aldığı yol ile orantılı olarak sistemden enerji kaybına sebep olur.

14.2 Salınımın Tanımı


Durgun haldeyken denge noktası etrafında dışardan verilen bir etki sonucu belirli bir frekansla yer-
değiştirmeye salınım denir. Salınımın periyodu,T, salınım aralığının tam bir tur alınması için geçen
süredir. Salınım frekansı da bir saniyede yapılan salınım sayısıdır (s-1) ve F ile gösterilir. Birimi Hertz’
dir, Hz ile gösterilir.

Harmonik hareket temel olarak düzgün dairesel hareke-


V tin yatay bileşenleri olarak da düşünülebilir. Yanda dairesel
hareket yapan bir cismin çizgisel hızı sabit olduğu halde yatay
düzlemdeki gölgenin hızı değişkendir.
Cisim dairesel pistin en üst ve en alt konumundayken yatay
w a hız bileşenleri maximum olduğu için golgenin hızı orta nok-
r tada Vmax olmaktadır. En uçlarda ise hız sıfır olur ve hareketin
genliği dairesel pistin yarıçapı r ye eşit olur.

Peki bu hareketi nasıl förmüle dönüştürebiliriz?


Yarıçapı r olan düzgün dairesel hareketin taradığı belirli bir
açı sonucunda x mesafesi üçgen eşitliğinden r.cos(a) olur.
Değişken olarak girilen açı a = w.t ve w.T=2p olduğundan
denklemin girdisi açısal hız, w cinsinden yazılmalıdır.
cos(w.t ) değeri +1 ile -1 arasında değişkenlik gösterdiği için
x mesafesini zamana bağlı olarak x(t) = r.cos(w.t + f) diye
tanımlayacağız.

x(t) = r.cos(w.t +f)

açıya bağlı genlik faz farkı


yatay konum açısı
14.3 Harmonik Hareketin Uygulamaları

Şekil 1 Şekil 2 Şekil 3

Harmonik harekette zamana bağlı konum denklemini elde ettikten sonra günlük hayatımızdaki
uygulmalarında sıkça kullandığımız cismin hızı ve ivmesi bizim için önemlidir. Örneğin şekil 3 teki
diske bağlı kolun C noktasının yatak içindeki anlık hızı ve ivmesi elde edilirken bildiğimiz konum
zaman deklemlerinden faydalanacaz.

Zamana bağlı konum denklemimizi x(t) = r.cos(w.t +f) olarak elde etmiştik.
Denklemin türevini alırsak hız denklemini buluruz. dx/dt=V(t)

dx(t)/dt =V(t) V(t)= - wr.sin(w.t +f)


Denklemin bir daha türevini alırsak bu kez zamana bağlı ivmesini buluruz.

d2x(t)/dt2 =dV(t)/dt dV(t)/dt = a(t)

a(t) = -w2r.cos(w.t +f) olur.

Konumun zamana göre değişimi trigonometrik bir fonksiyon


olan cos(q) fonksiyonun davranışı olduğundan bu fonksiyonun
türevleri de aynı frekanstaki dalganın benzeri olacaktır.
Yandaki şekilde de görüldüğü gibi konum, hız ve ivme grafikleri
arasında f=p/2 kadar faz farkı mevcuttur. Bu faz farkı da uza-
nımın maximum olduğu yerde hızın sıfır olduğu uç noktalar,
uzanımın sıfır olduğu yani denge konumunda hızın maximum
olduğunu denklem matematiksel gösterir gösterir. Aynı şekilde
ivmenin değeri de konum gibi cos(q) denklemine bağlı olduğu
için ivmenin maksimum olduğu nokta uç noktalar yani genliğin
en fazla olduğu hızın da sıfır olduğu noktalardır. Bu ilişki F=ma
dan da anlaşılabilir. Çünkü kütle sabit olduğu için yaydaki geri
çağırıcı kuvvet Dx in en fazla olduğu anda gerçekleşir ve ivme
a=F/m den bu noktada maksimumdur.
14.4 Basit Sarkaç
Genel bir tanımlama yaparsak yandaki şekilde görüldüğü gibi sarkaç, m kütleli
bir cismin tavanda sabit bir noktaya bağlı kütlesi ünemsiz esnemeyen bir ipin ucu-
na bağlanarak denge konumu etrafındaki periyodik hareketidir.
Sarkacın açı verildekten sonra denge konumuna gelmesini sağlayan kuvvet
cismin ağırlığının mgsin(q) bileşenidir. Bu bileşene aynı zamanda geri çağırıcı
kuvvet de denir. Ağırlığın mgcos(q) da ipteki gerilme kuvveti oluşturan bileşe-
nidir. Cisim denge konumuna gelmek istediği için zamanla açı küçülür ve sin(q)
değeri de küçülür. Bu değerin denge konumuna kadar küçülmesi sarkacın geri
çağırıcı kuvvetinin de azalıp denge konumunda sıfıra eşit olduğu an olur.

Burda önemli olan nokta sarkacın bir uçtan diğer uca gidince arada geçen süre-
nin bir tam periyot değil de periyodun yarısı, T/2 olduğudur. Yani cisim sağ uçtan
hareket ettikten sonra tekrar aynı noktaya gelince tam bir tur atmış oluyor ve bu
süre T ye eşit olur.

14.5 Fiziksel Sarkaç


Fiziksel sarkaç doğadaki herhangi asılı bir cismin salınımı sonucu yaptığı hareket olabilir. Buna örnek
duvar saatinin sarkacı yada arabanın dikiz aynasına asılı nazar boncuğu olarak verilebilir.
Salınım yapan cismin ağırlık merkezi tespit edilir ve dönme noktasına olan uzklığı hesaplanır.
Cismin ağırlığı mg nin dönme noktasına göre torku aynı zamanda eylemsizlik momentinin merkezcil
ivmesiyle oluşturduğu torka eşit olur.
mg.sin(q)L= I.a

Sistemin ivmesi, a, açı değişiminin dq zamana göre ikinci türevi alınarak bulunur.

Düzenleme sonucu fiziksel sarkacın küçük açılar için temel frekans ve periyot formülleri elde
edilir.
14.6 Harmonik Harekette Enerji
Bölümün başında da denildiği gibi basit harmonik harekette enerji
korunumu önemlidir. Sünümsüz sistemlerde enerji her zaman korunur
ve sistem kararlıdır. Yandaki şekilde yaya bağlı m kütleli cismin yaptığı
periyodik harekette hız değiştiği için kinetik enerjisi de değişir. Hızın
maximum olduğu denge konumunda sistemin bütün enerjisi kinetik
enerjiye dünüşür. Genliğin en fazla olduğu uçlarda da yay gerildiği için
K=Kinetik enerji U=Potansiyel enerji
enerji yayda potansiyel enerji olarak depolanmıştır.

Newton kanununa göre sisteme etkiyen net kuvvetin toplam kütle ile
sistemin ivmesi çarpımı F=m.a=-(mw2).x ve F=k.x den
k=mw2
w=(k/m)1/2 den

Hareketin periyodu saniye cinsinden


bulunur

Yayda depolanan potansiyel enerji cismin hızının olmadığı uçlarda enfazladır ve E= (1/2)k.x2 kadardır.
Bu enerji cisim denge konumundan yayın ne gergin nede sıkışmış olduğu durumdan geçerkenki toplam
kinetik enerjiye dönüşür.
(1/2)k.x2=(1/2)m.V2 denlemi yayın temel enerji eşitliği olur.

Basit sarkaç süzeneğindeki denklemler salınımın gerçekleştiği noktayay göre


torkların eşitliği ile bulunur.
Geri çağırıcı kuvvet mgsin(q) kadardır ve oluşturduğu t=mg.sin(q) torkunu
I.q ya eşitlersek mg.sin(q)L= I.a olur. burdaki I (I=mL2): cismin eylemsizlik
momenti, a: sistemdeki merkezcil ivmedir. Bu eşitlik sonucunda denklemler
düzeltilince
sarkacın periyodu elde edilir.

Sarkaç düzeneğindeki enerji kourunumu m kütleli cismin yükseklik potansi-


yel enerjisinin denge konumundaki kinetik enerjiye dönüşmesiyle sağlanır.

Cisim en uçtayken yükseklik farkı h=L-Lcos(q) kadar olur. Bu konumdaki potansiyel enerjiyi kinetik
enerjiye dönüştürürsek

mg(L-Lcosqmax)= (1/2)m.V2max basit sarkacın temel enerji denklemi olur.


14.7 Sönümlenmiş Salınımlar

Titreşimlerin sönümlenmesi, görültünün azaltılması güvenli bir sürüş için önemli


etkenlerdir. Seyahat halindeyken aracın kasisli yollarda nasıl sallandığını, bu salı-
nımın kazaya sebep olabileceğini uzmanlar bilimsel çalışmalarla belirlemiş ve buna
göre çözümler üretmiştir. Araba rotlarına takılan süspansiyonlar bu titreşimleri
azaltmak için üretilmiştir. İdeal sistemlerde sürtünme olmadığı için enerjinin ko-
runduğunu söylemiştik. Sürtünmenin sebep olduğu enerji kaybı hareketin sünüm-
lenmesine sebep olur ve sistem zamanla denge konumuna yaklaşır. Dışardan bir
kuvvet etkisiyle genliğin alarak hareketsiz hale gelmesiyle oluşan harmonik hareket-
lere sünümlü harmonik hareket denir. Hareketi sünümleyen kuvvete sünümleyici
kuvvet (F) ve sisteme olan etkisine sünümleme katsayısı (b) denir.

Sünümleyici kuvvetler genelde sürtünme kuvvetidir. Havada salınım yapan bir


cisim ile suda salınan bir cisme aynı oranda sürtünme etki etmez. Suyun visko-
zitesi havanınkinden fazla olduğu için sudaki sürtünme daha fazla olur.
k Sürtünme kuvvetinin formülünü (Fs=0.5k.A.V2 ) hatırlarsak ortamın katsayı-
x sı(k), hareket eden cismin kesit alanı(A) ve hızın karesiyle(V2 ) doğru orantılı
olduğu görülür. Yandaki şekilde salınım yapan sistemin sünümleme etkisi su-
m yun sünümleme katsayısı(b), sistemin hızının karesi (V=dx/dt)2 ve su içindeki
kısmın alanı(A) ile doğru orantılıdır. Sürtünme kuvveti sistemin hareketine ters
etki ettiği için negatif işaretle işleme katacağız.
su

Bu denklem bir differamsiyel denklem olduğundan çözümü trigonomerik ve üstel bir fonk-
siyonu kombinasyonu olur.

Şeklinde sünümleme
frekansları elde edilir.

sünümleme
faz farkı Burdaki w0 sistemin sünümleme olmadığı durumdaki açısal frekansıdır.
katsayısı açısal
ilk genlik frekans

Sünümlü Harekette Enerji


Sünümlü harektlerde mekanik enerji zamanla sürtünmeye dünüşerek sı-
fırlanır. Sistemin toplam mekanik enerjisi formülü
ile ifade edilir.
Birim zamanda kaybolan enerji bulunurken dx/dt=V ve dv/dt= a dönüşü-
mü yapılır ve ,
formül-
leri elde edilir.
Böylece ivme yerine yazarak

temel denklemi şeklinde ifade ederiz.


14.8 Kuvvetlendirilmiş Salınım ve Rezonans
Salıncakta sallanan kardeşinizin daha çok sallanması için vermeniz gereken enerji salınımın yönüne ve
salınım frekansına bağlı olduğunu bu bölümü okuyunca daha iyi anlayacaksınız. Sisteme dışardan verilen
ve sistemin salınımını arttırıcı etkenlere itici kuvvetler denir. Bu kuvvetın frekansı salınımın doğal frekan-
sı(w) ile özdeş olmalıdır. Yani dişardan verdiğimiz enerjiyi arttırıcı etki sistemin denge konumunun T/4
ve 3T/4 anlarında verilmelidir. Bu kuvvetlendirici etkenler denklemde genliğin artışına sebep olur yada
sürtünmeli sistemlerde sistemin başlangıç genliğinde sabit kalmasını sağlarlar. Demekki kuvvetlendirici
kuvvet zamana bağlı sürekli olarak şeklinde periyodik ya da T/4 ve 3T/4 anla-
rında anlık kuvvetler olmalıdır.

diffreansiyel denklemin çözümüyle kuvvetlendiricinin genliğini

olarak elde ederiz.

Rezonans ve Sonuçları

Rezonans günlük hayatımızda coğu zaman karşılaştığımız bir olaydır. Müzik aletlerindeki ses sistemlerin
görültüsünü ayarlarken, dinlemek istediğimiz radyonun frekansını girerken, elektrik devrelerinde akımı
düzenlerken vb. olaylarda rezonansın fiziksel uygulamasını yani kuvvetlendiricinin frekansını salınımın
yada dalgaların doğal frekansına eşitliyoruz. Tanımolarak, genliği salınımın genliği(A0) seviyesinde tutar-
ken kuvvetlendiricinin freknsını doğal frekansa eşitlemeye rezonans denir.

1940 ta rüzgarın türbülanslı frekansı ve Rüzgarın kuvvetlendirici etkisi yüzünden küprü


Tacoma Narrows Küprüsü nün salınım yıkılır.
frekansı rezonansa girer.
1. Yarıçapı 3 m olan dairesel bir pistte 8 rad/s açısal hızla saat yönünün tersine hareket eden bir oyuncak ara-
banın yatay ( x-koordinatı) konumunu veren denklemi türetiniz. Bulduğunuz denklemde t=0 anında araba
P(3, 0) noktasında ve kuzeye gidiyorsa arabanın t=8 saniyesindeki ivmesi kaç m/s^2 dir?
ÇÖZÜM
Dairesel bir pistte sabit açısal hızla hareket eden cisim aynı zamanda sabit çizgisel hızla da hareket ediyor-
dur.
Çizgisel hız: V= r x w formülünden bulunacaktır. V = (3 m)x(8rad/s) , V=24 m/s
Dairesel hareketin yatay eksendeki izdüşümü de basit harmonik hareket olacakatır.
x(t)= r.cos ( w.t) formülünden t=0 iken x=3 oluyorsa r=3 bulu-
nur. Buradan konumunu veren denklem x(t)= 3 cos ( 8.t) olur.
(cvp)

İvmeyi veren denklem ise konum denkleminin iki defa zamana


(t) göre türevi alınmasıyla bulunur. İlkin türev alındığında cismin
hız denklemi sonra bir kez daha türev alındığında ivme denklemi
bulunur.
dx/dt= V(t)= -24.t. sin ( 8.t) dV/dt= a(t)= -192.t^2.cos(8.t) ivme
denklemi bulunur.
t=8 denkleme yazılırsa a(8)= -192.8^2.cos(8.8)= -4815 m/s^2 .
(cvp)

2. Yatay konumdaki sürtünmesiz bir düzlemde yayın ucuna bağlı 2 kg kütleli bir cisim genliği A=10cm olacak
şekilde basit harmonik hareket yapıyor. Başlangıç, t=0 , anında hızı maximum ve sağa doğru V=+2 m/s dir.
a- Yayın salınım frekansı kaç Hz’ dir?
b- Yay sabiti, k , kaç N/m dir?

ÇÖZÜM
Yaya bağlı cisimlerin basit harmonik hereketlerini incelerken yayın salınım periyodunu veren T=2 π (m/k)^1/2
ve konumunu veren x(t)= A.cos ( w.t) formüllerini kullanacağız. Buradan F=1/T ve w =2 π f den frekansı
buluruz.
A=10cm=0.1m , x(t)=0.1 cos(w.t+a) ve t=0 için x=0 başlangıç koşuluyla faz farkı a=π/2 bulunur.
(dx/dt)=V(t)=-0.1w.sin(w.t+π/2) formülünden de V(t=0)=+2m/s başlangıç koşuluyla w=20 rad/s bulunur.
w =2 π F den de F= 3.18 Hz bulunur. (cvp.a)

T=1/F den T=0.31418 s bulunur T=2 π (m/k)^1/2 dan da k= 800 N/m bulunur. (cvp.b)

3. Bir yay üzerindeki bir kütle aşağı yukarı


titreşim hareketler yapmaktadır. Kütlenin
konumu zamanın bir fonksiyonu olarak
aşağıda verilmiştir. Verilen noktakardan
hangisinde hız pozitif ve ivme negatiftir?

Çözüm
Grafik konumun zaman bağlı fonksiyonu olduğuna için dx/dt=V(t) ve d2x/dt2=dV/dt= a(t) kriterlerine
bakacağız.

cevap = C
4. Düşey doğrultudaki k sabitli bir yaya asılı m kütleli cisim denge konumundan d
kadar yukarı kaldırılıp t=0 anında serbest bırakılıyor.
a- Hız denkleminin zamana göre fonksiyonu k, m ve d cinsinden yazınız.
b- İvme denkleminin zamana göre fonksiyonunu yazarakt= 2 π (m/k)^0.5 anında-
ki ivmeyi bulunuz.

ÇÖZÜM
Öncelikle cismin düşey konumunun zamana göre fonksiyonu ve salınım frekansı-
nı bulalım.
T=1/F= 2π.(m/k)^0.5 , y(t)=Asin(2π.F.t + a) y(t=0)=d başlangıç koşuluyla
d=d.sin(2π.F.0 +a) , a= π/2 faz farkı olarak bulunur. Düşey konum denklemimiz
y(t)=d.sin((k/m)^0.5 .t + π/2) olur. Hız denklemimiz bu denklemin zamana göre
türevi alınmış halidir. (dy/dt)=V(t)= d. (k/m)^0.5cos((k/m)^0.5.t + π/2). (cvp.a)

İvme denlemi de hız denkleminin zamana göre türevi olduğu için


(dV/dt)=a(t)=-d.(k/m).sin((k/m)^0.5 .t + π/2) olur ve t= 2 π (m/k)^0.5 değeri için
a(t= 2 π (m/k)^0.5 )=-d.(k/m).sin(2 π+ π/2) a=dk/m olur. (cvp.b)

5. İp uzunluğu 3m olan bir salıncakta oturan çocuğun salıncakla


birlikte kütlesi 48 kg dır. Denge konumuna göre salıncak ipi düşeyle
π/6 açı yapacakşekilde annesi tarafından çekilip serbes bırakılıyor.

a- Salıncağın salınım periyodunu, T’ yi, bulunuz.


b- t=T/3 saniye sonra salıncak ipi denge konumuna göre kaç derece
açı yapar?
c- Salıncağın denge konumundan geçerken ki çizgisel hızı kaç m/s
dir?
d- Çocuk ayağa kalkınca toplam kütle merkezi 0.3m yukarı kaydığı-
na göre yeni durumdaki salınım periyodunu bulunuz.

Salıncağın hareketi basit sarkaçın harmonik hareketi olarak incelenecektir. Öncelikle sarlacın periyodu salı-
nım yapan cismin kütlesine bağlı bağlı olmadığı unutulmamalıdır. Ayrıca bir salınımda taranan açı 2π değil
de 2π/3 alınmalıdır. T= (2π/3).(L/g)^0.5 ve g=9.81 m/s2 alınmalıdır.
T= 2π.(3/9.81)^0.5 = 1.16 s bulunur. (cvp. a)
w= 2π/T w=5.424 rad/s , @0= π/6 , @= @0.cos(wt) @=(π/6).cos(5.424 x (1.16/3))= 2π/9=400 (cvp.b)

Denge konumuna gelince toplam kütlenin kaybettiği potansiyel enerji kinetik


enerjiye dönüşür. mgh= 0.5 m V^2 , V=(2x9.81x0.4)^0.5=2.8m/s (cvp.c)

Çocuk ayağa kalkınca kütle merkezi 0.3m yukarı kaydığına göre sarkacın yeni
ip uzunluğu 3m-0.3m=2.7m olur.
Yeni durumdaki salınım periyodu T2= (2π/3).(L/g)^0.5 , T2=(2π/3).(2.7/9.81)^0.5
T2= 1.1 s olur. (cvp.d)
Bölüm Sonu Soruları
1- Rijit bir cisme ait harmonik hareket denkleminde konumun zamanla değişim fonksiyonu
x(t)=C.cos(ω0t)+D.sin(ω0t) şekline iki trigonometrik cebirsel ifadeyi tek fonksiyon olan x(t)= Acos( ω0t+φ)
şekline dönüştürülüyor. A başkatsayısı ve φ faz farkı açısı C ve D cinsinden bulunuz.

2- Kütlesi m olan rijit bir cisim yay sabiti k olan yayın ucuna bağlanarak sürtünmesiz yatay düzlemde haraket
ediyor. Yayın çekildiği t=0 anında denge konumundan yay maksimum x0>0 kadar çıldığına göre cismin hare-
ket periyodunu ve denge konumundan geçerken sahip olduğu hızı m, k ve x0 cinsinden bulunuz.
3- Şekildeki gibi k=250 N/m rijitli yayın ucunda tutulan m=5kg’lık cisim denge
konumundan itibare 20cm sıkıştırılarak serbest bırakılıyor.Sürtünmesiz yatay
düzlemde cisim geri dönerken yayın itki kuvveti ile +i yönünde atılırken sahip
olacağı hız kaç m/s olur?
4- P noktası çevresinde serbestçe dönebilen m=0.9kg kütleli cisim L= 0.75 m uzunlu-
ğuna sahip iple düşeyde q0= 300 açı yapacak şekilde çekilip bırakılıyor.(g= 9.81m/s2)
a) Cismin salınım frekansını ve salınım periyodunu bulunuz.
b) Cisim denge konumundan geçerken sahip olduğu kinetik enerjiyi bulunuz.
c) P noktasının 0.25m aşağısına ipin takılacağı şekilde bir çivi çakılmasıyla yeni
dönme noktası Ç olacağını varsayarak düşeyle yapılan maksimum açıyı bulunuz.

5- Sürtünmesiz yatay düzlemde m= 7kg kütleli , R= 10cm yarıçaplı bir silindir


k=450 N/m lik yaya merkezinden bağlanarak kaymadan dönüyor. Silindir
denge konumundan 8cm çekilip bırakılıyor. Silindirin denge konumundan
geçerken sahip olduğu açısal hızını bulunuz. ( I=0.5mR2)
6- İki tarafından atmosfer basıncına açık olan A= 6cm2 kesit alanına sahip U borusunda
d=1.5 g/cm3 özkütleli sıvı bulunmaktadır. Sıvı, borunun sol kolunda piston yardımıyla
denge konumundan x0 = 20cm kadar itilip serbest bırakılarak harmonik hareket yap-
ması sağlanıyor. Sürtünme ve viskoz kuvvetlerini ihmal ederek svınının boru içindeki
salınım frekansını bulunuz.(g= 9.81m/s2)

7- Yandaki sistemde m=3kg kütleli cisim k= 300N/m sabitli yaya bağlan-


mıştır. Cisim sönümlü harmonik hareket yaparak durgun hale gelecek-
tir. Sistemin sönümleme katsayısı c=0.3 ve yavaşlatıcı sürtünme kuvveti
Fs=-c.V2 cinsinden verildiğine göre yay denge konumundan 10cm sıkıştı-
rılarak serbest bırakılınca kaç saniye sonra cisim durgun hale gelir?

8- İlk boyu 2L ve rijitliği k olan yayın ucundaki m küteli cisim T periyotluk


harmonik hareket yapmaktadır. Yay ortadan ikiye ayrılıp parallel olarak
şekildeki gibi bağlanınca cismin yeni salınım periyodu kaç T olur?

9- Lastiğindeki amortisörün sönümleme katsayısı c=0.7 olan bir otomobil tümsekten


geçerken tekerleği F=1500N’luk bir tepki kuvvetine maruz kalıyor. Lastiğin salınım
denklemi x(t) = 30e-0.7tcos(12t) olduğuna göre spiral yayın katsayısını, k, ve amortisör-
deki darbe emici sıvının sürtünmeden dolayı kazanacağı enerjiyi bulunuz.

10-

Yandaki grafiğe bakarak sistamin hareket denkle-


mini elde ediniz. Denklemi bulurken sönümleme
katsayısını, c, salınım frekansını,wd ve denge
konumuna gelme süresini, ts’yi kullanarak elde
edileceğini unutmayınız.

You might also like